Sunteți pe pagina 1din 112

DEPARTAMENTUL

MATEMATICĂ-INFORMATICĂ
UNIVERSITATEA DIN PITES, TI

MATINF
PUBLICAT, IE BIANUALĂ
DE MATEMATICĂ S, I INFORMATICĂ
PENTRU ELEVI S, I PROFESORI

Anul III, nr. 5 / 2020

ISSN 2601-9426
ISSN-L 2601-8829

Editura
Universităt, ii din Pites, ti
Editată de: DEPARTAMENTUL MATEMATICĂ-INFORMATICĂ,
UNIVERSITATEA DIN PITES, TI
Comitetul de redact, ie:

Stelian Corneliu ANDRONESCU Eduard ASADURIAN


Tudor BĂLĂNESCU Costel BĂLCĂU - redactor s, ef
Loredana BĂLILESCU Doru CONSTANTIN
Şerban COSTEA Laurent, iu DEACONU
Maria-Crina DIACONU Ionut, DINCĂ
Mihaela DUMITRACHE Mihai Armand IONESCU
Florentin IPATE Constantin GEORGESCU
Raluca Mihaela GEORGESCU Camelia GHELDIU
Marius MACARIE Maria MIROIU
Gheorghe NISTOR Antonio Mihail NUICĂ
Viorel PĂUN Doru Anastasiu POPESCU
Marin POPESCU Nicolae-Doru STĂNESCU
Alina Florentina S, TEFAN Cristina TUDOSE
Adrian ŢURCANU Corneliu UDREA

Tehnoredactare computerizată: Mihail TĂNASE, e-mail: mihaimit@yahoo.it

Redact, ia: Departamentul Matematică-Informatică, Universitatea din Pites, ti, Str. Târgu
din Vale, nr. 1, Pites, ti, tel. 0348453247, e-mail: revista.matinf@upit.ro
Forma digitală a revistei poate fi accesată la adresa: http://matinf.upit.ro

Publicată de: Editura Universităt, ii din Pites, ti, https://www.upit.ro/ro/relatii-cu-mediul-


socio-economic/centre-suport/editura

Anul III, Nr. 5, Iulie 2020


Cuprins 3

Cuprins

ARTICOLE S, I NOTE DE MATEMATICĂ 5

L. Giugiuc, A.D. Pı̂rvuceanu


A generalisation of a problem from Gazeta Matematică . . . . . . . . . . . . . . . . . . . 5

D. Mărghidanu
O inegalitate pentru funcţii convexe/concave . . . . . . . . . . . . . . . . . . . . . . . . . 7

D. Văcaru
O metodă pentru a demonstra unele inegalităt, i . . . . . . . . . . . . . . . . . . . . . . . 12

ARTICOLE S, I NOTE DE INFORMATICĂ 15

C. Bălcău
Complexitatea algoritmului de sortare prin interclasare (mergesort) . . . . . . . . . . . . 15

M. Cristea
Protocoale . . . . . . . . . . . . . . . . . . . . . . . . . . . . . . . . . . . . . . . . . . . . 25

M.R. Găman, M. Miroiu


Concursul ,,Dräexlmaier IT Day”, Edit, ia a II-a, ianuarie 2020 . . . . . . . . . . . . . . . 34

RUBRICA DE ROBOTICĂ 41

D.A. Popescu
Repetarea operat, iilor unui robot Mindstorms Education EV3 . . . . . . . . . . . . . . . 41

PROBLEME DE MATEMATICĂ PENTRU EXAMENE 44


Teste pentru examenul de Evaluare Nat, ională . . . . . . . . . . . . . . . . . . . . . . . 44
Teste pentru examenul de Bacalaureat, specializarea S, tiint, e ale naturii . . . . . . . . . . 54
Teste pentru examenul de Bacalaureat, specializarea Matematică-Informatică . . . . . . 59
Teste pentru admiterea la facultate . . . . . . . . . . . . . . . . . . . . . . . . . . . . . 64
Teste grilă pentru admiterea la facultate . . . . . . . . . . . . . . . . . . . . . . . . . . 67

PROBLEME DE INFORMATICĂ PENTRU EXAMENE 74


Teste pentru examenul de Bacalaureat, specializarea S, tiint, e ale naturii . . . . . . . . . . 74
Teste pentru examenul de Bacalaureat, specializarea Matematică-Informatică . . . . . . 79

PROBLEME DE MATEMATICĂ PENTRU CONCURSURI 85


Rezolvarea problemelor pentru liceu din MATINF nr. 3 . . . . . . . . . . . . . . . . . . 85
Probleme propuse pentru liceu . . . . . . . . . . . . . . . . . . . . . . . . . . . . . . . . 95

PROBLEME DE INFORMATICĂ PENTRU CONCURSURI 99


Probleme propuse . . . . . . . . . . . . . . . . . . . . . . . . . . . . . . . . . . . . . . . 99
4 Cuprins

ISTORIOARE DIN LUMEA MATEMATICII S, I A INFORMATICII 109

S.C. Andronescu
Carl Friedrich Gauss, ,,. . .teoria numerelor este regina matematicii” . . . . . . . . . . . . 109
ARTICOLE S, I NOTE DE MATEMATICĂ

A generalisation of a problem from Gazeta Matematică

Leonard Giugiuc 1 and Alexandru Daniel Pı̂rvuceanu 2

The following is the problem 27767 from Gazeta Matematică, No. 11/2019:
27767. Let f : [0, 1] → R be a differentiable function with the property that
Z 1
f (x)dx = 0.
0

R1
Prove that there exists c ∈ (0, 1) such that f 0 (c) = −9 3
0
f (t)dt.

Marian Andronache and Dinu S, erbănescu, Bucharest

While we were solving this problem, we observed that a more general result holds. What is
interesting is that the official solution published in [2] can be adapted to prove this more general
result. In this article, we aim to present this generalisation, followed by our solution:
Problem. Let a, b ∈ R, a < b. Consider a differentiable function f : [a, b] → R such that
Z b
f (x)dx = 0.
a

Prove that for all x ∈ [a, b] there exists cx ∈ (a, b) such that
Z x
2 f (t)dt = (x − a)(x − b)f 0 (cx ).
a

Solution. If x = a or x = b, then there is nothing to prove, since the conclusion rewrites as


0 = 0.
For x ∈ (a, b), the conclusion is equivalent to proving that there exists cx ∈ (a, b) such that
Z x
2 f (t)dt
a
= f 0 (cx ).
(x − a)(x − b)
Z y
Consider the function F : [a, b] → R, F (y) = f (t)dt.
a
1
Professor, National ,,Traian” College, Drobeta-Turnu Severin, Romania, leonardgiugiuc@yahoo.com
2
Student, Faculty of Mathematics and Computer Science, University of Bucharest, Bucharest, Romania,
pirvuceanualexandrudaniel@gmail.com.

5
6 L. Giugiuc, A.D. Pı̂rvuceanu

Z x
2 f (t)dt
a
For an arbitrary x ∈ (a, b), let kx := .
(x − a)(x − b)
From Cauchy’s Mean Value Theorem on the intervals [a, x] and [x, b] we get that ∃ux ∈ (a, x)
and vx ∈ (x, b) such that :
1
F (x)(2ux − a − b) = (x − a)(x − b)f (ux ) ⇐⇒ f (ux ) = (2ux − a − b)kx
2
1
F (x)(2vx − a − b) = (x − a)(x − b)f (vx ) ⇐⇒ f (vx ) = (2vx − a − b)kx
2
From Lagrange’s Mean Value Theorem on [ux , vx ] we get that ∃cx ∈ (ux , vx ) ⊂ (a, b) such
that
kx 2vx −a−b 2ux −a−b

f (vx ) − f (ux ) 0 0 2
− 2
= f (cx ) ⇐⇒ f (cx ) = = kx ,
vx − ux vx − ux
which is the desired conclusion.
References
[1] Gazeta Matematică, No. 11/2019.
[2] Gazeta Matematică, No. 5/2020.
O inegalitate pentru funcţii convexe/concave

Dorin Mărghidanu 1

Pornind de la inegalitatea ponderată a lui Jensen se obt, ine o inegalitate foarte interesantă.
Prin particularizarea acestei inegalităt, i la diferite funct, ii convexe, se obt, in numeroase aplicat, ii.
Inegalitatea lui Jensen este foarte cunoscută s, i utilizată. În lumea inegalităt, ilor ea este chiar
considerată drept ,,regina inegalităt, ilor”, deoarece este o inegalitate foarte puternică. Multe din
inegalităt, ile celebre – de exemplu: inegalitatea mediilor, inegalitatea CBS, inegalitatea Young,
inegalitatea Hölder, inegalitatea Bergström, inegalitatea Radon, inegalitatea Huygens, etc. sunt
cazuri particulare sau se pot demonstra cu ajutorul inegalităt, ii lui Jensen sau sunt echivalente
cu inegalitatea lui Jensen (v. [1] - [3], [8], [9]). Ea are o enunt, are foarte simplă:

Teorema 1. (Inegalitatea ponderată a lui Jensen [1]) Dacă f : I ⊆ R → R este o funct, ie


convexă, I-interval, atunci !
Xn Xn
wk f (xk ) ≥ f w k xk (J)
k=1 k=1
n
P n
P
unde wk > 0, xk ∈ I, ∀k ∈ {1, 2, . . . , n}, wk xk ∈ I, wk = 1.
k=1 k=1

Dacă f este o funct, ie concavă pe I, semnul de inegalitate din (J) se inversează.


Egalitatea ı̂n (J) are loc dacă s, i numai dacă x1 = x2 = . . . = xn , sau când funct, ia f este
funct, ie liniară (afină).
Pornind de la această inegalitate celebră, vom obt, ine o nouă inegalitate pentru funct, ii
convexe, respectiv funct, ii concave.

Teorema 2. (Inegalitate de convexitate/concavitate) Dacă f : I ⊆ (0, ∞) → R este o funct, ie


S − ak
convexă pe intervalul I, astfel ı̂ncât ak > 0, ∈ I, ∀k ∈ {1, 2, . . . , n}, n − 1 ∈ I, iar
ak
Pn
ak = S atunci are loc inegalitatea
k=1

n  ‹
X S − ak
ak f ≥ f (n − 1) · S . (C)
k=1
ak

Dacă f este o funct, ie concavă pe I, semnul de inegalitate din (C) se inversează.


Egalitatea are loc dacă s, i numai dacă a1 = a2 = . . . = an , sau când funct, ia f este funct, ie
liniară.
1
Profesor dr., Colegiul Nat, ional ,,Al. I. Cuza”, Corabia, d.marghidanu@gmail.com

7
8 D. Mărghidanu

S − ak ak
Demonstraţie. În (J) luăm xk = s, i ponderile wk = P
n , pentru care avem evident:
ak
ak
k=1
n
P
wk > 0, ∀k ∈ {1, 2, . . . , n}, wk = 1. Cu aceste substitut, ii avem:
k=1

n n  ‹ n  ‹
X X ak S − ak 1X S − ak
wk f (xk ) = n ·f = ak f , (1)
P a k S a k
k=1 k=1 aj k=1
j=1

† 
n
! n n
! n 
!
X X ak S − ak X S − ak X ak 
f wk xk =f n · =f =f 1− = f (n−1).
P ak S S
k=1 k=1 aj k=1 k=1
j=1
(2)
Cu (1), (2) ı̂n inegalitatea lui Jensen, inegalitatea (C) rezultă imediat.
S − a1 S − a2 S − an
Egalitatea are loc dacă = = ... = ⇔ a1 = a2 = . . . = an .
a1 a2 an

Aplicat, ii

Alegând ı̂n mod potrivit diferite funct, ii convexe / concave ı̂n inegalitatea (C), se pot obt, ine
numeroase inegalităt, i. În cele ce urmează vom exemplifica – mai degrabă ı̂n scop paradigmatic –
prin câteva inegalităt, i specifice. Evident, pentru rezultatele prezentate pot exista s, i alte tipuri
de demonstrare. Este ceea ce am observat prin propunerea unora dintre ele ı̂n diferite grupuri
de matematică, on-line, v. [4]-[7], s, i unde au fost oferite felurite solut, ionări.

Propozit, ia 1. Dacă a1 , a2 , . . . , an > 0, p ≥ 1 sau p < 0 s, i a1 + a2 + . . . + an = S, atunci

(S − a1 )p (S − a2 )p (S − an )p
p−1 + p−1 + . . . + p−1 ≥ (n − 1)p S. (3)
a1 a2 an

Dacă 0 < p < 1, inegalitatea din (3) este inversată.

Demonstraţie. Se consideră funct, ia f : (0, ∞) → R, f (x) = xp care pentru p ∈ (−∞, 0] ∪ [1, ∞)


este funct, ie convexă.
Aplicând inegalitatea (C) se obt, ine inegalitatea (3).
Pentru 0 < p < 1, funct, ia f este concavă, deci inegalitatea (3) este inversată.

Observat, ia 1. Inegalitatea (3) se poate demonstra s, i cu inegalitatea lui Radon.

Corolarul 1. Pentru a, b, c > 0, p ≥ 1, avem:

(a+b)p (b+c)p (c+a)p p


+ p−1 + p−1 ≥2 · (a+b+c) . (4)
cp−1 a b
O inegalitate pentru funcţii convexe/concave 9

Corolarul 2. ([4]) Pentru a, b, c ≥ 0, avem:


È È È √
a · (b+c)3 + b · (c+a)3 + c · (a+b)3 ≤ 8 · (a+b+c) .
4 4 4 4
(5)

3
Demonstraţie. Se aplică inegalitatea (C) pentru funct, ia f : (0, ∞) → R, f (x) = x 4 care este
funct, ie strict concavă. Egalitatea are loc dacă

b+c c+a a+b


= = ⇔a=b=c.
a b c

Corolarul 3. ([5]) Dacă a1 , a2 , . . . , an > 0 s, i a1 + a2 + . . . + an = S, atunci


É É É
a1 a2 an S
a1 · + a2 · + . . . + an · ≥√ (6)
S − a1 S − a2 S − an n−1

1 1
Demonstraţie. Considerăm funct, ia f : (0, ∞) → R, f (x) = √ = x− 2 , care este funct, ie strict
x
convexă pe 0, ∞), pentru care se aplică inegalitatea (C).

Propozit, ia 2. ([6]) Dacă a1 , a2 , . . . , an > 0 s, i a1 + a2 + . . . + an = S, atunci

(S − a1 )(S − 2a1 ) (S − a2 )(S − 2a2 ) (S − an )(S − 2an )


2
+ 2
+ ... + ≥ n(n − 1)(n − 2). (7)
a1 a2 a2n

Demonstraţie. Fie funct, ia f : [0, ∞) → R, f (x) = x3 − x, pentru care avem f 00 (x) = 6x ≥ 0,


deci funct, ia f este convexă pe intervalul [0, ∞).
Atunci, aplicând inegalitatea (C), avem succesiv:
n  ‹
X S − ak
ak f ≥ f (n − 1) · S
ak
n –
k=1
‹2 ™
X S − ak S − ak
≥ (n − 1) (n − 1)2 − 1 · S
 
⇔ ak · −1
k=1
ak ak
n
X S 2 − 2Sak
⇔ (S − ak ) 2
≥ (n − 1)[(n − 1)2 − 1] · S
k=1
ak
n
X S − 2ak
⇔S (S − ak ) 2
≥ n(n − 1)(n − 2) · S
k=1
a k
n
X (S − ak )(S − 2ak )
⇔ ≥ n(n − 1)(n − 2).
k=1
a2k

Egalitatea are loc dacă s, i numai dacă a1 = a2 = . . . = an .

Propozit, ia 3. Dacă a1 , a2 , . . . , an > 0 s, i a1 + a2 + . . . + an = S, atunci


È È È √
(S − a1 )2 + a21 + (S − a2 )2 + a22 + . . . + (S − an )2 + a2n ≥ n2 − 2n + 2 · S. (8)
10 D. Mărghidanu


Demonstraţie. Considerăm funct, ia f : (0, ∞) → R, f (x) = x2 + 1, pentru care avem f 00 (x) =
1√
(x2 +1) x2 +1
> 0, deci funct, ia f este strict convexă.
 ‹ ʁ ‹
n S − ak n S − ak 2 n p
(S − ak )2 + a2k , s, i f (n − 1) · S =
P P P
Cum ak f = ak +1=
√ k=1 a k k=1 a k k=1
n2 − 2n + 2 · S, cu inegalitatea (C) rezultă s, i inegalitatea (8).
Egalitatea are loc dacă s, i numai dacă a1 = a2 = . . . = an .
Observat, ia 2. Inegalitatea (8) se poate demonstra s, i cu inegalitatea lui Minkowski.
Propozit, ia 4. Dacă a1 , a2 , . . . , an > 0 s, i a1 + a2 + . . . + an = S, atunci
a31 a32 a3n 1
+ + ... + ≥ · S 2. (9)
S − a1 S − a2 S − an (n − 1)n

1
Demonstraţie. Funct, ia f : (0, ∞) → R, f (x) = x(x+1)
este o funct, ie convexă.
n € Š n n
a3k
P S−ak P
 1 1
P 1
Cum ak f ak
= ak · S−ak S−ak
 = S
· S−ak
, iar f (n − 1) · S = (n−1)n
· S, cu
k=1 k=1 ak a
+1 k=1
k
inegalitatea (C) rezultă inegalitatea (9).
Egalitatea are loc dacă s, i numai dacă a1 = a2 = . . . = an .
Propozit, ia 5. Dacă a1 , a2 , . . . , an > 0, b > 0, b 6= 1 s, i a1 + a2 + . . . + an = S, atunci
S S S
a1 · b a1 + a2 · b a2 + . . . + an · b an ≥ bn · S (10)

Demonstraţie. Funct, ia f : (0, ∞) → R, f (x) = bx , este funct, ie convexă.


n € Š n S−ak n S
a
n S
S−ak
ak · b bk = 1b · ak · b ak , iar f (n − 1) · S = bn−1 · S,
P P P P
Cum ak f ak
= ak · b ak
=
k=1 k=1 k=1 k=1
cu inegalitatea (C) rezultă inegalitatea din enunt, .
Egalitatea are loc dacă s, i numai dacă a1 = a2 = . . . = an .
Propozit, ia 6. Dacă a1 , a2 , . . . , an > 0 s, i a1 + a2 + . . . + an = S, atunci
 ‹a1  ‹a2  ‹an
S S S
−1 · −1 · ... · −1 ≤ (n − 1)S . (11)
a1 a2 an

Demonstraţie. Funct, ia f : (0, ∞) → R, f (x) = ln x este funct, ie concavă. Aplicând inegalitatea


(C) - pentru funct, ii concave, avem succesiv:
n  ‹ n
X S − ak X S − ak
ak f ≤ f (n − 1) · S ⇔ ak ln ≤ S · ln(n − 1)
k=1
ak k=1
ak
n  ‹ak n  ‹ak
X S S
Y S
⇔ ln −1 ≤ ln(n − 1) ⇔ ln −1 ≤ ln(n − 1)S
k=1
a k
k=1
ak
n  ‹ak
Y S
⇔ −1 ≤ (n − 1)S .
k=1
ak
Egalitatea are loc dacă s, i numai dacă a1 = a2 = . . . = an .
O inegalitate pentru funcţii convexe/concave 11

Propozit, ia 7. ([7]) Dacă a1 , a2 , . . . , an > 0 cu notat, iile a1 +a2 +. . .+an = S s, i a1 ·a2 ·. . .·an = P ,
avem:
€ ŠS
aa11 · aa22 · . . . · aann · (S−a1 )S−a1 · (S−a2 )S−a2 · . . . · (S−an )S−an ≥ (n−1)n−1 · P . (12)

Demonstraţie. Considerăm funct, ia f : (0, ∞) → R, f (x) = x ln x, care este funct, ie strict


convexă. Calculăm s, i aici:
n  ‹ n n
X S − ak X S − ak S − ak X S − ak
ak f = ak · ln = (S − ak ) ln
k=1
ak k=1
ak ak k=1
ak
n  ‹S−ak n  ‹
X S − ak Y S − ak S−ak
= ln = ln
k=1
ak k=1
ak
n n n
Y (S − ak )S−ak 1 Y ak S−ak 1 Y ak
= ln = ln Q
n · a k (S − a k ) = ln ak (S − ak )S−ak ,
k=1
aS−a
k
k
aSk k=1
P S
k=1
k=1

respectiv f (n−1) · S= [(n−1) ln (n−1)] · S=ln (n−1)(n−1)S .


Aplicând inegalitatea (C) se obt, ine inegalitatea (12).
Egalitatea are loc dacă s, i numai dacă a1 = a2 = . . . = an .

Bibliografie

[1] J.L.W.V. Jensen, Sur les fonctions convexes et les inégalités entre les valeurs moyennes,
Acta Mathematica, 30 (1), pp. 175–193, 1906.
[2] D. Mărghidanu, Generalizations and refinements for Bergström and Radon’s Inequalities,
Journal of Sciences and Arts, Year 8, No. 1 (8), 2008, on-line, http://www.icstm.ro/DOCS/
josa/josa_2008_1/cuprins.htm
[3] D. Mărghidanu, Generalizări ale inegalităt, ilor lui Young, Hölder, Rogers s, i Minkowski,
Gazeta Matematică, seria A, Anul XXVI (CV), nr. 3 / 2008.
[4] D. Mărghidanu, Proposed problem, Mathematical Inequalities, https://www.facebook.
com/photo.php?fbid=3378302362228848&set=gm.2648165092138202&type=3&theater&
ifg=1
[5] D. Mărghidanu, Proposed problem, Romanian Mathematical Magazine, https:
//www.facebook.com/photo.php?fbid=2585007641829030&set=gm.1838155749642029&
type=3&theater
[6] D. Mărghidanu, Proposed problem, Math Facts, https://www.facebook.com/photo.php?
fbid=3390923744300043&set=gm.739274503502386&type=3&theater&ifg=1
[7] D. Mărghidanu, Proposed problem, Groupe Matheux, https://www.facebook.com/photo.
php?fbid=3400655483326869&set=gm.1593674704140714&type=3&theater&ifg=1
[8] D.S. Mitrinović, J.E. Pecarić, A.M. Fink, Classical and New Inequalities in Analysis, Kluwer
Acad. Press., 1993.
[9] P.M. Vasic, J.E. Pecaric, On the Jensen Inequality, Univ. Beograd. Publ. Electrotehn. Fak.,
Ser. Mat. Fiz., No. 634 – 677, pp. 50-54, 1979.
O metodă pentru a demonstra unele inegalităt, i

Daniel Văcaru 1

În acest articol vom prezenta trei exemple de demonstrat, ie a unor inegalităt, i ı̂n mai multe
variabile folosind următoarea metodă: considerăm o funct, ie ı̂ntr-o variabilă, fixând celelalte
variabile, s, i utilizăm proprietăt, ile acesteia.
Mai ı̂ntâi, vom rezolva o problemă apărută pe site-ul specializat de matematică www.ssmrmh.ro:
Fie x, y, z ∈ [−1,1]. Arătat, i că
xy + yz + zx ≥ −1

Nguyen Viet Hung

Să considerăm f(y,z) : [−1, 1] → R, f(y,z) (x) = (y + z) · x + yz, y, z ∈ [−1, 1].


Să analizăm cazurile posibile:
I)Dacă y + z > 0, atunci f(y,z) este (strict) crescătoare, s, i f(y,z) (x) ≥ f(y,z) (−1) = yz − y − z.
Pentru a arăta că yz − y − z ≥ −1, este suficient să probăm că yz − y − z + 1 ≥ 0 ⇔
(y − 1) · (z − 1) ≥ 0, fapt implicat de inegalităt, ile y, z ≤ 1.
II) Dacă y + z < 0, atunci f(y,z) este descrescătoare s, i f(y,z) (x) ≥ f(y,z) (1) = yz + y + z.
Pentru a arăta că yz +y +z ≥ −1, ajunge să probăm că yz +y +z +1 ≥ 0 ⇔ (y + 1)·(z + 1) ≥ 0,
care urmează din inegalităt, ile y, z ≥ −1.
III) Pentru cazul y + z = 0, avem f(y,z) (x) = yz, (∀) x ∈ [−1, 1]. Dar
y, z ∈ [−1, 1] ⇔ |y| , |z| ≤ 1 ⇒ |yz| ≤ 1 ⇒ yz ≥ −1,

finalizând demonstrat, ia noastră.


Să rezolvăm s, i o parte a unei probleme apărută pe site-ul de pregătire ı̂n matematică sust, inut
de Andrei Eckstein, anume https://pregatirematematicaolimpiadejuniori.wordpress.com/, al
cărei enunt, este:

a b c abc
Arătat, i că, dacă a, b, c ∈ [0, 1] ,atunci + + + ≤ 2.
1 + bc 1 + ca 1 + ab 2
a a
Cum avem 1 ≥ a ⇒ bc ≥ abc ⇒ 1 + bc ≥ 1 + abc ⇒ ≤ . Sumând cele trei
1 + bc 1 + abc
abc a b c abc a + b + c abc
relat, ii obt, inute s, i adunând cu , obt, inem + + + ≤ + .
2 1 + bc 1 + ca 1 + ab 2 1 + abc 2
a + b + c abc
Rămâne să probăm că + ≤ 2.
1 + abc 2
1
Profesor, Colegiul Economic ,,Maria Teiuleanu”, Pites, ti, danvaccag@gmail.com

12
O metodă pentru a demonstra unele inegalităt, i 13

După calcule obis, nuite, obt, inem că


a + b + c abc
+ ≤ 2 ⇔ 2a+2b+2c+abc+a2 b2 c2 ≤ 4+4abc ⇔ a2 b2 c2 −3abc+2a+2b+2c−4 ≤ 0.
1 + abc 2

Considerăm membrul stâng ca o funct, ie de gradul al II-lea ı̂n c, de forma

f(a,b) : [0, 1] → R, f(a,b) (c) = a2 b2 c2 − c · (3ab − 2) + 2a + 2b − 4.

Calculând, obt, inem f(a,b) (0) = 2a + 2b − 4 ≤ 0 s, i f(a,b) (1) = a2 b2 − 3ab + 2a + 2b − 2. Să


considerăm
gb : [0, 1] → R, gb (t) = b2 t2 + (2 − 3b) t + 2b − 2.

Găsim gb (0) = 2b − 2 ≤ 0 s, i gb (1) = b2 − b ≤ 0. Pentru b > 0 deducem că (0, 1) este un


interval pe care funct, ia de gradul al doilea gb păstrează semnul minus, afirmat, ie valabilă s, i
pentru b = 0, as, adar

a2 b2 − 3ab + 2a + 2b − 2 ≤ 0, (∀) a, b ∈ [0, 1] .

Am obt, inut că f(a,b) (1) ≤ 0, deci a2 b2 c2 − 3abc + 2a + 2b + 2c − 4 ≤ 0, (∀) a, b, c ∈ [0, 1],
stabilind că, ı̂ntr-adevăr,
a b c abc
+ + + ≤ 2.
1 + bc 1 + ca 1 + ab 2

Ca o ultimă aplicat, ie a metodei, avem s, i problema următoare.


AMS 11306. Propusă de Alexandru Ros, oiu, Universitatea din Bucures, ti, România.
a+b+c
Fie a, b s, i c lungimile laturilor unui triunghi nedegenerat ABC, p = , r s, i R razele
2
cercurilor ı̂nscris, respectiv circumscris triunghiului. Arătat, i că
c 4r − R È c
· ≤ (p − a) · (p − b) ≤ . (∗)
2 R 2

not not not


Să ı̂ncepem introducând unele notat, ii. Fie p−a = x, p−b = y, p−c = z. Evident, x, y, z > 0.
S
r p 4S 2 4p (p − a) (p − b) (p − c) 4xyz
Cu aceste notat, ii, avem = = = = .
R abc pabc pabc (x + y) (y + z) (z + x)
4S
Inegalitatea (*) este echivalentă cu:
 ‹
x+y xyz √ AM −GM x+y
· 16 · − 1 ≤ xy ≤ .
2 (x + y) (y + z) (z + x) 2
 ‹
x+y xyz √
Vom proba inegalitatea · 16 · − 1 ≤ xy considerând funct, ia
2 (x + y) (y + z) (z + x)
z x+y √
f : R∗+ → R, f (z) = 8xy · − − xy. Avem
(z + x) (z + y) 2
1 · (x + z) · (y + z) − z (2z + x + y)
f 0 (z) = 8xy ·
(x + z)2 · (y + z)2
14 D. Văcaru

xy + z 2 + z (x + y) − 2z 2 − z (x + y)
= 8xy ·
[(x + z) (y + z)]2
−z 2 + xy
= 8xy · , (∀) z > 0, (∀) x, y > 0.
[(x + z) (y + z)]2

Cu tabelul de monotonie, utilizând semnul funct, iei de gradul al doilea, deducem că f are un
√ √
maxim ı̂n xy. Calcularea lui f xy ne conduce către
√ √ √ 2
√ 8xy xy x+y √ 8xy x+ y
f ( xy) = √  √ − − xy = √ √ 2 − .
x + xy y + xy 2 x+ y 2

√ √ 2
8xy x+ y √ √ 4
Observăm, ı̂nsă, echivalent, a √ √ 2 − ≤ 0 ⇔ 16xy ≤ x + y , care
2

x+ y
√ 
este inegalitatea mediilor. Rezultă că f (z) ≤ f xy ≤ 0, ceea ce ı̂ncheie demonstrat, ia.
Dacă triunghiul este echilateral inegalităt, ile devin egalităt, i, iar ı̂n cazul triunghiurilor isoscele
cu a = b doar inegalitatea din dreapta devine egalitate.
ARTICOLE S, I NOTE DE INFORMATICĂ

Complexitatea algoritmului de sortare prin interclasare


(mergesort)

Costel Bălcău 1

Algoritmul de sortare prin interclasare (mergesort), inventat de către faimosul savant John
von Neumann ı̂n anul 1945, este unul dintre cei mai performant, i algoritmi de sortare.
În articolul de fat, ă vom analiza riguros complexitatea acestui algoritm, utilizând rezultate
de bază din teoria algoritmilor privind ordinele de complexitate s, i arborii de sortare prezentate
ı̂n [2], [3], precum s, i tehnici matematice de rezolvare a relat, iilor de recurent, ă.
Astfel vom putea studia eficient, a s, i optimalitatea acestui algoritm ı̂n cadrul clasei algoritmilor
de sortare bazat, i pe comparat, ii de chei.

Descrierea algoritmului

Pentru a sorta crescător un vector dat A = (a1 , a2 , . . . , an ), n ≥ 1, algoritmul de sortare


prin interclasare (mergesort) utilizează următorul procedeu specific metodei de programare
Divide et Impera:

• Dacă n = 1, atunci vectorul este deja sortat (având un singur element);


• Dacă n ≥ 2, atunci:
– Se ı̂mparte vectorul A ı̂n doi subvectori
› ž l m
n+1 n
(a1 , . . . , am ) s, i (am+1 , . . . , an ) , unde m = =
2 2
(deci am este elementul median s, i cei doi subvectori au dimensiuni cât mai apropiate
posibil);
– Se sortează crescător cei doi subvectori, prin acelas, i procedeu;
– Se interclasează cei doi subvectori sortat, i, adică se reconstruies, te vectorul A astfel
ı̂ncât să cont, ină toate componentele celor doi subvectori sortat, i (deci toate cele n
componente ale vectorului init, ial) s, i care să fie, de asemenea, sortat crescător.

Reamintim că bxc reprezintă partea ı̂ntreagă (inferioară) a numărului real x (adică aproximarea
ı̂ntreagă prin lipsă a lui x, notată de obicei cu [x]), iar dxe reprezintă partea ı̂ntreagă superioară
a numărului real x (adică aproximarea ı̂ntreagă prin adaos a lui x).
1
Conf. univ. dr., Universitatea din Pites, ti, cbalcau@yahoo.com

15
16 C. Bălcău

Descrierea ı̂n pseudocod a algoritmului are următoarea formă.


SORTINT(A, p, u) : // sortarea secvent, ei (ap , . . . , au )
if p < u then
jp + uk
m← ;
2
SORTINT(A, p, m); // sortarea subsecvent, ei (ap , . . . , am )
SORTINT(A, m + 1, u); // sortarea subsecvent, ei (am+1 , . . . , au )
INTERCLAS(A, p, u, m); // interclasarea celor două subsecvent, e sortate

unde procedura de interclasare este:


INTERCLAS(A, p, u, m) ; // interclasarea subsecvent, elor
// (ap , . . . , am ) s, i (am+1 , . . . , au ), sortate crescător
i ← p; // pozit, ia curentă din subsecvent, a (ap , . . . , am )
j ← m + 1; // pozit, ia curentă din subsecvent, a (am+1 , . . . , au )
k ← 0; // dimensiunea curentă a unui vector B, sortat crescător,
// ce va fi construit prin interclasarea celor două subsecvent, e,
// comparând componentele curente ai s, i aj din cele două subsecvent, e
// s, i copiind-o pe cea mai mică ı̂n B
while i ≤ m and j ≤ u do
// mai există componente necopiate ı̂n ambele subsecvent, e
if A[i] ≤ A[j] then
// mai mică este componenta curentă din prima subsecvent, ă,
k ← k + 1; B[k] ← A[i]; // deci o copiem ı̂n B
i ← i + 1; // s, i avansăm ı̂n prima subsecvent, ă
else
// mai mică este componenta curentă din a doua subsecvent, ă,
k ← k + 1; B[k] ← A[j]; // deci o copiem ı̂n B
j ← j + 1; // s, i avansăm ı̂n a doua subsecvent, ă
// una dintre subsecvent, e nu mai are elemente necopiate;
// copiem ı̂n B s, i componentele rămase necopiate
// din cealaltă subsecvent, ă
for q = i, m do // componente rămase necopiate ı̂n prima subsecvent, ă
k ← k + 1; B[k] ← A[q];
for q = j, u do // componente rămase necopiate ı̂n a doua subsecvent, ă
k ← k + 1; B[k] ← A[q];
// La final aducem componentele (sortate crescător) din B = (b1 , . . . , bk )
// ı̂napoi ı̂n subsecvent, a (ap , . . . , au ):
for i = 1, k do
A[p + i − 1] ← B[i];

Apel:
SORTINT(A, 1, n) // pentru sortarea ı̂ntregului vector A = (a1 , . . . , an ).

Exemplul 1. În figura următoare este reprezentat arborele extins de sortare asociat algoritmului
de sortare prin interclasare pentru n = 3. Observăm că algoritmul nu cont, ine comparat, ii de
Complexitatea algoritmului de sortare prin interclasare (mergesort) 17

chei redundante.

1?2
≤ >
1?3 2?3
≤ > ≤ >
2?3 (3,1,2) 1?3 (3,2,1)
≤ > ≤ >
(1,2,3) (1,3,2) (2,1,3) (2,3,1)

Observat, ia 1. Procedura de interclasare poate fi us, or ı̂mbunătăt, ită, eliminând copierile inutile
ı̂n vectorul intermediar B s, i ı̂napoi ı̂n vectorul A ale componentelor rămase necopiate (la ies, irea
din ciclul while) ı̂n una dintre cele două subsecvent, e interclasate. În acest sens, putem renunt, a
la ciclul for q = j, u, deoarece elementele rămase necopiate ı̂n a doua subsecvent, ă sunt deja
pe pozit, iile bune, deci nu are rost să le mai copiem ı̂n B. De asemenea, elementele rămase
necopiate ı̂n prima subsecvent, ă pot fi copiate direct pe ultimele pozit, ii ale secvent, ei (ap , . . . , au )
(ı̂n ordine inversă, pentru a evita pierderea valorilor unor componente prin suprapunere), fără a
le mai copia ı̂n B; pentru aceasta, putem ı̂nlocui ciclul for q = i, m cu:
for q = m, i, −1 do
A[u + q − m] ← A[q];

Observat, ia 2. Implementarea aleasă pentru algoritmul de sortare prin interclasare este una
recursivă, ı̂n care solut, ia problemei apelează solut, iile subproblemelor de dimensiuni mai mici,
adică foloses, te strategia Divide et Impera ı̂n varianta top-down. În cazul acestei vari-
ante este important că subproblemele sunt disjuncte s, i astfel calculul necesar rezolvării unei
subprobleme nu este reluat prin apelarea repetată a acesteia (ceea ce ar conduce la un timp
de calcul exponent, ial, as, a cum se ı̂ntâmplă la utilizarea acestei variante ı̂n rezolvarea unor
probleme prin metoda programării dinamice). Ment, ionăm că algoritmul poate fi descris s, i
implementat s, i iterativ, utilizând varianta bottom-up a strategiei Divide et Impera, ı̂n
care subproblemele se rezolvă ı̂n ordinea crescătoare a dimensiunii lor. Această variantă este
mult mai eficientă ı̂n cazul programării dinamice, când subproblemele se suprapun, dar ı̂n cazul
sortării prin interclasare are aceeas, i complexitate ca s, i varianta noastră, de tip top-down. Mai
multe detalii privind implementarea s, i complexitatea acestor două forme, precum s, i a altor
variante ale algoritmului de sortare prin interclasare, pot fi găsite ı̂n [9] s, i [8].

Complexitatea algoritmului

Teorema 1 (complexitatea algoritmului de sortare prin interclasare). Algoritmul de


sortare prin interclasare a unui vector cu n componente are complexitatea Θ(n log2 n).

Demonstraţie. Fie T (n) timpul de execut, ie al algoritmului SORTINT(A, 1, n). Vom demonstra
că T (n) = Θ(n log2 n) prin două metode.
18 C. Bălcău

Metoda 1) Vom estima efectiv timpul de execut, ie T (n). Vom număra numai operat, iile de
comparat, ie de chei (comparat, ii ı̂n care intervin elemente ale vectorului) s, i de deplasări
(atribuiri ı̂n care intervin elemente ale vectorului). Celelalte operat, ii care se efectuează au acelas, i
ordin de cres, tere cu cele pe care le analizăm. Avem

T (n) = c1 · Nc (n) + c2 · Nd (n), (1)

unde Nc (n) s, i Nd (n) reprezintă numărul de comparat, ii de chei, respectiv numărul de deplasări
efectuate de algoritm, iar c1 , c2 > 0 sunt constante ce reprezintă costurile ı̂n timp ale executării
unei comparat, ii de chei s, i, respectiv, ale unei deplasări.
Conform descrierii recursive a algoritmului, rezultă că numerele Nc (n) s, i Nd (n) verifică,
respectiv, relat, iile de recurent, ă
(
l n m 0,
j n k l n m j n k dacă n = 1,
Nc (n) = (2)
Nc + Nc + Nc , , dacă n > 1,
2 2 2 2
(
l n m 0,
j n k l n m j n k dacă n = 1,
Nd (n) = (3)
Nd + Nd + Nd , , dacă n > 1,
2 2 2 2
l n m j n k l n m j n k
unde N c , s, i N d , reprezintă numărul de comparat, ii de chei, respectiv
2 2 2 2
numărul de deplasări efectuate
› ž lalinterclasarea celor doi subvectori sortat, i, (a1 , . . . , am ) s, i
n+1 nm lnm lnm
(am+1 , . . . , an ), cu m = = , de dimensiuni m = , respectiv n − m = n − =
jnk 2 2 2 2
.
2
Conform descrierii procedurii de interclasare, la interclasarea celor doi subvectorilor avem:
l n m j n k
• Numărul de deplasări N d , este egal cu dublul numărul de componente ale
2 2
lui A (fiecare componentă - fie că se află ı̂n primul subvector, fie că se află ı̂n al doilea
subvector - este mutată o dată ı̂n vectorul intermediar B s, i o dată adusă ı̂napoi ı̂n A),
deci l n m j n k l n m j n k
Nd , =2 + = 2n. (4)
2 2 2 2
l n m j n k
• Numărul de comparat, ii de chei N c , este egal cu numărul de execut, ii ale
2 2
ciclului while. Cu fiecare astfel de comparat, ie are loc s, i o copiere ı̂n vectorul B a unei
componente din A, adică o deplasare. La ies, irea din ciclul while, componentele unuia
dintre cei doi subvectori ai lui A au fost toate copiate ı̂n B, iar ı̂n celălalt subvector au
mai rămas r componente necopiate ı̂n B, care vor fi s, i ele copiate ı̂n B, dar după ies, irea
din acest ciclu. Rezultă că l n m j n k
Nc , = n − r.
2 2
Evident, nl n m j n ko l n m
1 ≤ r ≤ max , = ,
2 2 2
deci l n m j n k nj n k j n k o
Nc , ∈ , + 1, . . . , n − 1 . (5)
2 2 2 2
Complexitatea algoritmului de sortare prin interclasare (mergesort) 19

Notând cu Nc+ (n) numărul de comparat, ii de chei ı̂n cazul cel mai defavorabil, din (2) s, i (5)
rezultă relat, ia de recurent, ă
(
0,j k dacă n = 1,
Nc+ (n) =
l n m n (6)
Nc+ + Nc+ + n − 1, dacă n > 1.
2 2
Rezolvăm această relat, ie de recurent, ă.
Pentru n = număr par, adică n = 2m, m ≥ 1, avem
Nc+ (n + 1) − Nc+ (n) = Nc+ (2m + 1) − Nc+ (2m)
= Nc+ (m + 1) + Nc+ (m) + 2m − Nc+ (m) + Nc+ (m) + 2m − 1
 

= Nc+ (m + 1) − Nc+ (m) + 1,


iar pentru n = număr impar, adică n = 2m − 1, m ≥ 2, avem
Nc+ (n + 1) − Nc+ (n) = Nc+ (2m) − Nc+ (2m − 1)
= Nc+ (m) + Nc+ (m) + 2m − 1 − Nc+ (m) + Nc+ (m − 1) + 2m − 2
 

= Nc+ (m) − Nc+ (m − 1) + 1,


deci pentru orice n > 1 avem
j n k  j n k
Nc+ (n + 1) − Nc+ (n) = Nc+ +1 − Nc+ + 1.
2 2
Notând
f (n) = Nc+ (n + 1) − Nc+ (n), ∀n ∈ N∗ , (7)
obt, inem că (
j n1, dacă n = 1,
f (n) = (8)
k
f + 1, dacă n > 1.
2
Demonstrăm că
f (n) = dlog2 (n + 1)e, ∀n ∈ N∗ , (9)
prin induct, ie.
Pentru n = 1 avem f (1) = 1 = dlog2 2e.
Presupunem egalitatea adevărată pentru orice număr din mult, imea {1, . . . , n − 1} s, i o
demonstrăm pentruj n k n (n ≥ 2). Într-adevăr, folosind relat, ia de recurent, ă (8) s, i ipoteza de
induct, ie pentru avem
2
j n k l j n k m
f (n) = f + 1 = log2 + 1 + 1.
2 2
Dar, notând dlog2 (n + 1)e = k, avem k ∈ N∗ , k ≥ 2 s, i, succesiv:
k − 1 < log2 (n + 1) ≤ k; 2k−1 < n + 1 ≤ 2k ;
n
2k−1 ≤ n < 2k ; 2k−2 ≤ < 2k−1 ;
jnk j2n k
k−2 k−1 k−2
2 ≤ <2 ; 2 < + 1 ≤ 2k−1 ;
2 j n k  2
k − 2 < log2 + 1 ≤ k − 1;
l j n k 2 m
log2 + 1 = k − 1,
2
20 C. Bălcău

deci
f (n) = k − 1 + 1 = k = dlog2 (n + 1)e,
ceea ce ı̂ncheie demonstrat, ia prin induct, ie a relat, iei (9).
Din (7) s, i (9) rezultă că

Nc+ (n + 1) − Nc+ (n) = dlog2 (n + 1)e, ∀n ∈ N∗ .

Aplicând succesiv această relat, ie avem

Nc+ (n) = Nc+ (n − 1) + dlog2 ne


= Nc+ (n − 2) + dlog2 (n − 1)e + dlog2 ne
...
= Nc+ (1) + dlog2 2e + dlog2 3e + . . . + dlog2 ne.

Cum Nc+ (1) = 0, obt, inem că

Nc+ (n) = dlog2 1e + dlog2 2e + . . . + dlog2 ne, ∀n ∈ N∗ . (10)

Dar n
X
dlog2 ie = ndlog2 ne − 2dlog2 ne + 1, ∀n ∈ N∗ .
i=1

Într-adevăr, egalitatea este evident valabilă pentru n = 1, iar pentru n ≥ 2 notând s = dlog2 ne
avem s ∈ N∗ s, i s − 1 < log2 n ≤ s, deci 2s−1 < n ≤ 2s , iar pentru orice i ∈ {2, 3, . . . , n} avem
echivalent, ele

dlog2 ie = j ⇔ j − 1 < log2 i ≤ j ⇔ 2j−1 < i ≤ 2j ⇔ 2j−1 + 1 ≤ i ≤ 2j ;

astfel avem
n n s−1 2 j n
X X X X X
dlog2 ie = dlog2 ie = dlog2 ie + dlog2 ie
i=1 i=2 j=1 i=2j−1 +1 i=2s−1 +1
s−1 2 j n s−1
X X X X
= j+ s= j(2j − 2j−1 ) + s(n − 2s−1 )
j=1 i=2j−1 +1 i=2s−1 +1 j=1
s−1
X
= j · 2j−1 + s(n − 2s−1 ) = s · 2s−1 − 2s + 1 + s(n − 2s−1 )
j=1

= ns − 2s + 1 = ndlog2 ne − 2dlog2 ne + 1.

Rezultă că n
X
Nc+ (n) = dlog2 ie = ndlog2 ne − 2dlog2 ne + 1, ∀n ∈ N∗ .
i=1

Dar
log2 n ≤ dlog2 ne < log2 n + 1,
deci
n ≤ 2dlog2 ne < 2n.
Complexitatea algoritmului de sortare prin interclasare (mergesort) 21

Astfel rezultă că ı̂n cazul cel mai defavorabil numărul de comparat, ii de chei verifică inegalităt, ile

n log2 n − 2n + 1 < Nc+ (n) < n log2 n + 1, ∀n ∈ N∗ . (11)

Notând cu Nc− (n) numărul de comparat, ii de chei ı̂n cazul cel mai favorabil, din (2) s, i (5)
rezultă relat, ia de recurent, ă
(
0, j k j k dacă n = 1,
Nc− (n) =
l n m n n
− (12)
Nc + Nc− + , dacă n > 1.
2 2 2
Demonstrăm că s, irul (Nc− (n))n≥1 este crescător, adică

Nc− (n + 1) ≥ Nc− (n), ∀ n ∈ N∗ ,

prin induct, ie.


Pentru n = 1 avem Nc− (2) = 1 > 0 = Nc− (1).
Presupunem inegalitatea adevărată pentru orice număr din mult, imea {1, . . . , n − 1} s, i o
l n m pentru n (n ≥ 2). Într-adevăr, folosind
demonstrăm j n krelat, ia de recurent, ă (12), ipoteza de induct, ie
pentru când n este par, respectiv pentru când n este impar, precum s, i inegalitatea
2› ž j k 2
n+1 n
evidentă ≥ , avem
2 2
¡ ¤‹ › ž‹ › ž
n+1 n+1 n+1
Nc− (n + 1) =Nc− + Nc− +
2 2 2
l n m j n k j n k
− −
≥ Nc + Nc +
2 2 2
= Nc− (n).

Demonstrat, ia prin induct, ie este ı̂ncheiată.


Fie n ∈ N∗ s, i
t = blog2 nc, (13)
deci t ∈ N s, i t ≤ log2 n < t + 1, deci 2t ≤ n < 2t+1 . Deoarece s, irul (Nc− (n))n≥1 este crescător,
rezultă că
Nc− (n) ≥ Nc− (2t ). (14)
Dar, conform relat, iei de recurent, ă (12) avem

Nc− (2t ) = 2Nc− (2t−1 ) + 2t−1 , ∀ t ∈ N∗ .

Rescriem această egalitate sub forma

Nc− (2t ) − t · 2t−1 = 2 Nc− (2t−1 ) − (t − 1)2t−2 , ∀ t ∈ N∗ ,




Aplicând succesiv această relat, ie avem

Nc− (2t ) − t · 2t−1 = 2 Nc− (2t−1 ) − (t − 1)2t−2




= 22 Nc− (2t−2 ) − (t − 2)2t−3




...
= 2t Nc− (2t−t ) − (t − t)2t−t−1

22 C. Bălcău

Cum Nc− (1) = 0 obt, inem că Nc− (2t ) − t · 2t−1 = 0, deci

Nc− (2t ) = t · 2t−1 , ∀t ∈ N. (15)

Din (14), (15) s, i (13) rezultă că ı̂n cazul cel mai favorabil numărul de comparat, ii de chei verifică
inegalităt, ile
1 1
Nc− (n) ≥ blog2 nc · 2blog2 nc−1 > · n log2 n − · n, ∀n ∈ N∗ . (16)
4 4
Din (11) s, i (16) rezultă că numărul de comparat, ii de chei verifică inegalităt, ile
1 1
· n log2 n − · n < Nc (n) < n log2 n + 1, ∀n ∈ N∗ . (17)
4 4

Din relat, iile (3) s, i (4) rezultă că numărul de deplasări efectuate de algoritm verifică relat, ia
de recurent, ă (
l n m 0, j n k dacă n = 1,
Nd (n) =
Nd + Nd + 2n, dacă n > 1.
2 2
Această relat, ie se rezolvă analog relat, iei (6), obt, inându-se
€ Š
Nd (n) = 2 ndlog2 ne − 2dlog2 ne + n , ∀n ∈ N∗ ,

deci
2n log2 n − 2n < Nd (n) < 2n log2 n + 2n, ∀n ∈ N∗ . (18)

Din relat, iile (1), (17) s, i (18) rezultă că T (n) = Θ(n log2 n).
Metoda 2) Vom aplica Teorema Master. Conform descrierii recursive l a algoritmului
n m j n k
s, i utilizând complexitatea procedurii de interclasare (formulele pentru N c , s, i
l n m j n k 2 2
Nd , ), timpul de execut, ie T (n) verifică relat, ia de recurent, ă
2 2
T (n) = 2T (n/2) + Θ(n), ∀ n > 1,

cu convent, iile de notat, ie din Teorema Master (Teorema 1 din [2]). Conform Cazului 2 al acelei
teoreme (pentru a = 2, b = 2, c = 1 = logb a s, i k = 0) rezultă că T (n) = Θ(n log2 n).

Următorul rezultat este o consecint, ă directă a teoremei anterioare s, i a Teoremei 1 din [3]
(care afirmă că orice algoritm optim de sortare, bazat pe comparat, ii de chei, are complexitatea
Θ(n log2 n)).

Corolarul 4 (optimalitatea algoritmului de sortare prin interclasare). Algoritmul de


sortare prin interclasare este asimptotic-optim (ı̂n clasa algoritmilor bazat, i pe comparat, ii de
chei), atât ı̂n raport cu timpul de execut, ie ı̂n cazul cel mai defavorabil, cât s, i ı̂n raport cu timpul
mediu de execut, ie.

Observat, ia 3. Mai mult, conform relat, iilor (11) s, i (17) s, i Teoremei 1 din [3] rezultă că, exact
ca la algoritmii optimi de sortare bazat, i pe comparat, ii de chei, numărul de comparat, ii de chei
efectuate de algoritmul de sortare prin interclasare a unui vector cu n componente este chiar
asimptotic echivalent cu n log2 n, atât ı̂n cazul cel mai defavorabil, cât s, i ı̂n cazul mediu (cazul
timpului mediu de execut, ie).
Complexitatea algoritmului de sortare prin interclasare (mergesort) 23

Totus, i, conform Corolarului 1 din [3], algoritmul de sortare prin interclasare nu este
optim, nici ı̂n raport cu timpul mediu de execut, ie, deoarece arborele extins de sortare
asociat nu are ı̂ntotdeauna toate nodurile externe situate doar pe ultimul s, i, eventual, pe
penultimul nivel, nici ı̂n raport cu timpul de execut, ie ı̂n cazul cel mai defavorabil,
deoarece ı̂nălt, imea acestui arbore nu este ı̂ntotdeauna egală cu dlog2 n!e. De exemplu, pentru
n = 5 există noduri externe nesituate pe acelas, i nivel sau pe două nivele consecutive, iar
ı̂nălt, imea arborele extins de sortare asociat este mai mare decât dlog2 5!e = dlog2 120e = 7, as, a
cum reiese din figura următoare, ı̂n care este reprezentată doar o parte a arborelui extins de
sortare.

1?2

1?3

2?3

4?5

1?4
≤ >
2?4 1?5
≤ ≤ >
3?4 2?5 (4,5,1,2,3)
≤ ≤
(1,2,3,4,5) 3?5
≤ >
(4,1,2,3,5) (4,1,2,5,3)

Observat, ia 4. O altă justificare a neoptimalităt, ii (pure a) algoritmului de sortare prin interclasare


este următoarea: ı̂n cazul cel mai defavorabil la sortarea unui vector cu n componente, numărul
de comparat, ii de chei efectuate de algoritmul mergesort este, conform (10),

Nc+ (n) = dlog2 1e + dlog2 2e + . . . + dlog2 ne,

iar numărul de comparat, ii de chei efectuate de orice algoritm optim este, conform Corolarului 1
din [3],
Ndef (n) = dlog2 n!e = dlog2 1 + log2 2 + . . . + log2 ne.
Evident, pentru orice numere reale x s, i y avem

dxe + dye ≥ dx + ye (19)


24 C. Bălcău

(deoarece dxe ≥ x, dye ≥ y, dxe + dye ≥ x + y s, i dxe + dye ∈ Z, deci dxe + dye = ddxe + dyee ≥
dx + ye ). Folosind (19) rezultă că

Nc+ (n) = dlog2 1e + dlog2 2e + . . . + dlog2 ne ≥ dlog2 1 + log2 2 + . . . + log2 ne = Ndef (n),

ceea ce era de as, teptat, conform optimalităt, ii invocate! Mai mult, avem Nc+ (5) = dlog2 1e +
dlog2 2e + dlog2 3e + dlog2 4e + dlog2 5e = 0 + 1 + 2 + 2 + 3 = 8, iar Ndef (5) = dlog2 5!e =
dlog2 120e = 7, deci
Nc+ (5) > Ndef (5).
Folosind această inegalitate strictă s, i inegalitatea (19), rezultă că pentru n ≥ 5 avem

Nc+ (n) = (dlog2 1e + . . . + dlog2 5e) + (dlog2 6e + . . . + dlog2 ne)


> dlog2 1 + . . . + log2 5e + dlog2 6 + . . . + log2 ne
≥ dlog2 1 + . . . + log2 5 + log2 6 + . . . + log2 ne = Ndef (n),

deci
Nc+ (n) > Ndef (n), ∀ n ≥ 5.
Prin urmare algoritmul mergesort nu este optim ı̂n raport cu timpul de execut, ie ı̂n cazul cel
mai defavorabil. Cum, conform Corolarului 1 din [3], optimalitatea ı̂n cazul timpului mediu de
execut, ie implică optimalitatea ı̂n cazul cel mai defavorabil, rezultă că algoritmul mergesort nu
este optim nici ı̂n raport cu timpul mediu de execut, ie.
Observat, ia 5. Complexitatea algoritmului mergesort este una dintre cele mai bune din punct de
vedere al numărului de comparat, ii de chei, ı̂n clasa algoritmilor de sortare bazat, i pe astfel de
comparat, ii. Un dezavantaj este ı̂nsă faptul că se utilizează un vector suplimentar de lucru, care
poate avea dimensiunea foarte mare (fiind egală cu cea a vectorului ce trebuie sortat).

Bibliografie

[1] Gh. Barbu, I. Văduva, M. Bolos, teanu, Bazele informaticii, Editura Tehnică, Bucures, ti, 1997.

[2] C. Bălcău, Optimalitatea algoritmului de căutare binară, MATINF I (2018), nr. 1, 39–51.

[3] C. Bălcău, Complexitatea algoritmilor de sortare, MATINF II (2019), nr. 3, 41–48.

[4] A. Carabineanu, Structuri de date, http://ebooks.unibuc.ro/informatica/carabineanu/CARA_


STR.pdf.

[5] T.H. Cormen, C.E. Leiserson, R.L. Rivest, C. Stein, Introduction to Algorithms, MIT Press,
Cambridge, 2009.

[6] H. Georgescu, Tehnici de programare, Editura Universităt, ii din Bucures, ti, Bucures, ti, 2005.

[7] D.E. Knuth, The Art Of Computer Programming. Vol. 3: Sorting and Searching, Addison-
Wesley, Reading, MA, 1998.

[8] R. Sedgewick, P. Flajolet, An Introduction to the Analysis of Algorithms, Addison-Wesley,


New Jersey, 2013.

[9] R. Sedgewick, K. Wayne, Algorithms, Addison-Wesley, Massachusetts, 2011.


Protocoale

Marin Cristea 1

În ret, ele există două scheme de adresare: una foloses, te adresa MAC, specifică nivelului
legături de date, iar cealaltă adresa logică identificată ı̂n cadrul nivelului Ret, ea din modelul
OSI. Un exemplu de adresă logică este adresa IP. Router-ul este un dispozitiv fizic care comută
pachetele ı̂ntre ret, ele ı̂n funct, ie de adresa nivelului 3. Este capabil să ia decizii optime ı̂n alegerea
rutei optime către destinat, ie. Comutarea pachetelor este realizată s, i de către o punte sau un
switch ı̂nsă ı̂n funct, ie de adresa fizică care este implementată ı̂n hardware s, i nu permite crearea
de ierarhii. Adresa IP este implementată ı̂n software s, i se poate diviza pe mai multe clase ı̂n
funct, ie de diferite criterii s, i poate fi foarte us, or schimbată. Este frecventă ı̂mpărt, irea stat, iilor
de către administrator ı̂n funct, ie de clasa de adresare, locat, ie geografică, departament, etc.
Switch-ul (puntea) se foloses, te pentru a conecta segmente de ret, ea; router-ul este utilizat optim
ı̂n conectarea ret, elelor care se pot afla la mari distant, e geografice. Acest aspect a fost punctul
de plecare ı̂n crearea Internetului.
Conexiunea dintre o ret, ea s, i router poartă numele de interfat, ă sau port. În rutarea pe bază
de adrese IP, fiecare interfat, ă trebuie să aibă o adresă de ret, ea/subret, ea unică.
Observat, ie. Oricare două ret, ele conectate la Internet, indiferent de locat, ia lor geografică,
trebuie să aibă adrese IP diferite pentru ca procesul de rutare să poată avea loc.
Metode de asignare a adreselor IP
Există două posibilităt, i de a asigna o adresă IP unei stat, ii: statică s, i dinamică. Indiferent
de metoda aleasă, două interfet, e nu pot avea aceeas, i adresă.
Asignarea statică implică deplasarea la fiecare stat, ie s, i atribuirea unei adrese IP manual.
Această abordare implică ment, inerea unor tabele foarte exacte cu fiecare stat, ie s, i adresa pe care
o are, ı̂n caz contrar putând apare dubluri de adrese. Sisteme de operare cum ar fi Windows
95/98/NT fac anumite verificări ı̂nainte de a init, ializa protocolul TCP/IP. Dacă se descoperă
dubluri de adrese, protocolul nu se ı̂ncarcă s, i se trimite un mesaj de eroare. Cum nu toate
sistemele de operare identifică această situat, ie, ment, inerea tabelelor este obligatorie. Asignarea
dinamică se poate realiza cu una din următoarele metode:
Protocolul de rezolut, ie inversă a
adresei [1] (Reverse Address Resolution Pro-
tocol - RARP) leagă o adresă MAC de o
adresă IP. Această legare permite anumitor
dispozitive să ı̂ncapsuleze datele ı̂nainte de a
le trimite ı̂n ret, ea. Metoda necesită prezent, a
unui RARP server ı̂n ret, ea care să răspundă
cererilor.

1
Profesor, Colegiul Nat, ional ,,Ion C. Brătianu”, Pites, ti, marin.cristea@gmail.com

25
26 M. Cristea

Să considerăm un exemplu ı̂n care o stat, ie dores, te să transmită un pachet, ı̂s, i cunoas, te
adresa MAC dar nu cunoas, te adresa IP. Pentru ca destinatarul să poată ret, ine datele ı̂n vederea
transmiterii către nivelurile superioare din modelul OSI, sursa trebuie să includă atât adresa
fizică cât s, i adresa logică (IP). În acest scop se init, iază un proces numit ,,cerere RARP” (RARP
request) ı̂n urma căruia va cunoas, te adresa IP. Sursa va construi un pachet de cerere RARP
care se va trimite către toate stat, iile din ret, ea folosind o adresă cu destinat, ie multiplă. În acest
pachet câmpul cu adresa IP a sursei este goală. Cererea este primită s, i de serverul RARP care
va răspunde stat, iei cu adresa solicitată.
Protocolul BOOTstarp
Acest protocol este folosit la pornirea unui dispozitiv pentru obt, inerea adresei IP. Un mesaj
UDP (User Datagram Protocol) este ı̂ncapsulat ı̂ntr-o datagramă IP s, i trimis ı̂n ret, ea sub
formă de mesaj cu destinat, ie multiplă. Serverul BOOTP primes, te cererea s, i răspunde ı̂n acelas, i
mod printr-un mesaj broadcast care include adresa IP. Analog RARP, protocolul BOOTstrap
lucrează ı̂ntr-un mediu client-server s, i necesită schimbul unui singur pachet. Deosebirea este că
primul furnizează numai adresa IP, pe când acesta mai transmite adresa router-ului (default
gateway), adresa serverului s, i ı̂ncă un câmp cu informat, ii opt, ionale. Nu poate ı̂nsă să furnizeze
adrese IP dinamice; serverul trebuie să cunoască un set de adrese din care va transmite atunci
când primes, te cerere.
Protocolul de configurare dinamică a gazdei (Dynamic Host Configuration Pro-
tocol - DHCP)
DHCP [2] este un succesor al protocolului BOOTstarp. În plus, DHCP permite unei gazde
să obt, ină o adresă IP repede s, i dinamic. Serverul DHCP trebuie să cunoască doar intervalul de
adrese pe care are dreptul să le folosească. Datele de configurare se transmit printr-un singur
mesaj, iar serverul poate să transmită s, i alte date auxiliare.
Atunci când o stat, ie DHCP este pornită,
se intră automat ı̂n starea de init, ializare,
ce constă ı̂n transmiterea unui mesaj numit
DHCP Discover, care este un pachet UDP.
Următoarea etapă este cea ı̂n care primes, te
din partea serverului un răspuns numit DHCP
Offer care cont, ine adresa cerută.Stat, ia va ne-
gocia cu serverul intervalul de timp pe care
ı̂l are la dispozit, ie ı̂n utilizarea adresei, fără
a o schimba, prin trimiterea unui alt mesaj
numit DHCP Request. Serverul confirmă ce-
rerea stat, iei printr-un răspuns numit DHCP
Ack (acknowledge). Abia după primirea aces-
tui ultim pachet stat, ia poate să folosească
adresa.

Cerere ARP
Comunicarea ı̂ntre două stat, ii este posibilă atunci când se cunosc ambele adrese MAC s, i IP.
Dacă se cunoas, te adresa IP a destinatarului, atunci trebuie să se determine s, i adresa fizică.
Modelul TCP/IP cont, ine un protocol numit ARP (Address Resolution Protocol) care poate
să determine automat adresa fizică dacă se cunoas, te adresa IP.
Protocoale 27

Pentru a fi ı̂n orice moment capabile să co-


munice ı̂ntre ele, stat, iile din acelas, i segment
păstrează ı̂n memoria RAM tabele ı̂n care
se află cele două tipuri de adrese necesare:
MAC s, i IP. Acestea poartă numele de tabele
ARP s, i practic leagă o adresă fizică de una
logică. Ele se ment, in automat pentru fiecare
gazdă din ret, ea, des, i este posibil ı̂n cazuri ex-
treme să se configureze manual. Atunci când
stat, ia cunoas, te adresa IP a destinatarului,
consultă tabelele ARP, extrage adresa fizică
s, i o inserează ı̂n pachet.

Dacă ı̂n tabela ARP nu se găses, te adresa fizică a unei stat, ii al cărei IP este cunoscut, atunci
se ı̂ncepe un proces numit ,,cerere ARP” (ARP request). Sursa va construi un pachet ARP pe
care ı̂l va trimite la toate stat, iile din ret, ea. Pentru a se asigura că toate stat, iile vor verifica
pachetul, se utilizează o adresă cu destinat, ie multiplă (broadcast).
Se va completa adresa IP cunoscută a destinatarului iar la adresa MAC se va scrie numai
cifra F astfel: FF-FF-FF-FF-FF-FF. Fiecare stat, ie va examina pachetul; unde va coincide adresa
IP, ı̂nseamnă că trebuie să se răspundă printr-un pachet care să cont, ină s, i adresa MAC. Acest
răspuns poartă numele de ARP reply. Stat, ia care a init, iat cererea va primi pachetul s, i ı̂s, i va
actualiza tabela ARP, moment ı̂n care poate să demareze schimbul de date cu destinatarul.
Un alt protocol inclus in TCP/IP este ICMP (Internet Control Message Protocol) care se
foloses, te de către o stat, ie pentru a raporta o problemă expeditorului unui mesaj. De exemplu
dacă un router primes, te un pachet pe care nu poate să-l transmită mai departe, atunci va apela
la protocolul ICMP pentru a-l anunt, a pe expeditor. De asemenea se poate folosi pentru a
determina dacă se poate ajunge la un anumit destinatar ı̂nainte de a trimite pachetul de date.
Operat, ia se numes, te ,,ping” s, i se bazează pe un ecou la cerere (echo request/echo reply).
Port implicit (default gateway)
Comunicarea ı̂ntre două gazde aflate ı̂n ret, ele diferite este posibilă numai dacă acestea au
primit o adresă drept port implicit (default gateway). Acesta trebuie să fie adresa IP a interfet, ei
router-ului care este conectat la segmentul gazdei respective iar gazda s, i interfat, a vor avea
adrese care apart, in segmentului comun.
Dacă nu este definit un port implicit, comunicat, ia este posibilă numai ı̂ntre gazdele care
apart, in aceluias, i segment. O stat, ie cu intent, ii de comunicare s, i care posedă IP-ul destinatarului
va căuta ı̂n tabela ARP proprie adresa MAC. Dacă nu o găses, te trebuie să cunoască un port
implicit pentru a trimite pachetele. Dacă portul nu este definit, datele nu se pot livra.
Comunicarea ı̂ntre subret, ele
O problemă importantă ı̂n ret, ele este comunicarea ı̂ntre gazde aflate ı̂n segmente diferite.
Sunt două etape ale acestei probleme. Prima ar fi obt, inerea adresei MAC a destinatarului iar a
doua transferul pachetului de la un segment la altul pentru a ajunge ı̂n final la destinat, ie.
28 M. Cristea

Proxy ARP este o variantă de protocol


ARP, ı̂n care un dispozitiv intermediar cum
ar fi router-ul, trimite un răspuns ARP către
gazda care a efectuat cererea. Router-ele care
,,ı̂nt, eleg” proxy ARP vor răspunde cu propria
adresa MAC la acele cereri pentru care adresa
IP nu se găses, te ı̂ntre segmentele pe care le
conectează. Practic va transmite adresa fizică
a interfet, ei care se găses, te ı̂n acelas, i segment
cu gazda.

Dacă router-ul observă că IP-ul pentru care se cere adresa fizică este local, atunci va ignora
cererea. Motivul pentru această decizie este că cererea are o adresă cu destinat, ie multiplă
(broadcast) s, i toate stat, iile au primit-o. Una dintre ele este destinatarul pentru care se cere
MAC-ul s, i aceasta va răspunde stat, iei care a efectuat apelul.
Protocol rutabil
IP este un protocol corespunzător nivelu-
lui ret, ea din modelul OSI s, i care se poate ruta
ı̂ntr-un sistem de ret, ele, cum ar fi Internetul.
Protocoalele care oferă suport pentru nivelul
ret, ea se numesc protocoale rutabile (routed
protocols sau routable protocols). Des, i IP
este cel mai cunoscut protocol rutabil, mai
există s, i altele, cum ar fi IPX/SPX sau Apple-
Talk. Din categoria protocoalelor care nu se
pot ruta se poate exemplifica NetBEUI, un
protocol rapid s, i eficient dacă se foloses, te ı̂ntr-
un singur segment de ret, ea.

Pentru ca un protocol să poată fi rutabil, trebuie să fie capabil de a asigna adresă unei ret, ele
s, i fiecărei gazde din ret, ea. Protocoale cum ar fi IPX necesită numai adresa ret, elei, deoarece
foloses, te adresa fizică pentru identificare. IP necesită furnizarea adresei complete plus masca de
ret, ea. Printr-o operat, ie logică se obt, ine adresa de ret, ea.
Pentru ca un protocol să poată fi rutabil, trebuie să fie capabil de a asigna adresă unei ret, ele
s, i fiecărei gazde din ret, ea. Protocoale cum ar fi IPX necesită numai adresa ret, elei, deoarece
foloses, te adresa fizică pentru identificare. IP necesită furnizarea adresei complete plus masca de
ret, ea. Printr-o operat, ie logică se obt, ine adresa de ret, ea.
Protocol de rutare
Protocolul de rutare (routing protocol) (a nu se confunda cu protocolul rutabil!) are rolul de
a determina calea pe care protocoalele rutabile trebuie să o străbată către destinat, ie. Exemple
de asemenea protocoale sunt: Routing Information Protocol (RIP), Interior Gateway Routing
Protocol (IGRP), Enhanced Interior Gateway Routing Protocol (EIGRP), s, i Open Shortest
Path First (OSPF).
Protocolul de rutare ajută router-ul să-s, i construiască o hartă internă cu router-ele vecine,
care este suportul pentru select, ia rutei optime sau comutarea pachetelor. Această hartă este o
parte din tabela de rutare a fiecărui router.
Protocoale 29

Router-ele folosesc protocoalele de rutare pentru a face schimb de tabele de rutare sau pentru
a partaja informat, ii de rutare. În interiorul unei ret, ele, cel mai cunoscut protocol folosit pentru a
schimba informat, ii de rutare ı̂ntre router-e (din aceeas, i ret, ea) este Routing Information Protocol
(RIP), care apart, ine unei categorii numite Interior Gateway Protocol (IGP). El calculează
distant, a până la destinatar exprimând-o ı̂n numărul de noduri (router-e sau hop-count) prin
care se trece. RIP permite router-elor să facă schimb de tabele de rutare la fiecare 30 de secunde.
Un dezavantaj al acestui protocol este că necesită o conectare permanentă ı̂ntre router-ele vecine
iar schimbul de tabele consumă o parte din lăt, imea de bandă. Dacă aceasta nu este suficient de
mare, traficul va avea de suferit.
Pentru a ajuta router-ele să determine ce cale să folosească ı̂n transmiterea datelor, RIP se
bazează pe un concept numit vector-distant, ă (distance-vector). De fiecare dată când un pachet
trece printr-un router către o nouă ret, ea, se consideră că a trecut de un hop. O cale care are un
hop-count egal cu patru ı̂nseamnă că pachetul trebuie să treacă prin patru router-e pentru a
ajunge la ret, eaua care cont, ine destinatarul. Dacă există mai multe rute către destinat, ie, atunci
router-ul va alege pe aceea care are un hop-count minim.
Deoarece vectorul distant, ă are un singur element de analiză, nu rezultă că ruta furnizată
este s, i cea mai rapidă. Alte elemente pot să influent, eze luarea deciziei, cum ar fi lăt, imea de
bandă, capacitatea de trecere (throughput) la diferite momente, etc. există protocoale de rutare
care analizează aceste informat, ii. Cu toate acestea, RIP rămâne ı̂ncă foarte popular s, i este
implementat de multe router-e, poate s, i datorită faptului că a fost primul protocol cu această
funct, ie.
O altă problemă care poate să apară dacă este folosit RIP-ul este atunci când destinat, ia este
prea ı̂ndepărtată pentru a fi atinsă. RIP limitează numărul maxim de hopuri la cincisprezece.
Dacă nu se poate ajunge la destinat, ie cu acest număr, atunci se concluzionează că ret, eaua nu se
poate accesa.
Se poate ment, iona că un router suportă ı̂n acelas, i timp mai multe protocoale de rutare
s, i actualizează permanent tabelele pentru cel mai cunoscute protocoale rutabile. Această
particularitate permite router-ului să transmită pachete de la diferite protocoale rutabile pe
aceeas, i legătură fizică.
Serviciu de ret, ea fără conexiune
O mare parte din serviciile unei ret, ele
sunt realizate fără conexiune (connectionless).
Fiecare pachet este tratat separat, ceea ce
ı̂nsemnă că pot ajunge pe rute diferite la
destinat, ie. Ele vor fi ı̂nsă reasamblate de
către destinatar după primirea ı̂ntregului set.
Destinatarul nu este contactat ı̂nainte de tri-
mite un pachet.

O comparat, ie se poate face cu sistemul pos, tal; expeditorul pune o scrisoare la pos, tă fără să
ı̂l anunt, e pe destinatar. Acesta va afla de scrisoare atunci când o va primi.
Serviciu de ret, ea cu conexiune
Serviciul de ret, ea cu conexiune (connection-oriented) stabiles, te o legătură directă ı̂ntre
expeditor s, i destinatar ı̂naintea ı̂nceperii transferului de date. Dacă legătura nu poate fi
30 M. Cristea

realizată, niciun transfer nu are loc. Un exemplu poate fi sistemul de telefonie unde mai ı̂ntâi se
realizează o legătură directă ı̂ntre apelant s, i apelat după care se trece la efectuarea convorbirii.
Serviciul de ret, ea fără conexiune se mai numes, te s, i comutare de pachete. Deoarece acestea
folosesc diferite rute, unele dintre ele pot să nici nu ajungă. Criteriile de select, ie a rutei pot să
difere de la un moment la altul, mai ales la protocoalele de rutare mai performante (nu RIP!).
Serviciul de ret, ea cu conexiune se mai numes, te s, i comutare de circuit. Se creează un circuit
virtual care este ment, inut atâta timp cât este nevoie pentru transferul datelor, după care este
ı̂ntrerupt. Toate pachetele vor folosi aceeas, i cale.
Internetul este o imensă ret, ea fără conexiune ı̂n care pachetele sunt manevrate de IP.
Protocolul TCP, care apart, ine nivelului 4 din modelul OSI, poate să adauge serviciul de
conexiune care implică sigurant, ă ı̂n livrarea datelor.
IP este un serviciu de ret, ea fără conexiune; fiecare pachet este tratat independent. De
exemplu, dacă se descarcă un fis, ier cu aplicat, ia FTP, IP nu va trimite fis, ierul ı̂ntr-un mare
flux de date. Fiecare poate avea o rută diferită de a celorlalte, sau se pot chiar pierde. În
aceste situat, ii intervin protocoalele nivelului Transport din modelul OSI care vor determina ce
pachete s-au pierdut s, i vor cere retransmiterea. Tot la nivelul transport se va realiza reordonarea
pachetelor chiar dacă acestea au sosit la momente diferite.
Particularităt, i ale tabelelor de rutare
Cu ajutorul tabelei de rutare un dispozitiv de ret, ea cum ar fi puntea sau switch-ul, leagă o
adresă IP de adresa fizică. Acestea intră in tabelă numai dacă dispozitivul care posedă este
,,văzut” ı̂n ret, ea ı̂n mod direct, fără intermediari. Se cunoas, te foarte put, in sau deloc despre ce
este ,,dincolo” de ret, eaua proprie.
Router-ele construiesc tabele care descriu toate ret, elele conectate la ele. Dacă s-ar memora
date numai despre vecinii router-elor, atunci mecanismul de select, ie a rutei nu ar funct, iona
eficient. În tabelele router-elor se găsesc informat, ii despre ret, ele aflate la multe hopuri distant, ă;
cu cât acest număr este mai mare cu atât alegerea rutei este mai bună.
Există situat, ia ca un router să primească un pachet a cărui destinat, ie să nu se găsească ı̂n
tabela de rutare. Pentru aceste cazuri el trebuie să fie configurat cu o adresă implicită unde
va livra pachetele; de obicei acestea se vor livra către un router cu mai multe conexiuni care
să aibă o perspectivă mai largă despre ret, elele din jurul lui. Dacă nici acest router nu găses, te
adresa destinatarului ı̂n tabele, atunci pachetul se ignoră.
O altă situat, ie ı̂n care router-ul nu poate să livreze mai departe un pachet ester atunci
când nu cunoas, te adresa MAC a următorului router. Se va proceda analog cu o stat, ie dintr-o
ret, ea locală, dar ı̂ntre router-e. Pe interfat, a unde dores, te să transmită pachetul, router-ul va
transmite un ARP request. Această cerere va ajunge la router-ul unde se dores, te transmiterea
pachetului, care va răspunde cu un ARP reply ı̂n care a pus s, i propria adresă MAC.
Rutare indirectă
Prin mecanismul numit Proxy ARP o
gazdă care nu cunoas, te adresa MAC a des-
tinatarului va primi din partea router-ului
adresa MAC a interfet, ei care conectează
(sub)ret, eaua gazdei respective. Router-ul cu
acest rol poartă numele de default gateway.
Protocoale 31

La ı̂ncapsularea pachetului, gazda va pune la adresa IP adresa destinatarului, iar la adresa


MAC va fi adresa interfet, ei router-ului.
În momentul primirii pachetului, router-ul observă cu nu este ultimul destinatar s, i ı̂s, i
consultă tabela de rutare pentru a identifica adresa IP din pachet. Dacă o găses, te, atunci
ı̂nlocuies, te adresa MAC a propriei interfet, e cu adresa MAC a router-ului care conectează ret, eaua
destinatarului, sau care reprezintă următorul hop (nod de ret, ea sau router) ı̂n drumul către
destinatar. Dacă tabela de rutare nu cont, ine adresa destinatarului, atunci va transmite pachetul
cu adresa MAC a router-ului implicit, dacă acesta a fost definit. Pachetul se pierde dacă nu
există un router implicit. Acest mecanism de ı̂nlocuire a adresei MAC la fiecare router prin care
pachetul trece ı̂n drumul către destinatar poartă numele de rutare indirectă.
Alte protocoale de rutare
As, a cum s-a amintit anterior, un protocol care se poate ruta ı̂ntre ret, ele poartă numele de
protocol rutabil. Alături de cel mai cunoscut protocol rutabil, IP, se mai pot adăuga IPX s, i
AppleTalk.
Router-ele folosesc protocoalele de rutare pentru a face schimb de tabele de rutare sau par-
tajarea informat, iilor. Altfel spus, protocoalele de rutare determină cum se pot ruta protocoalele
rutabile. Exemple de protocoale de rutare sunt RIP, IGRP, EIGRP, OSFP.
Protocoalele de rutare se pot clasifica ı̂n două categorii: interioare (Interior Gateway Protocol
- IGP) s, i exterioare (Exterior Gateway Protocol - EGP). Toate protocoalele de rutare enumerate
mai sus apart, in categoriei IGP. Un exemplu de protocol exterior este Border Gateway Protocol
– BGP.
RIP, ale cărui origini pornesc de la firma Xerox, a fost abordat ı̂n paragrafele anterioare.
IGRP s, i EIGRP au fost dezvoltate de corporat, ia Cisco Systems, care este considerată
proprietara acestor protocoale.
Un protocol precum IGRP (Interior Gateway Routing Protocol) a fost necesar pentru a
rezolva problemele apărute ı̂n sistemele de ret, ele de mari dimensiuni unde popularul RIP nu se
mai putea utiliza eficient. Analog RIP, IGRP este un protocol care foloses, te vectorul distant, ă ı̂n
alegerea celei mai bune rute, ı̂nsă cu mai multe criterii de decizie. Pe lângă numărul de hopuri,
se mai ia ı̂n considerare lăt, imea de bandă, gradul de utilizare, ı̂ntârzierile, s, i sigurant, a rutei. De
asemenea, administratorul de ret, ea poate să asocieze fiecărui criteriu un coeficient, influent, ând
astfel alegerea rutei după propria analiză. Numărul maxim de hopuri pe care un pachet poate
să le străbată ı̂n drumul către destinatar este 255.
EIGRP (Enhanced Interior Gateway Routing Protocol) este o versiune avansată a IGRP,
care poate să aplice funct, ii complexe ı̂n alegerea rutei. Se poate chiar renunt, a la vectorul
distant, ă dacă altă metodă este mai bună la un moment dat, pentru a reveni atunci când este
cazul.
OSFP (Open Shortest Path First) este un protocol performant care nu foloses, te vectorul
distant, ă. Router-ul care aplică acest protocol va construi o structură ierarhizată a router-elor
de a căror existent, ă s, tie, rădăcina fiind chiar el. Fiecare legătură ı̂ntre două router-e va avea un
cost strict pozitiv, care se calculează ı̂n funct, ie de diferit, i factori, cum ar fi viteza, intensitatea
traficului, sigurant, a legăturii s, i securitatea datelor. Către fiecare router cunoscut se va calcula
drumul optim folosind algoritmi adecvat, i. Un asemenea algoritm este algoritmul lui Dijkstra.
Ruta se recalculează de fiecare dată când se schimbă un criteriu care a fost folosit ı̂n select, ie.
32 M. Cristea

Informat, iile care se găsesc ı̂n tabelele de rutare se pot introduce manual de către administra-
torul ret, elei sau se pot adăuga automat printr-o ,,corespondent, ă” ı̂ntre router-e, atunci când
acestea ı̂s, i desfăs, oară activitatea. Dacă administratorul introduce manual o rută atunci aceasta
se va numi rută statică, toate celelalte numindu-se rute dinamice.
Dacă router-ul poate să ı̂nvet, e informat, iile de rutare automat poate părea fără sens intervent, ia
administratorului. Cu toate acestea, există situat, ii când se impune o configurare manuală.
Exemple ar putea fi:

1. Verificarea traficului pe o anumită legătură fizică. Se va configura router-ul astfel ı̂ncât


alte rute, des, i mai bune, să nu fie folosite.
2. Controlul select, iei realizate de router. Se poate testa dacă ruta aleasă la un moment dat
de router este cea mai bună sau nu.
3. Dacă există o singură legătură către o anu-
mită ret, ea. În această situat, ie chiar se
impune intervent, ia administratorului, deo-
arece nu are sens ca router-ul să consume
timp ı̂n căutarea altor variante de acces din
moment ce se s, tie că nu există. Ret, elele
cu un singur drum de acces poartă numele
de ,,stub network”.

Rutarea dinamică este atunci când router-ele ı̂s, i trimit periodic mesaje de actualizare a
tabelelor de rutare. De fiecare dată când un router primes, te un mesaj de la un alt router, el va
recalcula toate drumurile afectate de modificarea primite s, i va anunt, a la rândul lui mai departe.
Tabelele de rutare rămân astfel adaptate la condit, iile ret, elei.
Înainte de aparit, ia rutării dinamice, distribuitorii de router-e trebuiau să configureze fiecare
aparat ı̂n parte ı̂n funct, ie de natura ret, elelor pe care le conecta. Ei se deplasau la fiecare
modificare a ret, elei să reconfigureze router-ul. Pe măsură ce ret, elele au ı̂nceput să se dezvolte
ca număr de stat, ii sau locat, ii geografice, configurarea manuală era din ce ı̂n ce mai dificilă.
Rutarea dinamică a devenit o necesitate, fără de care Internetul nu ar exista. Toate protocoalele
descrise mai sus suportă rutarea dinamică.
Rutarea cu RIP
Să presupunea că dispunem de o ret, ea din
clasa B ı̂mpărt, ită ı̂n 8 subret, ele cu ajutorul
a 3 router-e. Gazda A dores, te să trimită un
pachet gazdei Z. Datele coboară de la nivelul
Aplicat, ie către nivelul Fizic. Nivelul Trans-
port ı̂mparte datele ı̂n segmente de lungimi
egale. La nivelul Ret, ea se completează adresa
IP a sursei s, i a destinatarului, transformând
segmentul ı̂ntr-un pachet. Nivelul imediat
următor, Legături de Date, adaugă la sursă
adresa MAC a gazdei iar la destinat, ie adresa
interfet, ei router-ului care conectează segmen-
tul gazdei.

Pachetul a devenit la nivelul 2 un cadru. Gazda A ,,vede” subret, eaua 8 ca o ret, ea separată,
Protocoale 33

deci nu va putea trimite datele direct, ci numai prin intermediul portului implicit (default
gateway) care este interfat, a router-ului 1.
Pe măsură ce cadrul se transmite ı̂n subret, eaua 1, nicio gazdă nu face copii dacă adresa MAC
nu se potrives, te. Router-ul este cel care preia datele ı̂n vederea prelucrării. El va extrage adresa
MAC s, i transferă datele nivelului Ret, ea care va verifica adresa IP. Adresa este corespunzătoare
unei gazde din subret, eaua 8, iar protocolul RIP determină că se poate ajunge acolo dacă se
trece de trei hopuri cu ies, ire prin interfat, a către subret, eaua 4.
Pachetul coboară din nou la nivelul 2 unde primes, te o nouă adresă MAC, aceea a interfet, ei
router-ului 2, ı̂n timp ce adresa IP rămâne neschimbată. După completarea adresei pachetul se
transmite in subret, eaua 4 către toate gazdele.
În subret, eaua 4 nicio gazdă nu preia pachetul deoarece adresa MAC nu se potrives, te. Router-
ul 2 observă că adresa din pachet coincide cu adresa interfet, ei, deci decapsulează cadrul s, i
transmite pachetul nivelului Ret, ea. Protocolul de rutare determină că ret, eaua destinatară este
cu două hopuri mai departe prin interfat, a corespunzătoare subret, elei 5.
Pachetul coboară la nivelul Legături unde adresa MAC a destinatarului se schimbă din nou
cu adresa interfet, ei router-ului 3. Este transmis ı̂n subret, eaua 5 cu aceeas, i adresă IP.
Router-ul 3 se recunoas, te ca destinatar al pachetului s, i ı̂l preia. La nivelul Ret, ea se determină
că destinatarul pachetului este o gazdă din subret, eaua 8, care este atas, ată direct la una din
interfet, e. Se coboară pachetul la nivelul 2 unde adresa MAC sursă devine adresa router-ului 3
iar adresa MAC destinat, ie va fi adresa gazdei Z pe care router-ul o cunoas, te. Adresa IP nu se
schimbă ı̂n timp ce pachetul este transmis ı̂n subret, eaua 8.
Fiecare stat, ie din subret, ea verifică pachetul dar numai stat, ia Z decide la nivelul 2 că adresa
MAC coincide s, i la nivelul Ret, ea că adresa IP din pachet este propria adresă. Datele sunt
decapsulate pe măsură ce urcă ı̂n nivelurile modelului OSI, până ce ajung la nivelul Aplicat, ie,
care le va oferi aplicat, iei utilizatorului.

Bibliografie

[1] http://www.cisco.com/

[2] http://www.ietf.org/
Concursul ,,Dräexlmaier IT Day”, Edit, ia a II-a, ianuarie
2020

Mihai-Radu Găman 1 s, i Maria Miroiu 2

Departamentul IT Center Pites, ti din cadrul companiei DRÄXLMAIER Group a


organizat ı̂n ianuarie 2020 a doua edit, ie a concursului ,,Dräxlmaier IT Day”. Au
participat la concurs student, i de la domeniul de licent, ă Informatică din cadrul Facultăt, ii de
S, tiint, e, Educat, ie Fizică s, i Informatică, Universitatea din Pites, ti.

Data limită de depunere a proiectelor a fost 21 ianuarie 2020, iar prezentarea proiectelor s-a
desfăs, urat pe 28 ianuarie 2019. Nu au fost restrict, ii ı̂n ceea ce prives, te limbajele de programare.

Premiile oferite: premiul I a constat dintr-un voucher de cumpărături pe platforma Emag ı̂n
valoare de 1400 lei s, i a fost câs, tigat de studenta Năstase Mariana Mădălina de la Informatică,
anul 3, premiul al II-lea a constat dintr-un voucher de cumpărături pe platforma Emag ı̂n
valoare de 700 lei s, i a fost câs, tigat de studentul Anghel Valentin George de la Informatică, anul
2, iar premiul al III-lea a constat dintr-un voucher de cumpărături pe platforma Emag ı̂n valoare
de 250 lei s, i a fost câs, tigat de o echipă de 3 student, i de la Informatică, anul 3: Ciolcă Andrei
Iulian, Zamfir Andrei-Gabriel s, i Drăgus, in Răzvan-Vasile. Juriul a fost compus din mai mult, i
specialis, ti din cadrul IT Center Pites, ti, Draexlmaier Group.

Scopul principal a constat ı̂n implementarea unui sistem de aplicat, ii prin intermediul
căreia utilizatorii să poată realiza o rezervare a unei săli de sedint, e. De asemenea, un
raport al acestor rezervări va trebui să fie disponibil la cerere. Datele vor fi stocate ı̂ntr-o bază
de date relat, ionale (MS Access, mySql, Oracle, etc.).

1 Model entităt, i

Informat, iilor stocate ı̂n sistem (tabele ı̂n baze de date relat, ionale) le sunt asociate următoarele
entităt, i: <camera sedinte>, <rezervare> s, i <utilizator>.

Diagrama conceptuală a acestor entităt, i s, i relat, iile dintre ele sunt descrise ı̂n imaginea
următoare.

1
Head of Delivery Supply Chain Management, IT Center Pites, ti, Dräexlmaier Group, Pites, ti,
mihai.gaman@draexlmaier.com
2
Lect. univ. dr., Universitatea din Pites, ti, maria.miroiu@gmail.com

34
Concursul ,,Dräexlmaier IT Day”, Edit, ia a II-a 35

Figura 1. Relat, iile ı̂ntre entităt, i

2 Descrierea relat, iilor

A – o <camera sedinte> poate fi rezervată o singură dată ı̂ntr-un interval orar dat;
B – un <utilizator> poate rezerva una sau mai multe săli de s, edint, e chiar s, i ı̂n acelas, i interval
de timp;
Această relat, ie defines, te de fapt cine este organizatorul rezervării.
C – defines, te lista participant, ilor la o sedint, ă dată de o rezervare;
D – defines, te care sunt utilizatorii participant, i la o s, edint, ă dată de o rezervare.

Entitatea <camera sedinte>


Reprezentarea unei săli de s, edint, e ı̂n sistemul nostru va avea un indentificator unic, dar s, i
un nume.

Atribut Tip Validare Descriere


Indentificatorul va fi o cheie artifi-
Id Text Obligatoriu
cială - un număr generat ı̂n secvent, ă
Numele sălii – va fi administrat de
Nume Text Obligatoriu
către utilizatori
Adresa Text Obligatoriu Adresa sălii de s, edint, e
Numărul maxim de participant, i ı̂ntr-
Capacitate Maxima Număr Obligatoriu
o s, edint, ă ı̂n acea sală

Entitatea <utilizator>
Un <utilizator> este o entitate ce va fi folosită pentru a stoca date despre utilizatorii
sistemului de rezervări.
36 M.R. Găman, M. Miroiu

Atribut Tip Validare Descriere


O cheie naturală sau artificială (de ex. CNP, număr
Id Text Obligatoriu
de marcă sau un număr oarecare)
Nume Text Obligatoriu Numele organizatorului
Emailul organizatorului, care va fi folosit pentru
Email Text Obligatoriu
validarea unei rezervări, dar s, i pentru un reminder
Un flag ce va indica dacă utilizatorul poate init, ia
Este Organizator Boolean Obligatoriu
sau nu rezervări

Entitatea <rezervare>
Entitatea va defini relat, ia de mai mult, i la mai mult, i ı̂ntre <sala sedinte> s, i <utilizator>.
Adiacent va stoca s, i informat, ii referitoare la statusul rezervării: dacă a fost sau nu confirmată
s, i cine este organizatorul.

Atribut Tip Validare Descriere


Identificatorul unic al rezervării (poate fi o secvent, ă
Id Numar Obligatoriu
numerică)
Description Text Obligatoriu Motivul rezervării
Id Organizator Text Obligatoriu Id-ul <utilizator>-ului care face rezervarea
Flag ce indică dacă s-a făcut confirmarea rezervarii.
Este Confirmata Boolean Obligatoriu Este folosită pentru securitate – utilizatorul va
primi un email pentru a confirma rezervarea.

Entitatea <participanti>
Această entitate se referă la o funct, ionalitate a sistemului ERP ce face posibilă startarea
unei proceduri de către utilizator.
Atribut Tip Validare Descriere
Indentificatorul unei rezervări, care face parte
Id rezervare Număr Obligatoriu din cheia primară ı̂mpreună cu id-ul utilizatorilor
(participant, ilor)
Id <utilizator>, necesar pentru a indentifica tot, i
Id Participant Text Obligatoriu
participant, ii ı̂ntr-o anume s, edint, ă rezervată
Valoare logică ce indică dacă participantul res-
A Confirmat Boolean Obligatoriu
pectiv a confirmat sau un participarea

3 Cerint, e funct, ionale

Proiectul presupune realizarea următoarele componente:

- o aplicat, ie care să permită introducerea unei noi rezervări ı̂n sistem;
- o aplicat, ie care să permită vizualizarea listei de rezervări.

Mentionăm că init, ializarea tabelei de utilizatori s, i cea a sălilor de s, edint, e se poate face prin
intermediul unui script (fără a fi nevoie de interfat, a grafică pentru administrare). Exemple de
date de test sunt furnizate ı̂n Anexa 1.
Concursul ,,Dräexlmaier IT Day”, Edit, ia a II-a 37

3.1 Aplicat, ie care să permită introducerea unei noi rezervări ı̂n sistem

Pagina de login

- utilizatorul va furniza adresa de email;

- ı̂n cazul logării cu succes, el va fi


redirect, ionat direct către pagina de cre-
are rezervare;

- altfel, va fi afis, at mesajul ”Combinatia


email/parola nu exista”.

Pagina de alegere cameră

- utilizatorul va putea selecta o cameră


de s, edint, ă deja introdusă ı̂n sistem;

- ı̂n cazul ı̂n care avem multe camere de


s, edint, e introduse ı̂n sistem, atunci este
bine să avem un ”live search” ı̂n care
aplicat, ia să afis, eze doar camerele care
sunt conform pattern-ului introdus de
utilizator (de exemplu, dacă utilizato-
rul introduce ’PIT’, sistemul va propune
toate sălile de sedint, e ce cont, in cuvântul
PIT: VR PIT 5 8 I, CR PIT 5 6 I, CR
PIT 5 6 II etc.);

Pagina de alegere a perioadei rezervării

- utilizatorul trebuie să selecteze data de


ı̂nceput s, i de final ale rezervării;

- dacă sala respectivă este deja rezervată


ı̂n perioada respectivă, atunci aplicat, ia
va semnaliza acest lucru printr-un me-
saj: ”Există deja o rezervare pentru
această perioada (de la: 2019.11.13
14:00 la 2019.11.13 18:00) efectuată de
Gigel Ion”).
38 M.R. Găman, M. Miroiu

Pagina de selectare a participant, ilor

- utilizatorul va putea sa selecteze


participant, i la sedint, ă, ı̂nsă nu este obli-
gatoriu să facă acest lucru;

- după ce se apasă butonul Submit, utili-


zatorul va fi redirect, ionat către pagina
care afis, ează detaliile sedint, ei.

Pagina de confirmare a s, edint, ei

- utilizatorul poate verifica detaliile re-


zervării, conform imaginii din stânga;

- ı̂n cazul ı̂n care vrea sa modifice detali-


ile, poate folosi butonul Back, succesiv,
până ajunge la pagina cu detaliile do-
rite;

- ı̂n cazul ı̂n care apasă Submit, re-


zervarea este efectuată ı̂n baza de
date, iar un email de informare va fi
direct, ionat către Organizator dar s, i
către Participant, i.

Utilizatorul va fi capabil să se ı̂ntoarcă la pagina anterioară de fiecare dată apăsând butonul
Back. Informat, iile selectate s, i introduse pe pagină vor fi salvate, astfel ı̂ncât ı̂n momentul
reintoarcerii pe o pagină, utilizatorul va vedea datele introduse anterior. De exemplu, utilizatorul
tocmai a selectat 3 participant, i, Ion, Gigel s, i Vasile, dar ı̂s, i dă seama că are nevoie de o altă
cameră de s, edint, ă. Va apăsa butonul Back până la pagina de select, ie a camerei de s, edint, e. Apoi
va putea să schimbe camera cu alta. După ce apasă butonul Submit va putea să vizualizeze
intervalul orar deja selectat pentru sedint, ă. De asemenea, va putea să schimbe acest interval.
După Submit, va putea să acceseze din nou participant, ii selectat, i anterior pe pagina de select, ie
a participant, ilor.

Diagrama de secvent, e UML

În continuare avem reprezentarea ı̂n standard UML pentru stările prin care poate trece
aplicat, ia:
Concursul ,,Dräexlmaier IT Day”, Edit, ia a II-a 39

3.2 Aplicat, ie care să permită vizualizarea rezervărilor unei camere de


s, edint, e

Pentru a folosi ı̂ntr-un mod cât mai util acest sistem de rezervări, fiecare sală de s, edint, e va
avea o tabletă la intrare care va afis, a detalii despre rezervarea curentă s, i rezervările ulterioare.
Aplicat, ia va trebui să reı̂mprospateze date la fiecare minut. De asemenea, select, ia camerei
pentru care se face afis, area să fie făcută o singură dată, ı̂n momentul instalării aplicat, iei.

Pagina de afis, are rezervare curentă - rezervare ı̂n curs

- afis, ează ı̂n partea de sus numele


sălii de s, edint, e s, i numărul maxim de
participant, i;
- la mijloc, se vor afis, a, pe un backgro-
und rosu: descrierea rezervării, până
când durează rezervarea, organizatorul
s, i participant, ii la s, edint, ă;
- ı̂n partea de jos se vor afis, a informat, ii
(aceleas, i detalii ca mai sus) despre
următoarea sedint, ă programată, iar
dacă nu există altă rezervare, nu se va
afis, a nimic.
40 M.R. Găman, M. Miroiu

Pagina de afis, are rezervare curentă - fără rezervare ı̂n curs


- afis, ează ı̂n partea de sus numele
sălii de s, edint, e s, i numărul maxim de
participant, i;
- la mijloc, se vor afis, a, pe un backgro-
und verde până când este disponibilă
camera; ı̂n cazul ı̂n care nu există re-
zervări, se va afis, a doar mesajul ”Ca-
mera disponibila”;
- ı̂n partea de jos se vor afis, a informat, ii
despre următoarea s, edint, ă programată:
descrierea rezervării, până când durează
rezervarea, organizatorul, participant, ii
la s, edint, ă;
- nu se va afis, a nimic dacă nu există altă
rezervare.

4 Cerint, e tehnice

• Sistemul realizat poate fi dezvoltat ı̂n orice technologie. Pentru a doua aplicat, ie, cea
care va afis, a continuu date despre sala de s, edint, e, este indicat folosirea unei technologii
compatibile cu dispozitive mobile: Android, Angular, aplicatii web ce pot rula pe orice
dispozitiv ce dispune de browser.
• Bazele de date pot fi sisteme de gestiune a datelor (Oracle, mySql etc.) sau pot fi simple
fisiere text, xml, json, s, .a.m.d. În al doilea caz, integritatea datelor s, i a relat, iilor va trebui
asigurată ı̂n interiorul aplicat, iei de rezervări.

5 Anexa 1

• Camere de s, edint, e:
Id Nume Adresa Capacitate Maxima
1 VR PIT 5 8 II Pitesti 5
2 VR PIT 5 8 I Pitesti 6
3 CR PIT 5 8 III Pitesti 3
4 VR PIT 5 6 I Pitesti 2
5 VR TEM 5 6 II Timisoara 1
6 VR PIT 2 1 I Pitesti 2
• Utilizatori:
Id Nume Email Este Organizator
1402478 Gaman Mihai mihai.gaman@draexlmaier.com TRUE
1402479 Ionescu Gheorghe abu.1@yahoo.com TRUE
1402480 Popescu Ghita gita.pope@gmail.com TRUE
1402481 Mihaela Berta gigi.miky@email.ro TRUE
RUBRICA DE ROBOTICĂ

Repetarea operat, iilor unui robot Mindstorms Education


EV3

Doru Anastasiu Popescu 1

Introducere

În mediul de programare Mindstorms EV3 grupul Flow Control (figura 1) cont, ine blocul Loop,
care ne permite să repetăm unul sau mai multe blocuri.

Fig. 1: Grupul Flow Control

Condit, ia de oprire a execut, iei blocurilor din corpul blocului Loop poate cont, ine informat, ii de
la senzori sau expresii logice. Blocul Loop se termină (figura 2) conform cu modul de configurare
True, respectiv False ı̂n zona de setare.

Fig. 2: Blocul Loop

Problemă rezolvată

Pentru a pune ı̂n evident, ă modul de repetare a operat, iilor pe care le poate face un robot
Minstorms EV3 prezentăm aplicat, ia 1.
Aplicat, ia 1. Trebuie sa realizăm un proiect pentru un concurs, care prin intermediul robotului
să numere câte discuri de culori diferite de alb (culoarea plans, ei) se găsesc pe o linie dreaptă
de lungime L cm. L este un număr natural de două cifre generat aleator. În regulamentul
concursului este precizat faptul că robotul poate poarcurge distant, a o singură dată.
1
Conf. univ. dr., Universitatea din Pites, ti, dopopan@yahoo.com

41
42 D.A. Popescu

Solut, ie. Vom scrie partea cea mai complicată a proiectului, după care vom trece la copierea unei
port, iuni din program s, i lipirea ei de mai multe ori, ca să acopere toată distant, a cea mai lungă
(99 cm) ce poate să apară. În program se folosesc următoarele variabile: L pentru lungimea
traseului (generată aleator, ca să verifice programul ı̂n mai multe situat, ii), rot pentru numărul
de rotat, ii corespunzătoare distant, ei L, k numărul de rotat, ii până la un moment dat, Nr numărul
de buline de culoare diferită de alb ı̂ntâlnite pe traseu. La o rotat, ie se parcurg aproximativ 18
cm (valoare memorată ı̂ntr-o constantă) dacă se folosesc rot, i de diametrul 5,5 cm. În program
se vor introduce s, i două blocuri Switch, primul pentru a verifica cu senzorul de culoare, dacă se
ı̂ntâlnes, te o bulină colorată, iar al doilea pentru a verifica dacă numărul de rotat, ii până la un
moment dat, nu depăs, es, te valoarea lui rot.

Fig. 3: Programul de numărare a discurilor fără bloc de repetare

Pentru că trebuie să parcurgem cel mult 99 cm, atunci blocurile ar trebui repetate de 99/0.25
ori. Se poate mics, ora acest număr dacă se ı̂nlocuies, te ı̂n blocul Move Steering valoarea 0.25
cu o valoare mai mare, dar există riscul să nu mai numere toate bulinele. Acest lucru poate fi
evitat folosind blocul Loop (care are ca efect repetarea execut, iei s, irului de blocuri din interiorul
său până când o anumită expresie logică este adevărată).

Fig. 4: Programul de numărare a discurilor cu bloc de repetare (Loop)


Repetarea operat, iilor unui robot Mindstorms Education EV3 43

Probleme propuse

Pentru fiecare din problemele următoare scriet, i câte un proiect folosind mediul interactiv de
programare Mindstorm Evolution EV3.

1. Se dă un traseu ı̂n formă dreptunghiulară (figura 5) cu laturile de lungime a s, i b (unitatea


de măsură este centimetrul, 1≤a,b≤60). Deplasat, i robotul pe acest traseu pornind dintr-un
colt, s, i avertizat, i sonor când numărul de rotat, ii este par. Se va folosi sunetul unei culori
(exemplu: red – ros, u).

Fig. 5: Traseu dreptunghiular cu sensul de deplasare precizat


2. Pe o plans, ă se află un cub s, i la o distant, ă de cel mult 50 cm de acesta un robot. Deplasat, i
robotul până la cel put, in 7 cm de cub s, i afis, at, i pe ecran distant, a parcursă. Afis, area se va
păstra pe o perioadă de 5 secunde.
3. Simulat, i modul de parcare a unei mas, ini folosind un robot. Proiectat, i o parcare cu trei
locuri pe partea dreaptă (figura 6). Plasat, i aleator ı̂n două din locuri cuburi ce reprezintă
mas, ini stat, ionate de culori verde s, i galben s, i deplasat, i robotul astfel ı̂ncât să se as, eze ı̂n
locul liber, fără să atingă cuburile.

Fig. 6: Parcare cu specificarea locului de parcare

Bibliografie

[1] D.A. Popescu, Programarea robot, ilor LEGO folosind mediul Mindstorms EV3, MATINF, nr.
1, 2018.

[2] D.A. Popescu, Programarea robot, ilor Lego folosind structura alternativă ı̂n mediul grafic
interactiv Mindstorms Education EV3, MATINF, nr. 2, 2018.

[3] D.A. Popescu, S. Profeanu, S. Dobrescu, Manual de informatică pentru clasa a V-a, Editura
CD-Press, 2017.

[4] Minsdtorms EV3 – Ghid de Utilizare, LEGO Group, 2013.

[5] L. Negrescu, L. Negrescu, Construirea s, i programarea robot, ilor LEGO Mindstorms EV3,
Editura Albastră, 2015.

[6] J. Olayvar, E. Lindberg, LEGO Mindstorms EV3 Programming Basics, Washington State
Library, 2016.
44 PROBLEME DE MATEMATICĂ PENTRU EXAMENE

PROBLEME DE MATEMATICĂ PENTRU


EXAMENE

Teste pentru examenul de Evaluare Nat, ională

Testul 1

Adrian Ţurcanu 1

SUBIECTUL I
Încercuies, te litera corespunzătoare răspunsului corect.
1. Cel mai mic multiplu comun al numerelor 125 şi 200 este:
a) 1; b) 25000; c) 1000; d) 500.
2. Suma numerelor de forma 2aa divizibile cu 5 este:
a) 200; b) 255; c) 455; d) 901.
3. Suma celor mai mici 25 de numere naturale nenule distincte este:
a) 300; b) 325; c) 351; d) 378.
4. Un produs costă 120 lei. După o scumpire cu 20% preţul produsului va fi:
a) 122 lei; b) 134 lei; c) 140 lei; d) 144 lei.
5. Notele obţinute de elevii unei clase la teza la matematică sunt date ı̂n tabelul de mai jos:

Nota 5 6 7 8 9 10
Nr. elevi 3 4 7 5 3 3

Media notelor obţinute de elevii clasei la această teză este:


a) 7,5; b) 7,4; c) 7,2; d) 7.

6. Temperatura minimă ı̂n dimineaţa zilei de joi a fost −4 C, iar temperatura minimă ı̂n
dimineaţa zilei de vineri a fost +2◦ C. Mihai afirmă vineri: ,,Temperatura minimă de
astăzi a fost cu 6◦ C mai mare decât temperatura minimă de ieri”. Cum este afirmaţia lui
Mihai?
a) adevărat;
b) fals;
SUBIECTUL al II-lea
Încercuies, te litera corespunzătoare răspunsului corect.
1. Fie punctele A, B, C, D coliniare, ı̂n această ordine, astfel ı̂ncât BC = 2AB şi CD = 3AB.
Care dintre următoarele afirmaţii este adevărată?
a) C este simetricul lui A faţă de B;
b) D este simetricul lui B faţă de C;
c) B este mijlocul segmentului AC;
d) C este mijlocul segmentului AD.
2. Într-un trapez, suma lungimilor bazelor este de 30 cm, iar lungimea ı̂nălţimii reprezintă
60% din lungimea liniei mijlocii. Aria trapezului este:
1
Lect. univ. dr., Universitatea din Pites, ti, adrianturcanu85@yahoo.com
PROBLEME DE MATEMATICĂ PENTRU EXAMENE 45

a) 270 cm2 ; b) 135 cm2 ; c) 540 cm2 ; d) 39 cm2 .


3. Un loc de joacă de formă circulară cu raza de 12 m se ı̂mprejmuieşte cu două rânduri de
plasă de sârmă. Un sul de plasă de sârmă are lungimea de 25 metri liniari. Care este
numărul minim de suluri necesare pentru ı̂mprejmuire?
a) 3; b) 4; c) 6; d) 7.
4. În triunghiul dreptunghic ABC, ^A = 90◦ , AD ⊥ BC, D ∈ BC, BD = 18 cm şi DC = 8
cm. Aria triunghiului ABC este egală cu:
a) 19 cm2 ; b) 144 cm2 ; c) 156 cm2 ; d) 312 cm2 .
5. Apotema unei piramide patrulatere regulate cu latura bazei de 12 cm formează un unghi
de 60◦ cu planul bazei. Volumul piramidei este:
√ √ √ √
a) 864 3 cm3 ; b) 288 3 cm3 ; c) 144 3 cm3 ; d) 72 3 cm3 .
6. Un paralelipiped dreptunghic are dimensiunile 18 cm, 4 cm şi respectiv 3 cm. Care este
suma lungimilor laturilor unui cub care are volumul egal cu volumul paralelipedului?
√ √
a) 48; b) 72 6; c) 48 6; d) 72.

SUBIECTUL al III-lea
Scriet, i rezolvările complete.
1. Dintre cei 25 de elevi ai unei clase, 11 au participat la o competiţie de tenis de masă şi 16
la o competiţie de badminton. Ştiind că doi dintre elevi nu au paricipat la niciuna dintre
cele două competiţii, aflaţi:
a) Câţi elevi au participat la ambele competiţii?
b) Câţi elevi au participat doar la competiţia de tenis de masă?
2. Fie expresia E(x, y) = x2 + y 2 − xy, x, y ∈ R.
a) Calculaţi E(2, −3).
b) Determinaţi numărul real pozitiv a astfel ı̂ncât E(a, −a) = 48.
3. Fie funct, ia f : R → R, f (x) = 2x + a, a ∈ R.
a) Determinaţi a astfel ı̂ncât punctul A(−1, 3) să aparţină graficului funcţiei f .
b) Pentru a = 5, calculaţi suma S = f (1) + f (2) + ... + f (10).
4. În triunghiul ABC, M este mijlocul laturii BC şi aria triunghiului ABM este 7, 5 cm2 .
a) Calculaţi aria triunghiului ABC.
b) Ştiind că distanţa de la A la BC reprezintă 30% din lungimea laturii BC, determinaţi
lungimea laturii BC.
5. L şi l, lungimea şi respectiv lăţimea unui dreptunghi, sunt invers proporţionale cu 0, (1) şi
respectiv 0, 1(6).
a) Arătaţi că 2L = 3l.
b) Ştiind că perimetrul dreptunghiului este egal cu 120 cm, determinaţi aria acestuia.
6. Un tub de tablă de forma unui cilindru circular drept se confecţionează prin rularea unei
foi de tablă de formă pătrată cu perimetrul de 12 m, fără pierderi de material.
a) Arătaţi că ı̂nălţimea tubului este de 3 m.
b) Arătaţi că raza bazei tubului este mai mică de 50 cm.
46 PROBLEME DE MATEMATICĂ PENTRU EXAMENE

Testul 2

Costel Anghel 2 s, i Florea Badea 3

SUBIECTUL I
Încercuies, te litera corespunzătoare răspunsului corect.
1. Rezultatul calculului 7 · 10 − 100 : (2 · 5) este egal cu:
a) 40; b) 60; c) 30; d) 90.
2. Într-o zi, temperatura aerului ı̂nregistrată dimineat, a era de −7◦ C. În aceeas, i zi, tempera-
tura măsurată la prânz era de +12◦ C. Temperatura măsurată, ı̂n acea zi, la prânz era
mai mare decât cea măsurată dimineat, a cu:
a) 5◦ C; b) 8◦ C; c) 18◦ C; d) 19◦ C.
3. Dintre seturile de numere de mai jos, cel scris ı̂n ordine crescătoare este:
a) 5,12; 5,1(2); 5,(12); 5,122; c) 5,12; 5,(12); 5,122; 5,1(2);
b) 5,1(2); 5,122; 5,(12); 5,12; d) 5,122; 5,12; 5,(12); 5,1(2).
4. Suma dintre cel mai mare număr ı̂ntreg s, i cel mai mic număr natural din intervalul [−3, 15)
este:
a) 13; b) 15; c) 14; d) 9.
x+3
5. Dacă 16
= 85 , atunci x este egal cu:
a) 6; b) 7; c) 4; d) 11.
√ √ √
6. Media aritmetică a numerelor 9 2, 5 2, −29 2 este:
√ √ √ √
a) −5 2; b) 5 2; c) 8 2; d) −4 2.

SUBIECTUL al II-lea
Încercuies, te litera corespunzătoare răspunsului corect.
1. Punctele A s, i B sunt simetrice fat, ă de punctul C. Dacă AC = 7, 5 cm atunci distant, a
dintre punctele A s, i B este egală cu:
a) 14 cm; b) 1,5 dm; c) 20 cm; d) 1,45 dm.
2. Dacă două unghiuri adiacente sunt complementare atunci măsura unghiului format de
bisectoarele lor este egală cu:
a) 40◦ ; b) 38◦ ; c) 46◦ ; d) 45◦ .
3. Un triunghi are măsurile unghiurilor direct proport, ionale cu numerele 9, 5 s, i 6. Măsura
unghiului mai mare este egală cu:
a) 83◦ ; b) 80◦ ; c) 81◦ ; d) 72◦ .
4. Se consideră trapezul dreptunghic ABCD, m(^A) = m(^D) = 90◦ , AB||CD||M N , unde
M ∈ (AD) s, i N ∈ (BC). Diagonala (AC) intersectează dreapta M N ı̂n punctul P . Dacă
4CN P este echilateral s, i AD = 4 cm atunci aria trapezului este egală cu:
√ √ √ √
a) 8 3 cm2 ; b) 6 3 cm2 ; c) 8 2 cm2 ; d) 9 6 cm2 .

5. Coarda [AB] subı̂ntinde un arc de cerc de 120◦ ı̂n cercul C(O; 7 2). Paralela prin A la
OB intersectează a doua oară cercul ı̂n punctul C. Aria patrulaterului AOBC este egală
cu:
2
Profesor, Colegiul Nat, ional ,,Ion Minulescu”, Slatina, anghelcostel2012@yahoo.com
3
Profesor, Şcoala Gimnazială ,,Nicolae Coculescu”, Scornices, ti
PROBLEME DE MATEMATICĂ PENTRU EXAMENE 47

√ √
a) 49 cm2 ; b) 49 3 cm2 ; c) 49 2 cm2 ; d) 50 cm2 .
6. În cubul ABCDEF GH, punctele M s, i N reprezintă mijlocul muchiei

[CD] respectiv
9 6 2
centru fet, ei ADHE. Dacă aria triunghiului AM N este egală cu 2 cm atunci volumul
cubului este egal cu:
√ √
a) 216 cm3 ; b) 81 3 cm3 ; c) 343 cm3 ; d) 162 6 cm3 .

SUBIECTUL al III-lea
Scriet, i rezolvările complete.
1. După o reducere cu 8% din pret, , un televizor costă 1472 lei.
a) Aflat, i pret, ul televizorului ı̂nainte de reducere.
b) Aflat, i cel mai mic număr natural p, astfel ı̂ncât, dacă reducerea de pret, ar fi fost cu
p%, atunci pret, ul televizorului ar fi scăzut
 50x sub 1000 lei.
1+x 1+2x
2. Se consideră expresia E(x) = 2x − 5x · x+3 , x 6= 0 s, i x 6= −3.
a) Arătat, i că E(10) ∈ N.
b) Demonstrat, i că E(x) este constantă, oricare ar fi x ∈ R \ {0; −3}.
3. Se consideră funct, iile f : R → R, f (x) = 3x − 2 s, i g : R → R, g(x) = −2x + 3.
a) Determinat, i coordonatele punctului de intersect, ie dintre graficele celor două funct, ii.
b) Calculat, i f (1) + f (2) + ... + f (100) + 2 · [g(1) + g(2) + ... + g(100)].
4. În triunghiul M N P , M P = 12 cm, P N = 9 cm, m(^M P N ) = 60◦ s, i [M A] ı̂nălt, ime,
A ∈ (N P ). Se construies, te dreptunghiul AM BN .
a) Calculat, i perimetrul dreptunghiului AM BN .
b) Calculat, i sinusul unghiului ^BP M .
5. În paralelipipedul dreptunghic ABCDM N P Q, AB = 15 cm, BC = 10 cm s, i AM = 12
cm.
a) Aflat, i volumul paralelipipedului.
b) Determinat, i pozit, ia punctului S pe muchia (CP ) astfel ı̂ncât perimetrul triunghiului
BSQ să fie minim.
6. Se consideră cubul ABCDEF GH s, i punctele M ∈ (AB), N ∈ (BC), P ∈ (F G), Q ∈
(EF ) astfel ı̂ncât AM = CN = EQ = GP = 13 · AB. Ştiind că aria patrulaterului M N P Q

este egală cu 54 2 cm2 se cer:
a) Aria sect, iunii diagonale a cubului.
b) Distant, a de la punctul A la planul (M N P ).

Testul 3

Ana Maria Iordache 4

SUBIECTUL I
Încercuies, te litera corespunzătoare răspunsului corect.
È √
−2
1. Dacă n ∈ Z s, i 43 : 9 − 0, 0025 ∈ (n − 1, n), atunci numărul n este:
1
a) 1; b) 5; c) 0; d) 5
.
2. Se consideră un pătrat cu latura de 60 cm. Atunci 5% din dublul ariei pătratului este:

4
Profesor, Liceul Tehnologic ,,Virgil Madgearu”, Ros, iorii de Vede, anamariayord@yahoo.com
48 PROBLEME DE MATEMATICĂ PENTRU EXAMENE

a) 36; b) 7200; c) 360; d) 72 C.


3. Două unghiuri exterioare ale unui triunghi sunt direct proport, ionale cu 2 s, i 3, iar suma
lor este 270◦ . Atunci triunghiul este:
a) isoscel; b) echilateral; c) dreptunghic; d) obtuzunghic.
4. Perimetrul unui hexagon regulat cu latura de 10 cm este:
a) 30 cm; b) 60 cm; c) 106 cm; d) 600 cm.
5. În tabelul de mai jos sunt precizate valorile temperaturilor ı̂nregistrate la ora 8 A.M. ı̂ntr-o
săptămână a lunii februarie.
Ziua I II III IV V VI VII
Punctaj 1 -2 -4 -3 1 1 0
Valoarea absolută a celei mai mici temperaturi ı̂nregistrată este:
a) -4; b) 4; c) 0; d) 2.

6. A doua zecimală a numărului 1, 6 este:
a) 6; b) 2; c) 1; d) 4.

SUBIECTUL al II-lea
Încercuies, te litera corespunzătoare răspunsului corect.
1. Un ornament de brad are forma unei piramide triunghiulare regulate. Numărul vârfurilor
piramidei este:
a) 3; b) 4; c) 5; d) 1.
2. În figura alăturată dreptele M N s, i P Q M N
sunt paralele. Măsura ^M N Q = 40◦ iar
măsura ^M P Q = 80◦ . Atunci măsura
^M ON este:
a) 60◦ ; c) 90◦ ;

b) 120 ; d) 30◦ . Q P

3. Dreptunghiul ABCD reprezintă o zonă cu flori albastre având perimetrul egal cu 14


m, ı̂n care lăt, imea reprezintă 40% din lungimea dreptunghiului ABCD. Atunci aria
dreptunghiului este egală cu:
a) 10 cm2 ; b) 5 cm2 ; c) 7 cm2 ; d) 14 cm2 .
4. Se consideră cercul de centru O s, i rază 25 cm. Fie A, B două puncte situate pe cerc astfel
ı̂ncât AB = 30 cm. Atunci distant, a de la centrul cercului la coarda AB este:
a) 20 cm; b) 400 cm; c) 40 cm; d) 10 cm.
5. Rezultatul calculului cos 30◦ + sin 30◦ − sin 60◦ − cos 60◦ este:
√ √
3 1 3+1
a) 0; b) 2
; c) 2
; d) 2
.
6. În triunghiul ABC, dreptunghic ı̂n A, ı̂nălt, imea AD, D ∈ (BC) este egală cu 12 cm. Dacă
DB 9
DC
= 16 , atunci aria triunghiului ABC este:
a) 300 cm2 ; b) 60 cm2 ; c) 25 cm2 ; d) 150 cm2 .

SUBIECTUL al III-lea
Scriet, i rezolvările complete.
1. Numerele reale x s, i y au raportul 5 s, i diferent, a 16.
PROBLEME DE MATEMATICĂ PENTRU EXAMENE 49

a) Determinat, i media aritmetică a numerelor x s, i y.


b) Dacă x s, i y sunt catetele €unui triunghi dreptunghic
Š aflat, i aria acestuia.
x+1 1 6 x+35
2. Se consideră expresia E(x) = x2 +x−6 + 2−x + x+3 : x+3 .
a) Aflat, i mult, imea V pentru care este definită expresia E(x).
b) Aducet, i expresia la forma cea mai simplă.
3. Se consideră funct, ia f : R → R, f (x) = 4x − 3f (1).
a) Determinat, i f (x).
b) Rezolvat, i ı̂n Z inecuat, ia: 2f (x) + 3f (2x) ≥ −20.
4. Un pătrat s, i un dreptunghi au acelas, i perimetru. Dacă lungimea dreptunghiului este de
10 cm s, i aria pătratului este de 64 cm2 , atunci:
a) Calculat, i latura pătratului.
b) Calculat, i aria dreptunghiului.
5. În paralelogramul ABCD se cunosc AD = 10 cm, AB = 14 cm s, i m(^B) = 120◦ .
a) Aflat, i lungimea diagonalei AC.
b) Dacă E este punctul de intersect, ie al bisectoarei unghiului ^ADC cu AC, calculat, i
EC 2 − AE 2 .
6. Un teren de joacă are forma unui pătrat ABCD cu latura de 20 cm. În mijlocul M al
laturii AB este montat un stâlp de iluminat M P cu lungimea de 12 cm.
a) Aflat, i distant, a de la P la latura terenului.
b) Determinat, i sin ^((P AB), (P DC)).

Testul 4

Dragos, Drăgus, in 5

SUBIECTUL I
Încercuies, te litera corespunzătoare răspunsului corect.
1. Rezultatul calculului 1, 2 − 0, 2 · 10 este:
1
a) 0,8; b) 10; c) -0,8; d) 2
.
2. Se consideră numărul n = |x − 2y + 1|. Valoarea lui n pentru x = −2 s, i y = 3 este:
a) 7; b) -7; c) 3; d) -3.
3. Un televizor costă 1990 lei. După o ieftinire cu 10% pret, ul televizorului devine:
a) 1900; b) 1791; c) 1791,2; d) 199.
4. Un robinet umple un bazin ı̂n 9 ore. Timpul ı̂n care umplu bazinul 3 robinete cu acelas, i
debit ca s, i primul este:
a) 27; b) 6; c) 3; d) 1.
√ È √
5. Fie numerele a = | 2 − 2| s, i b = (2 + 2)2 . Suma dintre media aritmetică s, i media
geometrică a numerelor a s, i b este:
√ √
a) 4; b) 2 + 2; c) 2+2 2 ; d) 8.
√ √
6. Fie intervalul A = (−3; 2] s, i mult, imea B = {− 3; 3; 1, 7; 32 }. Cel mai mare element din
A ∩ B este:

5
Profesor, Liceul Tehnologic ,,Regele Mihai I”, Curtea de Arges, , dragusin dragos@yahoo.com
50 PROBLEME DE MATEMATICĂ PENTRU EXAMENE

√ √ 3
a) 3; b) − 3; c) 1,7; d) 2
.

SUBIECTUL al II-lea
Încercuies, te litera corespunzătoare răspunsului corect.
1. Se consideră două drepte concurente AB s, i CD, cu AB ∩ CD = {O}, OB = OA = 4 cm,
OD = OC = 5 cm s, i m(^AOD) = 45◦ . Patrulaterul ABCD este:
a) pătrat; c) dreptunghi;
b) romb; d) paralelogram.
2. Se consideră unghiul ^AOC adiacent cu unghiul ^EOC, [OB s, i [OD bisectoarele unghiu-
rilor ^AOC, respectiv ^EOC. Ştiind că m(^AOD) = 120◦ s, i m(^BOE) = 90◦ , măsura
unghiului ^BOD este:
a) 100◦ ; b) 120◦ ; c) 70◦ ; d) 30◦ .
3. Se consideră punctele A s, i B pe un cerc de centru O astfel ı̂ncât m(^AOB) = 80◦ , iar
_ _
punctul C ∈AB (arcul mare AB). Măsura unghiului ^ACB este:
a) 80◦ ; b) 40◦ ; c) 160◦ ; d) 180◦ .
4. Se consideră punctele necoliniare A, B s, i C astfel ı̂ncât m(^BAC) = 90◦ , AB = 4 cm s, i
AC = 3 cm. Distant, a de la A la BC este:
a) 2,4 cm; b) 5 cm; c) 12 cm; d) 6 cm.
5. Perimetrul unui romb cu diagonalele egale cu 12 cm s, i 16 cm este:
a) 40 cm; b) 40 dm; c) 20 cm; d) 30 cm.
6. Cantitatea de apă care se revarsă dintr-un recipient la scufundarea ı̂n acesta a unei pietre
cubice cu dimensiunea de 3 cm este:
a) 9 ml; b) 27 ml; c) 9 l; d) 27 l.

SUBIECTUL al III-lea
Scriet, i rezolvările complete.
1. La un test de admitere cu 30 de ı̂ntrebări se acordă 4 puncte pentru un răspuns corect s, i
se scad 2 puncte pentru un răspuns gres, it. Pentru a fi admis, un elev trebuie să aibă un
punctaj de minim 100 de puncte.
a) Dacă Ionel a gres, it la 4 ı̂ntrebări, este declarat admis? Justificat, i răspunsul.
b) Care este numărul minim€ de răspunsuri corecte Š 2 pentru a fi admis?
2. Se consideră expresia E(x) = x−2 − 2+x − x2 −4 · x −4x+4
3 2 10
x
, unde x ∈ R \ {−2, 0, 2}.
x−2
a) Arătat, i că E(x) = x+2 .
b) Determinat, i valorile ı̂ntregi ale lui x pentru care E(x) ∈ Z.
3. Fie funct, ia f : R → R, f (x) = −x + 2.
a) Determinat, i aria triunghiului format de graficul funct, iei cu axele de coordonate Ox
s, i Oy.
b) Aflat, i punctul de pe graficul funct, iei care are abscisa egală cu dublul ordonatei.
4. Trapezul isoscel ABCD cu AB||CD are [AB] ≡ [DC] ≡ [BC], AD = 18 cm s, i m(^ACD) =
30◦ .
a) Arătat, i că AC ⊥ BC.
b) Aflat, i aria trapezului ABCD.
5. Fie triunghiul ABC dreptunghic ı̂n A, cu AB = 8 cm s, i AC = 6 cm. Se consideră punctul
N ∈ (AB), AN = 3 cm s, i M ∈ (AC) astfel ı̂ncât ^M N A ≡ ^BCA.
PROBLEME DE MATEMATICĂ PENTRU EXAMENE 51

a) Arătat, i că distant, a de la N la BC este egală cu AN .


b) Aflat, i aria patrulaterului BN M C.
6. O prismă dreaptă ABCDA0 B 0 C 0 D0 are bazele pătrate cu AB = 12 cm s, i AB 0 = 13 cm.
a) Aflat, i tangenta unghiului format de dreapta B 0 O cu planul (ABC) unde O este
punctul de intersect, ie al diagonalelor prismei.
b) Aflat, i volumul piramidei OABCD.

Testul 5

Elena Tit, escu 6

SUBIECTUL I
Încercuies, te litera corespunzătoare răspunsului corect.
1. Rezultatul calculului 15 − 5 : (1 + 4) este:
a) 2; b) 14; c) 10; d) 5.
2. O păpus, ă costă 25 lei. După o reducere cu 20%, pret, ul păpus, ii va fi:
a) 20 lei; b) 30 lei; c) 15 lei; d) 25 lei.
3. În tabelul de mai jos sunt prezentate temperaturile medii ale celor trei luni de iarnă:

Decembrie Ianuarie Februarie


-8 -12 3

Conform tabelului diferent, a dintre temperatura maximă s, i cea minimă este de:
a) 9◦ C; b) 11◦ C; c) −15◦ C; d) 15◦ C.
4. Media aritmetică a numerelor 15, 10 s, i 17 este:
a) 42; b) 14; c) 16; d) 14.
5. Inversul numărului a = 0, 2 + 0, (3) este egal cu:
8 5 8 15
a) 5
; b) 8
; c) 15
; d) 8
.

SUBIECTUL al II-lea
Încercuies, te litera corespunzătoare răspunsului corect.
1. În figura alăturată, măsura unghiul ^AOB M
este egală cu 50◦ , M O ⊥ OA, iar punctele
B, O, N sunt coliniare. Măsura unghiului
^M ON este: B

a) 140 ;
A
b) 120◦ ;
c) 110◦ ;
d) 130◦ .
N
◦ 0
2. Un unghi are măsura egală cu 48 40 . Măsura complementului său este de:

6
Profesor, Şcoala Gimnazială ,,Naum Râmniceanu”, Corbi, amalia elena19@yahoo.com
52 PROBLEME DE MATEMATICĂ PENTRU EXAMENE

a) 39◦ 400 ; b) 41◦ 200 ; c) 131◦ 300 ; d) 131◦ 200 .


3. În figura alăturată dreptele a s, i b sunt pa-
ralele, iar c este o secantă. Valoarea lui x x + 60o
este: a

a) 20 ;
b) 30◦ ; 4 x - 30o
c) 40◦ ; b
d) 50◦ .

4. În figura alăturată√este reprezentată o placă de faiant, ă ı̂n formă de pătrat ABCD cu
diagonala AC = 6 2 cm. Aria plăcii de faiant, ă este:
a) 36 cm2 ; b) 72 cm2 ; c) 12 cm2 ; d) 24 cm2 .
5. Un cerc are raza cu lungimea de 10 cm. Raportul dintre lungimea cercului s, i diametrul
acestuia este de:
a) 2 π; b) 2; c) π; d) 1.
6. În figura alăturată este reprezentată o M' P'
prismă triunghiulară regulată dreaptă
M N P M 0 N 0 P 0 cu M N = 8 cm s, i M M 0 =
4 cm. Măsura unghiului dintre planele N'
(M 0 N P ) s, i (M N P ) este:
P
a) 30◦ ; M
b) 45◦ ;
c) 60◦ ;
d) 90◦ . N

SUBIECTUL al III-lea
Scriet, i rezolvările complete.
1. Mama s, i fiica au ı̂mpreună 55 de ani, iar raportul vârstelor lor este de 92 .
a) Aflat, i vârsta fiicei.
b) Peste cât, i ani vârsta mamei va €fi dublul Š
vârstei fiicei?
3x2
2. Se consideră expresia E(x) = x−1 : 1 − 1−x2 , unde x ∈ R \ {−1, − 12 , 12 , 1}.
2x+1

x+1
a) Arătat, i că E(x) = 2x−1 , ∀x ∈ R \ {−1, − 21 , 12 , 1}.
b) Rezolvat, i ecuat, ia E(x) = 2.
3. Fie funct, ia f : R → R, f (x) = ax + 2. Punctul A(2, 8) este situat pe graficul funct, iei f .
a) Arătat, i că a = 3.
b) Determinat, i punctul B de pe graficul funct, iei care are coordonatele egale.
4. În figura alăturată sunt reprezentate două A D E
pătrate, ABCD s, i CDEF având latura
egală cu 12 cm.
a) Determinat, i distant, a de la punctul C
la dreapta AF .
b) Calculat, i sin(^F AC).
B C F
PROBLEME DE MATEMATICĂ PENTRU EXAMENE 53

5. În figura alăturată este reprezentat triun- A


ghiul echilateral ABC, cu AB = 12 cm,
iar punctul D apart, ine prelungirii laturii
BC, astfel ı̂ncât CD = 6 cm.

a) Arătat, i că AD = 6 7 cm. B C D
b) Demonstrat, i că aria triunghiului
ACD este mai mică decât 32 cm2 .
6. În figura alăturată este reprezentat un cort S
ı̂n formă de piramidă patrulateră regu-
lată dreaptă SABCD, cu AB = 4 m s, i
SO = 1, 5 m.
a) Fet, ele laterale ale cortului sunt vop-
site cu albastru. Aflat, i suprafat, a vop-
D C
sită.
b) Aflat, i sinusul unghiului format de o
fat, ă laterală cu planul bazei.
A B
54 PROBLEME DE MATEMATICĂ PENTRU EXAMENE

Teste pentru examenul de Bacalaureat, specializarea S, tiint, e ale naturii

Testul 1
Marius Macarie 1

SUBIECTUL I
1. Să se determine rat, ia progresiei geometrice (bn )n∈N? ı̂n care
§
b2 − b1 = 2
b3 − b1 = −6

2. Să se determine m ∈ R astfel ı̂ncât vârful parabolei asociate funct, iei f : R → R,


f (x) = x2 − 2mx + m√− 1 se află√ pe dreapta √ de ecuat, ie y = 2x − 1.
3. Să se rezolve ecuat, ia 4 − x + 8 + x = 2 3.
4. Să se determine numărul submult, imilor mult, imii {1, 2, 3, 4, 5} care au un număr impar de
elemente. →
− → − − →
− →

5. Să se determine m ∈ R astfel ı̂ncât vectorii →

u = (m − 1) i − j s, i → v = −4 i + (m + 2) j
sunt coliniari.
6. Se consideră triunghiul ABC cu AB = 4, AC = 5, BC = 7. Să se calculeze sin A.
SUBIECTUL al II-lea
1. Se consideră sistemul de ecuat, ii

x + ay − 2z = 8 1 a −2
( !
2x + ay + 3z = b s, i matricea A(a) = 2 a 3 .
3x + ay + 4z = −2 3 a 4

a) Să se arate că det A(1) + det A(2) + . . . + det A(10) = 220.
b) Pentru a = 1 s, i b = −1, să se rezolve sistemul de ecuat, ii.
c) Să se determine a, b ∈ R astfel ı̂ncât sistemul de ecuat, ii să fie nu aibe solut, ii.
2. Fie polinomul f = (X − 1)2020 + (X − 2)2019 + mX 2 + X + 1 cu rădăcinile xk ∈ C,
k = 1, 2020 s, i m ∈ R.
a) Să se determine m ∈ R astfel ı̂ncât f este divizibil cu g = X − 1.
b) Să se arate că x21 + x22 + . . . + x22020 = 6057.
c) Să se determine restul ı̂mpărt, irii lui f la polinomul h = X 2 − 3X + 2.
SUBIECTUL al III-lea
e2x − x + 1
1. Se consideră funct, ia f : R → R, f (x) = .
e2x
f (x) − f (0)
a) Calculat, i lim .
x→0 x
b) Să se demonstreze că tangenta la graficul funct, iei f ı̂n punctul A 32 , f 3

2
este
paralelă cu asimptota spre +∞ la graficul funct, iei f .
c) Să se determine intervalele de convexitate s, i concavitate ale funct, iei f .
ln x
2. Se consideră funct, ia f : (0, ∞) → R, f (x) = 2 .
x
Z 3
f (x) 1
a) Să se arate că dx = .
2 ln x 6 √
b) Să se arate că orice primitivă a funct, iei f este convexă pe (0, e].
1
Lect. univ. dr., Universitatea din Pites, ti, macariem@yahoo.com
PROBLEME DE MATEMATICĂ PENTRU EXAMENE 55

c) Să se calculeze volumul corpului obt, inut prin rotat, ia ı̂n jurul axei Ox a graficului
funct, iei g : [1, e] → R, g(x) = f (x).

Testul 2
Monica Dumitrache 2

SUBIECTUL I
1. Arătat, i că log2 5 · log5 8 = 3.
2. Se consideră funct, ia f : R → R, f (x) = 2x − 4. Determinat, i x ∈ R pentru care
f (x)
≥ 0.
10 + f (x)
3. Rezolvat, i ı̂n mult, imea numerelor reale ecuat, ia 3x + 8 · 3−x = 9.
√ 8
4. Determinat, i al cincilea termen al dezvoltării ( 3 x + 3x) .
5. Se consideră dreapta d de ecuat, ie 2x − 4y + 1 = 0. Scriet, i ecuat, ia dreptei care trece prin
punctul A (4, −1) s, i este perpendiculară pe dreapta d.
2 1 √
6. În triunghiul ABC avem sin A = √ , sin B = √ s, i BC = 2 3. Calculat, i AC.
13 3
SUBIECTUL al II-lea
1 1 1
1. Fie determinantul D (a, b) = a b 1 , unde a, b ∈ Z.
a2 b 2 1
a) Calculat, i D(2, 3).
b) Determinat, i m ∈ Z s, tiind că D (m, 3) = −6.
c) Demonstrat, i că numărul D(a, b) este divizibil cu 2, oricare ar fi a, b ∈ Z.

2. Pe mult, imea numerelor reale se defines, te legea de compozit, ie asociativă s, i cu element


x y 3
neutru x ⊗ y = xy − − + .
2 2 4
a) Arătat, i că x ⊗ y = x − 21 y − 12 + 12 , pentru orice x, y ∈ R.
 
3
b) Determinat, i simetricul elementului x = , ı̂n raport cu legea de compozit, ie ,,⊗”.
2
1 2 3 19
c) Rezolvat, i ı̂n R ecuat, ia logx 2 = ⊗ ⊗ ⊗ · · · ⊗ .
2 3 4 20
SUBIECTUL al III-lea
ex
1. Se consideră funct, ia f : (0, +∞) → R, f (x) = x
.
a) Arătat, i că f 0 (e) = ee−1 − ee−2 .
b) Determinat, i ecuat, ia tangentei la graficul funct, iei f ı̂n punctul de abscisă x = 1, situat
pe graficul funct, iei f .
c) Demonstrat, i că ex − ex ≥ 0, pentru orice x ∈ (0, +∞).
1
2. Se consideră funct, ia f : [1, 3] → R, f (x) = x2 + 2 .
x
a) Calculat, i volumul corpului obt, inut prin rotirea graficului funct, iei g : [1, 3] → R,
g (x) = f (x) − x12 , ı̂n jurul axei Ox.
R3
b) Calculat, i xf (x) ln xdx.
1
c) Arătat, i că orice primitivă a lui f este funct, ie convexă.
2
Profesor, Colegiul Economic ,,Ion Ghica”, Târgovis, te, dumitrache m0nica@yahoo.com
56 PROBLEME DE MATEMATICĂ PENTRU EXAMENE

Testul 3
Mihai Florea Dumitrescu 3

SUBIECTUL I

1. Se consideră progresia aritmetică (an )n≥1 cu rat, ia pozitivă. S, tiind că a3 = 7 s, i produsul
primilor cinci termeni ai progresiei este egal cu 3640, aflat, i primul termen al progresiei.
2. Să se determine numărul real m, astfel ı̂ncât axa Ox este tangentă parabolei y = x2 +mx+4.
3. Rezolvat, i ı̂n mult, imea numerelor reale ecuat, ia log2 x = log4 (x + 2).
4. Să se determine probabilitatea ca, alegând un număr din mult, imea numerelor naturale de
două cifre, acesta să aibă cifra zecilor divizibilă cu 4.

− → − − →
− → −
5. Aflat, i numărul real a, pentru care vectorii →

u = a i + j s, i →
v = i − j sunt perpendiculari.
1
6. Un triunghi ABC are BC = 2R, unde R este raza cercului circumscris 4ABC , sin B =
√ 2
s, i aria egală cu 8 3. Aflat, i R.

SUBIECTUL al II-lea
 ‹
5 1
1. Se consideră matricea A = .
−10 −2
a) Calculat, i det (A2 ).
b) Aflat, i perechile de numere ı̂ntregi (a, b), astfel ı̂ncât are loc egalitatea (I2 + aA) ·
(I2 + bA) = I2 + A.
c) Aflat, i numărul natural n pentru care are loc egalitatea A + A2 + A3 + ... + A2020 =
3n − 1
A.
2
2. Fie polinomul f = X 3 + (1 − 2a) X 2 + (a2 − 2a) X + a2 , unde a este număr real nenul.
a) Aflat, i restul ı̂mpărt, irii polinomului f la polinomul X − 1.
b) Pentru a ∈ Z aflat, i rădăcinile polinomului f .
c) Determinat, i a ∈ R, pentru care are are loc egalitatea
x1 + x2 + 1 x1 + x3 + 1 x3 + x2 + 1
+ + = −1.
x3 x2 x1

SUBIECTUL al III-lea
1. Se consideră funct, ia f : R \ {1} → R, f (x) = ln |x − 1|.
a) Determinat, i ecuat, ia asimptotei verticale la graficul funct, iei f .
b) Arătat, i că funct, ia f este strict descrescătoare pe intervalul (−∞, 1).
c) Arătat, i că funct, ia f este concavă pe intervalul (1, +∞).
2. Se consideră funct, ia fn : R → R, fn (x) = xn · sin x, n ∈ N.

a) Calculat, i π2 f0 (x) dx.
R 3π
b) Calculat, i π2 f1 (x) dx.
6
c) Calculat
 π ,i volumul corpului obt, inut prin rotat, ia ı̂n jurul axei Ox a graficului funct, iei
g : 0, 4 → R, g(x) = f1 (x).

3
Profesor, Liceul ,,S, tefan Diaconescu”, Potcoava, florin14mihai@yahoo.com
PROBLEME DE MATEMATICĂ PENTRU EXAMENE 57

Testul 4

Adina-Florina Militaru 4

SUBIECTUL I
1. Să se calculeze (1 + 2i)2 + (1 − 2i)2 .
2. Să se rezolve ı̂n mult, imea numerelor reale ecuat, ia

log2 (4x − 2) = log2 (3 · 2x − 4).

3. Să se calculeze suma f (1) + f (2) + . . . + f (1000), s, tiind că f : R → R, f (x) = 4x + 1.


4. Să se determine probabilitatea ca alegând un număr din mult, imea {51, 52, . . . , 100} acesta
să fie divizibil cu 7.
5. Aflat, i distant, a de la
 punctul A(1, 2) la dreapta de ecuat, ie 3x − 4y + 2 = 0.
π 1
6. Ştiind că α ∈ 2 , π , sin α = 3 , să se calculeze cos 2α.
SUBIECTUL al II-lea
1 −1 1
!
1. Fie matricea A = 0 1 2 ∈ M3 (R).
m −1 2
a) Determinat, i m ∈ R astfel ı̂ncât det A = 2.
b) Pentru m = 0, să se determine x ∈ R s, tiind că det(A + xI3 ) = 0.
1
!
c) Pentru m = 1 să se determine X ∈ M3,1 (R) astfel ı̂ncât A · X = 1 .
1
2. Considerăm mult, imea G = (3, +∞) s, i legea de compozit, ie

x ∗ y = xy − 3x − 3y + 12, ∀x, y ∈ G.

a) Arătat, i că x ∗ y = (x
¨ − 3)(y − 3) + 3, ∀x, y ∈ G.
x ∗ y = 15
b) Rezolvat, i sistemul
x ∗ 6 = y.
c) Să se arate că legea este asociativă.
SUBIECTUL al III-lea
2x2 −3x−1
1. Se consideră f : R \ {1} → R, f (x) = (x−1)2
.
x−3
a) Arătat, i că pentru x 6= 1 avem f (x) = 2 + (x−1) 2.
0
b) Determinat, i f (x) s, i arătat, i că funct, ia f este descrescătoare pe (−∞, 1).
c) Determinat, i punctul de pe graficul funct, iei ı̂n care tangenta la grafic are panta egală
cu −5.
2. Fie f : (0, ∞) → R, f (x) = x ln x.
a) Să se determine a, b ∈ R astfel ı̂ncât funct, ia F : (0, ∞) → R, F (x) = ax2 ln x + bx2
să fie o primitivă a lui f .
b) Să se calculeze aria suprafet, ei cuprinsă ı̂ntre graficul funct, iei f , axa Ox s, i dreptele
de ecuat, ii x = 1, x = e.
Rx
f (t)dt
1
c) Să se calculeze lim x3
.
x→∞

4
Profesor, Grup Şcolar Construct, ii de Mas, ini, Colibas, i, popescuadina73@yahoo.com
58 PROBLEME DE MATEMATICĂ PENTRU EXAMENE

Testul 5
Ioana Diana Bucs, an 5

SUBIECTUL I
1. Se consideră numărul complex z = 1 − 2i. Calculat, i z 2 .
2. Să se calculeze suma primilor s, ase termeni ai unei progresii aritmetice (an )n≥1 , s, tiind că
a1 = 1 s, i a2 = 4.
3. Se consideră funct, ia f : R → R, f (x) = −3x + 8. Să se determine punctul care apart, ine
graficului funct, iei f s, i are abscisa egală cu ordonata.
4. Să se determine câte numere naturale de câte trei cifre distincte se pot forma cu elementele
mult, imii {5, 6, 7, 8}
5. În reperul cartezian xOy se consideră punctele A(3, −4) s, i B(4, −3). Să se determine
coordonatele simetricului punctului A fat, ă de punctul B.
6. Să se calculeze lungimea razei cercului circumscris triunghiului ABC, s, tiind că BC = 25
s, i m(^A) = 30◦ .
SUBIECTUL al II-lea
 ‹
2a + 7 3
1. Se consideră matricea A(a) = , unde a ∈ R.
3−a 2
a) Să se calculeze det A(−2).
b) Determinat, i numărul real a s, tiind că det A(a) = −9.
c) Determinat, i inversa matricei A(0).
2. Pe mult, imea numerelor reale se defines, te legea de compozit, ie asociativă x ◦ y = 3xy −
9x − 9y + 30.
a) Arătat, i că x ◦√y = √
3(x − 3)(y√− 3) + 3, pentru orice numere reale x s, i y.
b) Calculat, i 1 ◦ 2 ◦ 3 ◦ . . . ◦ 2020.
c) Rezolvat, i ı̂n mult, imea numerelor reale ecuat, ia x ◦ x ◦ x = x.
SUBIECTUL al III-lea
ex
1. Se consideră funct, ia f : R? → R, f (x) = .
x2
a) Să se calculeze f 0 (x), x ∈ R? .
b) Să se demonstreze √
că funct
√,
ia f este descrescătoare pe (0, 2].
3 2
c) Să se arate că 2e ≤ 3e .
1
2. Se consideră funct, ia f : (2, ∞) → R, f (x) = 2 .
x −4
R5
a) Arătat, i că (x − 2) · f (x)dx = ln 65 .
3
R5
b) Calculat, i (x2 + x − 6) · f (x)dx.
3
c) Calculat, i aria suprafet, ei delimitate de graficul funct, iei f , axa Ox s, i dreptele de ecuat, ii
x = 3 s, i x = 4.

5
Profesor, S, coala Gimnazială Dobres, ti, ioana.diana117@yahoo.com
PROBLEME DE MATEMATICĂ PENTRU EXAMENE 59

Teste pentru examenul de Bacalaureat, specializarea


Matematică-Informatică

TESTUL 1

Marius Macarie 1

SUBIECTUL I (30p)
1. Să se calculeze suma primilor 8 termeni ai progresiei geometrice (bn )n≥1 , s, tiind că b2 = 32,
b3 = 16. (5p)
2. Se consideră funct, ia bijectivă f : (0, ∞) → (4, ∞), f (x) = x2 + 4. Să se calculeze f −1 (5),
unde f −1 este inversa lui f . (5p)
5
3. Să se rezolve ı̂n mult, imea numerelor reale ecuat, ia 21−2x −
+ 2 = 0. (5p)
2x
4. Să se calculeze probabilitatea ca, alegând o submult, ime nevidă a mult, imii A =
{1, 2, 3, 4, 5, 6, 7, 8}, aceasta să aibă toate elementele pare. (5p)
5. Fie A(2, 1), B(−2, 4), C(0, 2). Să se determine ecuat, ia dreptei ce trece prin A s, i este
paralelă cu mediatoarea segmentului BC. (5p)
 ‹
3π 5
6. Ştiind că a ∈ π, s, i cos a = − , să se calculeze tg 2a. (5p)
2 13
SUBIECTUL al II-lea (30p)
 „ Ž
 x + 2y − 2z = 0 1 2 −2
1. Se consideră sistemul mx − 4y + z = 0 , m ∈ R, s, i matricea A(m) = m −4 1 .
2x − 2y − z = 0 2 −2 −1

a) Să se determine valorile reale ale lui m pentru care sistemul admite numai solut, ia banală.
(5p)
„ Ž
−3 2 0
b) Să se determine X ∈ M3 (R) astfel ı̂ncât A(0) · X = 6 0 2 . (5p)
0 −6 2
5
c) Pentru m = 1, să se determine solut, ia sistemului (x0 , y0 , z0 ) pentru care x20 + 2y02 + z02 =
2
s, i x0 , y0 , z0 sunt numere reale negative. (5p)

5
2. Pe mult, imea G = (0, ∞), se consideră legea de compozit, ie x ? y = xlog3 y .
a) Arătat, i că 2 ? 243 = 2. (5p)
b) Demonstrat, i că legea ,,?” este asociativă. (5p)
c) Determinat, i x ∈ G care sunt egale cu simetricele lor ı̂n raport cu legea ,,?”. (5p)
SUBIECTUL al III-lea (30p)
x−2
1. Fie f : R → R, f (x) = √ .
x2 + 4
1
Lect. univ. dr., Universitatea din Pites, ti, macariem@yahoo.com
60 PROBLEME DE MATEMATICĂ PENTRU EXAMENE

2(x + 2)
a) Arătat, i că f 0 (x) = √ , x ∈ R. (5p)
(x2 + 4) x2 + 4
b) Calculat, i lim (f (x))x . (5p)
x→∞

c) Determinat, i imaginea funct, iei f . (5p)


Z 1
xn
2. Se consideră s, irul In = 2 + 5x + 6
dx, n ∈ N∗ .
0 x
a) Să se calculeze I1 . (5p)
1
b) Să se arate că In+2 + 5In+1 + 6In = , n ∈ N∗ . (5p)
n+1
c) Să se calculeze lim nIn . (5p)
n→∞

TESTUL 2
Roxana Maria Manea 2

SUBIECTUL I (30p)
1. Determinat, i rat, ia progresiei aritmetice (an )n≥1 , s, tiind că a1 = 9, a6 = 49. (5p)
2. Aflat, i coordonatele punctelor de intersect, ie ale graficului funct, iei f : R → R, cu axa
absciselor, unde f (x) = x2 − 2x − 3. (5p)
1
3. Rezolvat, i ı̂n mult, imea numerelor reale ecuat, ia 9x−4 =
. (5p)
3−2
4. Calculat, i probabilitatea ca, alegând un număr din mult, imea numerelor naturale de două
cifre, acesta să nu fie pătrat perfect. (5p)
√ √
5. În reperul cartezian xOy se consideră punctul P (1 − 3, 3 − √3). Determinat
√ , i ecuat, ia
dreptei ce trece prin punctul P s, i este paralelă cu dreapta de ecuat, ie 3 2x + 6y − 5 = 0. (5p)
6. Arătat, i că (sin x + 4 cos x)2 + (4 sin x − cos x)2 = 17, ∀ x ∈ R. (5p)
SUBIECTUL al II-lea (30p)
„ Ž
−p + 1 0 2p
1. Se consideră matricea A(p) = 0 1 0 , p ∈ R.
−p 0 2p + 1
a) Arătat, i că detA(−1) = 0. (5p)
b) Arătat, i că A(p2 ) · A(q 2 ) = A(p2 q 2 + p2 + q 2 ), pentru orice numere reale p s, i q. (5p)
c) Arătat, i că nu există numere reale p astfel ı̂ncât să aibă loc relat, ia (5p)

A(p2 ) · A(p2 ) · A(3) = A(0).

2. Se consideră polinomul f = (X 2 + 1)(X + 2) − p, p ∈ R.


a) Arătat, i că f (−1) = 2 − p, p ∈ R. (5p)
2
Profesor, Şcoala Gimnazială ,,Nicolae Crevedia”, Crevedia Mare, roxam86@yahoo.com
PROBLEME DE MATEMATICĂ PENTRU EXAMENE 61

b) Pentru p = 1, calculat, i x31 + x32 + x33 , unde x1 , x2 , x3 sunt rădăcinile polinomului f . (5p)
c) Pentru p = 2, aflat, i rădăcinile polinomului f . (5p)
SUBIECTUL al III-lea (30p)
√ √
1. Se consideră funct, ia f : R → R, f (x) = 3x − 3x2 + 1.
−3x √
a) Arătat, i că f 0 (x) = √ + 3, x ∈ R. (5p)
3x2 + 1
b) Aflat, i ecuat, ia asimptotei orizontale spre +∞ la graficul funct, iei f . (5p)
c) Arătat, i că funct, ia f este concavă pe R. (5p)
2. Se consideră funct, ia f : (0, ∞) → R, f (x) = ln x + ex .
Re√ √
a) Arătat, i că 1 e(f (x) − ex )dx = e. (5p)
b) Arătat, i că orice primitivă a funct, iei f este strict crescătoare pe intervalul (1, ∞). (5p)
c) Aflat, i aria suprafet, ei plane delimitate de graficul funct, iei g : (0, ∞) → R, g(x) = xf (x),
axa Ox s, i dreptele de ecuat, ii x = 1 s, i x = e. (5p)

TESTUL 3

Raluca Mihaela Georgescu 3

SUBIECTUL I (30p)
1. Să se afle |z12 + z2 |, s, tiind că z1 s, i z2 sunt rădăcinile complexe ale ecuat, iei z 2 + 4z + 5 = 0.
(5p)
2. Să se determine valorile ı̂ntregi ale lui a, s, tiind că (a + 2)x2 + (a − 1)x − a ≥ 0, pentru
orice x ∈ R. (5p)
x x
3. Să se rezolve ecuat, ia 3log2 4 − 4 · 3log2 2 + 3 = 0. (5p)
√ √
4. Să se determine termenul care cont, ine x4 din dezvoltarea binomului (2 3 x + x)10 .(5p)
5. Fie A(2, 5), B(−1, 2), C(1, 3). Să se determine lungimea ı̂nălt, imii din C ı̂n triunghiul
ABC. (5p)

6. În triunghiul ABC se cunosc AB = 5 2, BC = 5, m(^A) = 30◦ . Să se determine
lungimea laturii AC. (5p)
SUBIECTUL al II-lea (30p)
„ Ž
ln e2 ln e 0
1. Fie A ∈ M2 (R), A = 0 ln e ln e2 .
0 0 ln e
a) Să se verifice că detA = 2. (5p)
b) Să se determine B ∈ M2 (R), astfel ı̂ncât A + B = I3 . (5p)
c) Să se calculeze An , n ∈ N∗ . (5p)
3
Lect. univ. dr., Universitatea din Pites, ti, gemiral@yahoo.com
62 PROBLEME DE MATEMATICĂ PENTRU EXAMENE

√ √
√ Pe mult, imea numerelor reale este definită legea de compozit, ie x ? y = xy −
2. 2x − 2y +
2 + 2.
√ √ √
a) Să se arate că x ? y = (x − 2)(y − 2) + 2, x, y ∈ R. (5p)
b) Să se rezolve ı̂n R ecuat, ia x ? x ? x = x. (5p)
c) Ştiind că legea este asociativă, să se calculeze

2log4 1 ? 2log4 2 ? 2log4 3 ? . . . ? 2log4 2020 .

(5p)
SUBIECTUL al III-lea (30p)
x3 + 3x2 − 2x + 1
1. Fie f : D = R \ {−1, 1} → R, f (x) = .
x2 − 1
x4 − x2 − 8x + 2
a) Să se verifice că f 0 (x) = , ∀ x ∈ D. (5p)
(x2 − 1)2
b) Să se determine asimptotele funct, iei f . (5p)
c) Să se determine numărul rădăcinilor reale ale ecuat, iei f 0 (x) = 0. (5p)
|x2 − 4|
2. Fie f : R∗ → R, f (x) = .
x
R5
a) Să se calculeze 3
x2 f (x)dx. (5p)
b) Să se determine aria port, iunii plane determinate de graficul lui f , axa Ox s, i dreptele de
ecuat, ii x = 4 s, i x = 5. (5p)
c) Să se determine volumul corpului definit prin rotirea graficului funct, iei g : [1, 2] → R,
g(x) = xf (x), ı̂n jurul axei Ox. (5p)

TESTUL 4
Antonio Nuică 4

SUBIECTUL I (30p)
1. Să se determine suma primilor 100 de termeni ai progresiei geometrice (bn )n≥1 , pentru
care b1 = 1, b3 = 4. (5p)
2. Să se determine m ∈ R pentru care x2 − mx + 1 > 0, ∀ x ∈ R. (5p)
3. Să se rezolve ecuat, ia log3 (x2 − 1) = 1. (5p)
4. Să se afle probabilitatea ca alegând un număr impar de două cifre, suma cifrelor lui să fie
4. (5p)
5. Să se determine aria triunghiului ABC, cu AB = 5, BC = 6, CA = 7. (5p)
1
6. Să se rezolve ecuat, ia cos 4x = − . (5p)
2

4
Lect. univ. dr., Universitatea din Pites, ti, antonio 74nm@yahoo.com
PROBLEME DE MATEMATICĂ PENTRU EXAMENE 63

SUBIECTUL al II-lea (30p)


„ Ž
x y 1
1. Fie A(x, y) = 1 2 1 , x, y ∈ R, s, i punctele M (1, 2), N (2, 3).
2 3 1
a) Să se determine det A(1, 0). (5p)
b) Deducet, i că dreapta d de ecuat, ie det A(x, y) = 0 coincide cu M N . (5p)
c) Să se determine punctele P (x, y) cu x − y = 1 pentru care aria triunghiului M N P este 3.
(5p)
2. Se dă polinomul f = X 3 − 2X 2 + mX + n, m, n ∈ R, cu rădăcinile x1 , x2 , x3 ∈ C.
a) Să se determine x1 + x2 + x3 . (5p)
b) Să se determine m, n ∈ R pentru care X − 1 f s, i x21 + x22 + x23 = 8. (5p)
c) Pentru m s, i n determinat, i la b), să se rezolve ecuat, ia f (x) = 0. (5p)
SUBIECTUL al III-lea (30p)
x
1. Fie funct, ia f : R → R, f (x) = .
ex
a) Să se arate că graficul lui f admite asimptote orizontale. (5p)
b) Să se determine intervalele de monotonie ale funct, iei. (5p)
c) Să se determine intervalele de convexitate ale funct, iei. (5p)
x
2. Fie f : D = R \ {2, 3} → R, f (x) = 2 .
x − 5x + 6
a b
a) Să se determine a, b ∈ R, astfel ı̂ncât f (x) = + , ∀ x ∈ D. (5p)
x−2 x−3
b) Să se determine primitivele lui f pe (3, ∞). (5p)
c) Să se calculeze (5p)
Z n
1
lim f (t)dt.
n→∞ n 4
64 PROBLEME DE MATEMATICĂ PENTRU EXAMENE

Teste pentru admiterea la facultate

Testul 1

Raluca Mihaela Georgescu 1

SUBIECTUL I
Fie polinomul P (X) = X 4 − 5X 3 + mX 2 + 3X − 10, cu m ∈ R, care are rădăcinile
x1 , x2 , x3 , x4 ∈ C.
a) Să se determine m ∈ R, s, tiind că 2 + i este rădăcină a polinomului s, i apoi să se determine
x1 , x 2 , x 3 , x 4 .
b) Să se calculeze x41 + x42 + x43 + x44 pentru m = 7.
c) Pentru m = 7 să se determine polinomul de gradul 4 ale cărui rădăcini sunt inversele
rădăcinilor lui P .
SUBIECTUL al II-lea
Fie funct, ia f : R → R, f (x) = (x2 − 4x + 6 − a)ex , cu a ∈ R.
a) Să se determine a ∈ R s, tiind că 2 este punct de inflexiune.
b) Pentru a = 4 să se determine ecuat, iile tangentelor ı̂n punctele de inflexiune.
c) Pentru a = 4 să se calculeze aria subgraficului delimitat de graficul funct, iei f , axa Ox s, i
dreptele de ecuat, ii x = 0, x = 1.
SUBIECTUL al III-lea
În planul de coordonate xOy se consideră punctele A(1, 1), B(4, 2) s, i dreapta d de ecuat, ie
y = x + 4.
a) Să se determine coordonatele unui punct C ∈ d astfel ı̂ncât aria triunghiului ABC să fie
12.
b) Pentru punctul C determinat la punctul a) cu abscisa pozitivă, să se determine punctul
D ∈ d astfel ı̂ncât ABCD să fie trapez.
c) Să se determine coordonatele punctului de intersect, ie al diagonalelor trapezului găsit la
punctul b).

Testul 2

Maria-Crina Diaconu 2

SUBIECTUL I
Se consideră funct, ia f : R → R, f (x) = (3m − 1)x2 − 3(m + 1)x + m − 2, unde m este un
parametru real diferit de 31 .
a) Pentru m = 1 rezolvat, i ecuat, ia f (x) = 0.
b) Să se determine valorile lui m pentru care f (x) ≤ 3, ∀x ∈ R.
c) Să se determine valorile lui m pentru care ecuat, ia f (x) = 0 admite rădăcini reale s, i
negative.
1
Lect. univ. dr., Universitatea din Pites, ti, gemiral@yahoo.com
2
Asist. univ. dr., Universitatea din Pites, ti, crina.diaconu@upit.ro
PROBLEME DE MATEMATICĂ PENTRU EXAMENE 65

d) Dacă x1 , x2 ∈ C sunt solut, ii ale ecuat, iei f (x) = 0 s, i pentru n ∈ N∗ definim Sn = xn1 + xn2 ,
aflat, i m ∈ R astfel ı̂ncât S5 = −1.
SUBIECTUL al II-lea
Fie funct, ia f : R → (1, +∞), f (x) = 9x + 3x + 1.
R2
a) Calculat, i 0 f (x)dx.
b) Să se arate că f este bijectivă s, i să se determine g = f −1 .
c) Să se calculeze g 0 (2).
d) Să se stabilească semnul funct, iei f 00 s, i să se precizeze monotonia funct, iei f 0 .
SUBIECTUL al III-lea
În reperul de coordonate xOy se consideră punctele A(2, 1), B(1, 3), C(1, 2).
a) Precizat, i coordonatele centrului s, i raza cercului circumscris 4ABC.
b) Precizat, i coordonatele centrului s, i raza cercului ı̂nscris ı̂n 4ABC.
c) Deducet, i că arctg x + arcctg x = π2 , ∀x ∈ R.
d) Rezolvat, i ecuat, ia 2 sin4 x + 3 cos 2x + 1 = 0.

Testul 3
D.M.I. 3

Algebră

3

1. Să se rezolve ecuat, ia 2−x+ x − 1 = 1, unde x ∈ R.
8x + 27x 7
2. Să se rezolve ecuat, ia x x
= , unde x ∈ R.
12 + 18 6
3. Ştiind că suma coeficient, ilor binomiali ai dezvoltării (1 + x)n + (1 + x)n+1 (n ∈ N) este 1536,
să se determine coeficientul lui x6 .

4. Să se rezolve s, i să se discute sistemul



 mx + y + z = 1
x + my + z = m
x + y + mz = m2 + 5m + 7,

unde m ∈ R.

5. a) Determinat, i toate polinoamele f ∈ Z4 [X] astfel ı̂ncât f 2 = 0̂ s, i grad (f ) = 2.


b) Rezolvat, i aceeas, i problemă pentru polinoamele inelului Zp [X], unde p este număr prim.

Elemente de Analiză Matematică


n cos π
2k
P
1. Fie s, irul (an )n≥1 , an = k
. Arătat, i că:
k=1 2
3
Universitatea din Pites, ti, revista.matinf@upit.ro
66 PROBLEME DE MATEMATICĂ PENTRU EXAMENE

a) s, irul este strict monoton;


b) s, irul are termenii pozitivi;
c) s, irul este mărginit. § ª
2
2. Fie funct, ia f : (0, ∞) → R, f (x) = min x, . Studiat, i continuitatea s, i derivabilitatea
1 + x2 R
n
funct, iei f , apoi calculat, i limita s, irului (an )n≥1 , an = 0 f (x)dx.
x
3. Fie funct, ia f : R → R, f (x) = e 2 . Determinat, i prin induct, ie completă f (k) (x). Arătat, i că
n
f (k) (x) = f (x).
P
lim
n→∞ k=1

4. Arătat, i că funct, ia f : R → R, f (x) = (x2 + x)arctg x admite primitive s, i determinat, i


aceste primitive.
R en 1+ln t
5. Fie integrala In = e t(ln t)(1+ln2 t)
dt, n ∈ N. Calculat, i In folosind schimbarea de varia-
bilă x = ln t s, i lim In .
n→∞

Geometrie s, i trigonometrie

1. Fie AB s, i CD două coarde perpendiculare ı̂ntre ele ale unui cerc, {P } = AB ∩ CD s, i E


punctul diametral opus lui A pe cerc. Să se arate că:
a) patrulaterul cu vârfurile ı̂n punctele B, C, D, E este trapez isoscel.
b) suma P A2 + P B 2 + P C 2 + P D2 este constantă.

2. Să se rezolve ecuat, ia cos2 x + 4 sin x cos x − 2 sin2 x − 2 = 0.

3. Într-un vas ı̂nchis ı̂n formă de con circular drept cu ı̂nălt, imea h, cu vârful ı̂n jos s, i baza
orizontală se găses, te un lichid care se ridică până la ı̂nălt, imea h1 , 0 < h1 < h. Dacă vasul se
răstoarnă cu vârful ı̂n sus s, i baza orizontală, determinat, i ı̂n funct, ie de h s, i h1 ı̂nălt, imea până la
care se ridică lichidul.

4. a) Demonstrat, i că ı̂n orice triunghi ABC are loc inegalitatea

A B C 1
sin sin sin ≤ .
2 2 2 8

b) În ce caz are loc egalitatea?

5. În patrulaterul convex ABCD, E este mijlocul laturii [AB] s, i F mijlocul laturii [CD].
Să se demonstreze că dacă AB nu este paralelă cu CD s, i dacă AB nu este perpendiculară pe
CD, atunci mijloacele segmentelor [AF ], [CE], [BF ], [DE] sunt vârfurile unui paralelogram ce
nu poate fi romb.

(Admiterea la Universitatea din Pites, ti, specializările Matematică s, i Matematică-Informatică, 1998)


PROBLEME DE MATEMATICĂ PENTRU EXAMENE 67

Teste grilă pentru admiterea la facultate

Testul 1
Mihai Florea Dumitrescu 1

1. Mult, imea valorilor numărului real„ Ž are loc egalitatea det(A · B) = det(B · A),
x, astfel ı̂ncât
 ‹ 2 3
x 1 −1
unde A = s, i B = x −1 , x ∈ R este:
0 2 1
0 1
§ ª § ª § ª
1 1 1
a) {0, 1}; b) 1, ; c) 0, ; d) {−1, 1}; e) 0, .
6 √ 6 √ 2
2. Suma solut, iilor ecuat, iei 3 7x + 1 + x = 3x este:
2
a) 1; b) 2; c) 0; d) 3; e) .
9
3. Suma S = arcsin(sin 2) + arcsin(sin 4) este egală cu:
a) 6; b) Rπ + 6; c) 3; d) 2π − 6; e) π − 6.
1
4. Integrala 0 x3 (x2 + 1)4 dx este egală cu:
41 43 17 29 7
a) ; b) ; c) ; d) ; e) .
20 20 tg 2x60 30 15
5. Limita lim ( tgxx ) x−tg x este egală cu:
x→0

a) 0; b) 1; c) e; d) e2 ; e) e−2 .
6. Punctul fix al familiei de drepte dm : (m + 1)x − (2m − 1)y = m + 2, m ∈ R, este:
 ‹  ‹  ‹  ‹  ‹
5 1 4 1 5 1 5 1
a) P , 2 ; b) P 1, ; c) P , ; d) P , ; e) P , .
3 3 3 3 3 2 3 3
7. Mult, imea solut, iilor ecuat, iei cos 2x + cos 3x + cos 4x = −3, x ∈ R este:
¦ ©
a) {kπ, k ∈ Z}; b) {2k + 1)π, k ∈ Z}; c) kπ + π2 , k ∈ Z ; d) ; e) (3k+1)π

2
,k ∈ Z .
n
Ck2
P

8. Limita lim k=2 n este egală cu:


x→∞ P
nk
k=2
1 1 1
a) 0; b) 1; c) ; d) ; e) .
3 6 2
9. Dacă ecuat, ia f (x) = m, m ∈ R unde f : R \ {2, 3} → R, f (x) = ln |x2 − 5x + 6| are exact
două rădăcini reale, atunci:
a) m ∈ ; b) m ∈ (− ln 4, +∞); c) m ∈ (−∞, − ln 4);
d) m ∈ (−∞, 1); e) m ∈ (3, +∞).
10. Restul ı̂mpărt, irii polinomului f = X 2020 − X 20 + 2 la polinomul g = X(X + 1)2 este:
a) r = 2000X 2 + 2; b) r = 2000X 2 + 2000X; c) r = X 2 + 2000X + 2;
d) r = 2000X 2 +√2000X + 2; e) r = 2020X 2 + 2.
11. Limita l = lim n 1n + 2n + 3n + ... + 2020n este egală cu:
n→∞

a) 0; b) 1; c) 2021; d) 2020; e) 2020.
1
Liceul ,,S, tefan Diaconescu”, Potcoava, florin14mihai@yahoo.com
68 PROBLEME DE MATEMATICĂ PENTRU EXAMENE

„ Ž
1 5 −1
12. Valoarea lui a ∈ R pentru care rangul matricei A = a 2 a este 2, este:
5 7 3
a) 0; b) 2; c) 4; d) -2; e) 3.
13. Dacă ε = cos 2kπ
5
+ i sin 2kπ
5
, k ∈ {1, 2, 3, 4}, atunci numărul (1 + ε) · (1 + ε2 ) · . . . · (1 + ε2020 )
este egal cu:
a) 2404 ; b) √
22020 ; c) 2√505 ; d) 2; e) 0.
p
3 3
14. Suma S = [ 12 · 4] + [ 22 · 5] + · · · + [ 3 n2 (n + 3] este egală cu:
n2 + n n+1 n
a) n2 ; b) n2 + n; c) ; d) ; e) .
2 2 2
R 1 x4
15. Integrala −1 x dx este egală cu:
e +1
1 3 1 2
a) e; b) − ; c) ; d) ; e) .
5 5 5 5

Testul 2

Mihaela Gabor 2

1. Negat, i propozit, ia ”Tot, i copiii au ochii verzi s, i părul negru”.


a) Nu tot, i copiii au ochii verzi s, i părul negru.
b) Nu tot, i copiii au ochii verzi sau părul negru.
c) Există copii care nu au ochii verzi s, i părul negru.
d) Există copii care nu au ochii verzi • sau ˜ părul negru.
2x x+1
2. Rezolvat, i ı̂n numere reale ecuat, ia = .
3 2
a) {2,3,4}; b) {3}; c) {3,5,7}; d) {5,7}.
 ‹
2 5 30
3. Să se calculeze cel mai mare termen al dezvoltării + .
7 7
a) T1 ; b) T31 ; c) T21 ; d) T22 .
4. Rezolvat, i ı̂n numere reale ecuat, ia: x3 lg x = 10x2 .
1 1 1
a) 1,- ; b) 1; c ) 10, 3 ; d) 10, √ 3
.
3 10 10
|x| − 2 |x| + 1
5. Rezolvat, i ı̂n numere reale ecuat, ia + = 6.
4 2
a) {2,3}; b) {-8,8}; c) {-6,6}; d) {-7,7}.
6. Câte funct, ii f : {1, 2, 3} → {1, 2, 3, 4, 5} cu f (2) 6= f (3), se pot defini?
a) 75; b) 100; c) 125; d) 50.
7. Studiat, i graficul din imagine s, i stabilit, i intervalele ı̂n care funct, ia este strict negativă.

2
Colegiul Nat, ional ,,C. Carabella”, Târgovis, te, mihaela gab0r@yahoo.com
PROBLEME DE MATEMATICĂ PENTRU EXAMENE 69

a) (−∞, 1); b) (−1, 1] ∪ (4, ∞); c) (−∞, 0); d) (−1, 1) ∪ (3, ∞).
8. Studiat, i graficul de mai sus s, i stabilit, i imaginea fiecărei ramuri.
 
 Im f1 = (−∞, −2]  Im f1 = [−2, ∞)
a) Im f2 = (1, 3) b) Im f2 = {1}
Im f3 = (−∞, 3] Im f3 = (−∞, 3]
 
 
 Im f1 = [−2, ∞)  Im f1 = (−∞, −2]
c) Im f2 = {1} d) Im f2 = {1}
Im f3 = (−∞, 3) Im f3 = (−∞, 3]
 
9. Se dau mult, imile A = {−2, 2}, B = {−1, 1, 3}. Să se determine A × B.


 x + ay + z = 0
10. Fie sistemul ax − ay + z = 0 , a ∈ R. Valoarea parametrului real a ∈ R
x + (2a − 1)y + z = 0

70 PROBLEME DE MATEMATICĂ PENTRU EXAMENE

pentru care sistemul admite s, i solut, ii diferite de solut, ia nulă este:


a) a = −1; b) a = 5; c) a = 2; d) a = 1.
11. Fie s, irul {xn }n∈N , dat de formula
n  ‹
1X 2
xn = ln 1 + , n ∈ N, n > 0.
n k=1 k

Atunci limita s, irului este:


a) 1; b) e; c) e2 ; d) 0. §
ln3 x, dacă x ∈ (0, e]
12. Valoarea constantelor a, b ∈ R pentru care funct, ia f (x) =
ax + b, dacă x > e
este derivabilă pentru orice x > 0 este:
a) a = e, b = e; b) a = 3/e, b = −2; c) a = 1, b = 1; d) a = 1, b = −1.
13. Ecuat, ia tangentei la graficul funct, iei f : R → R, f (x) = x3 − 6x2 + 5x + 4 ı̂n punctul de
inflexiune este:
a) y = −x; b) y = x; c) y = −7x  + 12; d) y = 3x − 4.
π
R 1 cos 2 ln x
14. Valoarea integralei e dx este:
x
a) I = 1; b) I = 0; c) I = − π2 ; d) I = ln π2 .
15. Se consideră funct, ia f : R − {−1} → R, f (x) = x + 4(x + 1)−2 s, i fie S(a) aria domeniului
plan delimitat de graficul funct, iei s, i de dreptele y = x, x = 1, x = a, a > 1. Atunci
L = lim S(a) este:
a→∞

a) L = 1; b) L = 2; c) L = 1 + 4 ln 2; d) L = ln 2.

Testul 3
Rădulescu Maria-Florentina 3

1. Într-o progresie aritmetică (an )n≥1 se cunosc a4 = 7 s, i a9 = 22. Atunci termenul a12 este
egal cu:
a) 31; b) 9; c) 6; d) 8; e) 12.
2x
2. Imaginea funct, iei f : R → R, f (x) = este:
x2 +1
a) [-1,1]; b) (-1,1]; c) [-1,1); d) (-1,1); e)(-1,2].
3. Solut, iile reale ale ecuat, iei f 2 (x) + 2f (x) − 1 = 0, s, tiind că f (x) = x + 1, f : R → R, sunt:
√ √
a) {1, 2}; b) {0,4}; c) {-4,0}; d) {−2 − 2, −2 + 2}; e) {6,8}.
4. Dacă x1 , x2 sunt rădăcinile reale ale ecuat, iei x2 − 6x + 5 = 0, atunci ecuat, ia care are
x1 + 1 x2 + 1
rădăcinile s, i este echivalentă cu:
2 2
a) y 2 − 3y + 3 = 0; b) 2y 2 − 4y + 3 = 0; c) y 2 + 4y − 3 = 0;
d) y 2 − 4y + 3 = 0; e) y 2 + 2y + 3 = 0.
1 + ln x
5. Fie funct, ia f : (0, ∞) \ {e} → R, f (x) = . Atunci f 0 (x) este:
1 − ln x
1 2 2 3 2
a) ; b) 2
; c) ; d) ; e) .
x(1 − ln x) (1 − ln x) x ln x x(1 − ln x) x(1 − ln x)2
3
Profesor, S, coala Gimnazială Nr.1, Pietros, ani, raducumaria31@gmail.com
PROBLEME DE MATEMATICĂ PENTRU EXAMENE 71

 ‹
2 1
6. Se consideră matricea A = s, i funct, ia f : M2 (R) → M2 (R), f (X) = AX 2 .
−3 2
Atunci f (A) este:
 ‹  ‹  ‹  ‹  ‹
10 9 −10 −9 10 −9 9 10 −10 9
a) ; b) ; c) ; d) ; e) .
27 10 −27 10 27 10 10 27 −10 −27
√  1
7. Rezultatul calculului [ 2020] + 6 · − 3 , unde [x] reprezintă partea ı̂ntreagă a lui x s, i
{x} reprezintă partea fract, ionară a lui x este:
a) 44; b) 46; c) 48; d) 42; e) 40.
8. Ştiind că x1 , x2 , x3 ∈ C sunt solut, iile ecuat, iei x3 +4x = 0, atunci valoarea determinantului
x1 x2 x3
x2 x3 x1 este:
x3 x1 x2
a) 1; b) 2; c) 0; d) „ -2; e) 4. √ Ž
lg 10 eln 16
!
− lg 2 4
9. Se dau matricele A = 1 √ ;B= e . Calculat, i C =
e ln e log3 9 − + 6
log9 3 2
A + B.
 ‹ 8 4
!  ‹  ‹  ‹
lg 5 8 4 7 5 −8 4 8
a) ; b) e ; c) ; d) ; e) .
4 6 3 4 6 4 6 5 6
2
10. Se consideră polinomul f ∈ C[X], f = (X + i)100 + (X − i)100 , care are forma algebrică
f = a100 X 100 + a99 X 99 + ... + a1 X + a0 . Diferent, a a100 − a99 este:
a) 3; b) i; c) 4; d) 2; e) 1.
1 R3
11. Se consideră funct, iile fa : R → R, fa (x) = , a ∈ R. Atunci f2 (x)dx este:
|x − a| + 4 0
20 20 90 90 15
a) ln ; b) − ln ; c) − ln ; d) ln ; e) ln .
9 9 17  17 ‹ 8
3 3
12. Se consideră matricea A ∈ M2 (R), A= . Atunci (A − At )2020 este:
2 2
 ‹  ‹  ‹  ‹  ‹
3 0 0 1 1 0 −1 0 0 1
a) ; b) ; c) ; d) ; e) .
0 2 1 0 0 −1 0 1 −1 0
13. Solut, ia ı̂ntreagă a ecuat, iei
2020
|x ◦ x ◦{z· · · ◦ x} = 2 + 3,
de 2020 ori

s, tiind că x ◦ y = xy − 3x − 3y + 12, ∀x, y ∈ Z, este:


a) 4; b) 5; c) 3; d) 6; e) 2.
 4x + 3y = 5
14. Solut, ia sistemului 5x + 3z = 4 este:
5y + 4z = 3

 ‹  ‹  ‹  ‹
3 4 3 4 4 3 1 2 3
a) , , 1 ; b) , 1, ; c) (0,1,2); d) , , 0 ; e) , , .
5 5 5 5 √5 5 5 5 5
 ‹
2 3 2x + 1
15. Funct, ia f pentru care F : R → R, F (x) = arctg √ , x ∈ R, este o primitivă
3 3
a sa, are forma:
1 1 1
a) x2 + x + 1; b) x2 − x + 1; c) 2 ; d) 2 ; e) 2 .
x −x−1 x −x+1 x +x+1
72 PROBLEME DE MATEMATICĂ PENTRU EXAMENE

Testul 4
Raluca Mihaela Georgescu 4
√ √
3
1. Suma solut, iilor reale ale ecuat, iei x+2= 3x + 2 este:
a) 2; b) 4; c) 1; d) 3; e) 0.
2. Valorile reale ale lui x pentru care este definit logaritmul log x2 − 4 (x − 1) sunt:
2x − 5
 ‹  ‹  ‹  ‹
5 5 5 5
a) (1, 2) ∪ , ∞ ; b)[1, 2] ∪ , ∞ ; c) (−1, 2) ∪ , ∞ ; d) (−2, 2) ∪ ,∞ ;
2  ‹ 2 2 2
5
e) (−1, 1) ∪ (1, 2) ∪ ,∞ .
2
x3 + 2x2 + x + 3
3. Fie f : R → R, f (x) = . Atunci lim F (x), unde F (x) este acea primitivă
x2 + 1 x→0
π
a lui f pentru care F (1) = , este:
4
5 5 1 1
a) ; b) − ; c) ; d) − ; e) 0.
2 2 2 2 2
4. Modulul sumei rădăcinilor ecuat, iei 2x +x−1 = 32 este:
a) 1; b) 5; c) 2; d) 3; e) 4.
x3 + 2
5. Numărul asimptotelor funct, iei f : R → R, f (x) = este:
x2 + 5x + 6
a) 1; b) 2; c) 3; d) 4; e) 0.
6. Dacă ı̂ntr-o progresie geometrică (bn )n≥1 cu rat, ia supraunitară avem b1 · b2 · b3 = 27 s, i
b1 + b2 + b3 = 13, atunci S10 este:
310 − 1 311 − 1 310 + 1 311 + 1 310 − 1
a) ; b) ; c) ; d) ; e) .
2 2 2 2 6
7. Valorile parametrilor a, b ∈ R pentru care polinomul f = X 5 −5X 4 −6X 3 +76X 2 −8aX +4b
are o rădăcină reală triplă sunt:
a) a = 18, b = 22; b) a = 19, b = 24; c) a = −19, b = 24;
d) a = 19, b = −22; e) a = 19, b = −24.
f (x)
8. Fie f : R → R, f (x) = x4 − 24x2 + 45x − 10. Atunci lim , unde a > 0 este
x→a x2 +x−6
punct de inflexiune, este:
19 19 12 12
a) − ; b) ; c) − ; d) ; e) −12.
5 5 5 ‚5 1 3
Œ
3 log3 5
3 log4 16 − log2 4
9. Fie matricea A ∈ M2 (R), A = 1
p . Atunci An
log4 3 − 2 log2 3 3
log3 38 + log3 3−7
este:
 ‹  ‹  ‹  ‹  ‹
5n−1 0 5n+1 0 5n 0 5n 0 5n+1 0
a) ; b) ; c) ; d) ; e) .
0 1 0 0 0 1 0 0 0 1
10. Mult, imea solut, iilor ecuat, iei x3 − 2̂x2 + x + 3̂ = 0̂ peste corpul Z5 este:
a) {4̂}; b) {2̂}; c) {1̂, 3̂}; d) {3̂}; e) {2̂, 3̂}.
|x2 − 9|
11. Aria subgraficului determinat de graficul funct, iei f : R∗ → R, f (x) = , axa Ox s, i
|x|
dreptele de ecuat, ii x = 2, x = 4 este:
4
Lect. univ. dr., Universitatea din Pites, ti, gemiral@yahoo.com
PROBLEME DE MATEMATICĂ PENTRU EXAMENE 73

9 9 8 3 3
a) 1 − 9 ln ; b) 1 + 9 ln ; c) 2 + 9 ln ; d) 1 − 9 ln ;; e) 1 + 9 ln .
8 8 9 2 2
12. Numărul rădăcinilor din intervalul (2, 5) ale ecuat, iei x3 − 6x2 + 9x + 4 = 0 este:
a) 1; b) 3; c) 2; d) 0; e) 4.
13. Valorile parametrului real m pentru care parabola asociată funct, iei f : R → R, f (x) =
mx2 + (m + 2)x + 3 se află deasupra axei Ox sunt:
√ √ √ √
a) m ∈ (−4 3, 4 3); b) m ∈ (4 3, ∞); c) m ∈ [−4 3, 0);
√ √
d) m ∈ (0, 4 3]; e) m ∈ (0, 4 3).
x 3 0
x+3 x−1
14. Suma pătratelor rădăcinilor ecuat, iei 1 x 4 = este:
1 5
3 1 x
a) 22; b) 23; c) 10; d) 20; e) 21.
R2 x2 +3x
log3 3 ln x
15. Valoarea integralei dx este:
1 x+3
4 3 3
a) ln ; b) 2 ln 2 − 13; c) ln 4e; d) ln 4 + ; e) ln 4 − .
e 4 4
74 PROBLEME DE INFORMATICĂ PENTRU EXAMENE

PROBLEME DE INFORMATICĂ PENTRU


EXAMENE

Teste pentru examenul de Bacalaureat, specializarea S, tiint, e ale naturii

Testul 1

1
Maria Miroiu

Limbajul C/C++
SUBIECTUL I (20 de puncte)
Pentru fiecare dintre itemii de la 1 la 5, scriet, i pe foaia de examen litera cores-
punzătoare răspunsului corect.

1. Care dintre următoarele expresii C/C++ are valoarea 1 dacă s, i numai dacă variabilele x
s, i y (de tip int) memorează două numere impare consecutive? (4p.)
a) (x+y)%2==0 c) x%2==1 && y%2==1
b) (x-y==2) || (y-x==2) d) abs(x-y)==2 && y%2==1
2. Care din următoarele expresii C/C++ are ca rezultat valoarea maximă dintre x s, i y?
(4p.)
a) (x+y-abs(x-y))/2 c) (x-y-abs(x+y))/2
b) (x+y+abs(x-y))/2 d) x+y-abs(x-y)/2
3. Care din următoarele instruct, iuni C/C++ realizează eliminarea cifrei sutelor pentru un
număr natural nenul x dat? (4p.)
a) x = x%1000*100+x%100; c) x = x%100*100+x/100;
b) x = x/1000*100+x%100; d) x = x/1000+x%100;
4. Pentru a verifica dacă ı̂ntr-un tablou unidimensional există elementul cu valoarea x=37
prin aplicarea metodei căutare binară, presupunem că succesiunea de elemente a căror
valoare se compară cu x este 44, 26, 37. În acestă situat, ie, care pot fi elementele tabloului
unidimensional? (4p.)
a) 26,29,37,44,48,63,68 c) 14,26,37,44,48,63,68,72
b) 14,26,37,41,45,62,67,73 d) 3,26,37,41,44,63,68,72
5. Care este numărul de comparat, ii realizate la aplicarea algoritmului de interclasare a
vectorilor (7,9,9) s, i (6,10,14,15)? (4p.)
a) 5 b) 6 c) 4 d) 3

SUBIECTUL al II-lea (40 de puncte)


Scriet, i pe foaia de examen răspunsul corect pentru fiecare dintre cerint, ele
următoare.

1. Se consideră algoritmul următor, descris ı̂n pseudocod, ı̂n care toate variabilele sunt de tip
ı̂ntreg, cu valori strict pozitive. Notat, ia x%y reprezintă restul ı̂mpărt, irii valorii variabilei x
la valoarea variabilei y, iar [x] partea ı̂ntreagă a valorii numărului real x.
1
Lect. univ. dr., Universitatea din Pites, ti, maria.miroiu@gmail.com
PROBLEME DE INFORMATICĂ PENTRU EXAMENE 75

a) Scriet, i ce valoare se va afis, a, dacă citeste n , d


se citesc, ı̂n această ordine, valorile m ← 0
5,3,15,54,7,9,27. (6p.) pentru i←1 , n executa
b) Dacă pentru n s, i d se citesc valorile 4, | citeste x
| k ← 0
respectiv 2, scriet, i un s, ir de n valori
| cat timp x % d =0 executa
de intrare care s-ar putea citi, astfel | | k ← k +1
ı̂ncât, ı̂n urma executării algoritmu- | |_ x ← [x/d]
lui,să afis, eze valoarea 4. (6p.) | daca k > m atunci
c) Scriet, i un program C/C++ cores- |_ |_ m ← k
punzător algoritmului dat. (10p.) scrie m
d) Scriet, i ı̂n pseudocod un algoritm, echi-
valent cu cel dat, ı̂nlocuind structura
cat timp ... executa cu o altă
structură repetitivă. (6p.)
2. Se consideră variabilele A,B,C de tip structură cu eticheta punct2D, structură ce memorează
valorile reale ale abscisei s, i ordonatei unui punct din plan. Scriet, i o secvent, ă de instruct, iuni
C/C++ pentru afis, area abscisei s, i ordonatei centrului de greutate al triunghiului de vârfuri
A,B,C. (6p.)
3. În secvent, a de instruct, iuni de mai jos, variabilele i s, i j sunt de tip ı̂ntreg. Fără a utiliza
alte variabile, scriet, i una sau mai multe instruct, iuni C/C++ ce pot ı̂nlocui punctele de
suspensie, astfel ı̂ncât, ı̂n urma executării secvent, ei obt, inute, să se afis, eze matricea de mai
jos, dreapta. (6p.)
for ( i =1; i <=5; i ++) { < < < < *
for ( j =1; j <=5; j ++) < < < * >
............... < < * > >
printf ("\ n ");| cout < < endl ; < * > > >
} * > > > >

SUBIECTUL al III-lea (30 de puncte)


Scriet, i pe foaia de examen răspunsul corect pentru fiecare dintre cerint, ele
următoare.
1. Scriet, i, ı̂n pseudocod, un algoritm care cites, te de la tastatură două numere naturale n
s, i m de maxim 9 cifre, apoi determină s, i afis, ează factorii primi care se găsesc atât ı̂n
descompunerea ı̂n factori primi a lui n, cât s, i ı̂n descompunerea ı̂n factori primi a lui m,
la aceeas, i putere, iar dacă nu există factori comuni la aceeas, i putere se va afis, a mesajul
Imposibil. (10p.)
Exemple: Pentru n=2100 s, i m=600 se afis, ează 3, 5 (2100 = 22 · 3 · 52 · 7, 600 = 23 · 3 · 52 ),
pentru n=120 s, i m=1400 se afis, ează 2 (120 = 23 · 3 · 5, 1400 = 23 · 52 · 7), iar pentru n=6 s, i
m=9 se afis, ează Imposibil (6 = 2 · 3, 9 = 32 ).
2. Se citesc de la tastatură trei numere naturale nenule: n, m s, i k. Considerând un tablou
unidimensional x cu elementele 1,2,...,n, scriet, i un program C/C++ care elimină ele-
mente din x, ı̂ncepând cu al m-lea, apoi cele din k ı̂n k. (10p.)
Exemplu: Pentru n=15, m=2 s, i k=3, tabloul unidimensional x devine (1,3,4,6,7,
9,10,12,13,15).
3. Fis, ierul bac.txt cont, ine pe prima linie valoarea naturală nenulă a variabilei n (≤ 106 ), iar
pe următoarea linie un s, ir de n numere naturale de cel mult 2 cifre fiecare, acestea fiind
separate prin spat, ii. Se cere afis, area pe ecran a celui mai mare palindom de 7 cifre impare
care apar ı̂n component, a celor n numere de pe linia a doua din fis, ier (dacă nu se poate
contrui un astfel de palindrom, se va afis, a pe ecran valoarea 0).
76 PROBLEME DE INFORMATICĂ PENTRU EXAMENE

a) Descriet, i ı̂n limbaj natural un algoritm eficient de rezolvare a problemei. (3p.)


b) Scriet, i un program C/C++ care cites, te datele din fis, ier s, i rezolvă problema. (7p.)
Exemplu: Dacă ı̂n fis, ierul bac.txt se află numerele:
10
65 19 38 4 67 16 4 7 81 15
atunci pe ecran se vor afis, a valorile 7519157.

Testul 2

2
Maria Miroiu

Limbajul Pascal
SUBIECTUL I (20 de puncte)
Pentru fiecare dintre itemii de la 1 la 5, scriet, i pe foaia de examen litera cores-
punzătoare răspunsului corect.

1. Care dintre următoarele expresii Pascal are valoarea true dacă s, i numai dacă variabilele x
s, i y (de tip integer) memorează două numere impare consecutive? (4p.)
a) not odd(x+y) c) (odd(x)) and (odd(y))
b) (x-y=2) or (y-x=2) d) (abs(x-y) = 2) and (odd(y))
2. Care din următoarele expresii Pascal are ca rezultat valoarea maximă dintre x s, i y? (4p.)
a) (x+y-abs(x-y)) div 2 c) (x-y-abs(x+y)) div 2
b) (x+y+abs(x-y)) div 2 d) x+y-abs(x-y) div 2
3. Care din următoarele instruct, iuni Pascal realizează eliminarea cifrei sutelor pentru un
număr natural nenul x dat? (4p.)
a) x = (x mod 1000)*100+(x mod 100);
b) x = (x div 1000)*100+(x mod 100);
c) x = (x mod 100)*100+(x div 100);
d) x = x div 1000 + x mod 100;
4. Pentru a verifica dacă ı̂ntr-un tablou unidimensional există elementul cu valoarea x=37
prin aplicarea metodei căutare binară, presupunem că succesiunea de elemente a căror
valoare se compară cu x este 44, 26, 37. În acestă situat, ie, care pot fi elementele tabloului
unidimensional? (4p.)
a) 26,29,37,44,48,63,68 c) 14,26,37,44,48,63,68,72
b) 14,26,37,41,45,62,67,73 d) 3,26,37,41,44,63,68,72
5. Care este numărul de comparat, ii realizate la aplicarea algoritmului de interclasare a
vectorilor (7,9,9) s, i (6,10,14,15)? (4p.)
a) 5 b) 6 c) 4 d) 3

SUBIECTUL al II-lea (40 de puncte)


Scriet, i pe foaia de examen răspunsul corect pentru fiecare dintre cerint, ele
următoare.
2
Lect. univ. dr., Universitatea din Pites, ti, maria.miroiu@gmail.com
PROBLEME DE INFORMATICĂ PENTRU EXAMENE 77

1. Se consideră algoritmul următor, descris ı̂n pseudocod, ı̂n care toate variabilele sunt de tip
ı̂ntreg, cu valori strict pozitive. Notat, ia x%y reprezintă restul ı̂mpărt, irii valorii variabilei x
la valoarea variabilei y, iar [x] partea ı̂ntreagă a valorii numărului real x.
a) Scriet, i ce valoare se va afis, a, dacă citeste n , d
se citesc, ı̂n această ordine, valorile m ← 0
5,3,15,54,7,9,27. (6p.) pentru i←1 , n executa
b) Dacă pentru n s, i d se citesc valorile 4, | citeste x
| k ← 0
respectiv 2, scriet, i un s, ir de n valori
| cat timp x % d =0 executa
de intrare care s-ar putea citi, astfel | | k ← k +1
ı̂ncât, ı̂n urma executării algoritmului, | |_ x ← [x/d]
să afis, eze valoarea 4. (6p.) | daca k > m atunci
c) Scriet, i un program Pascal cores- |_ |_ m ← k
punzător algoritmului dat. (10p.) scrie m
d) Scriet, i ı̂n pseudocod un algoritm, echi-
valent cu cel dat, ı̂nlocuind structura
cat timp ... executa cu o altă
structură repetitivă. (6p.)
2. Se consideră variabilele A,B,C de tip record cu eticheta punct2D ce memorează valorile
reale ale abscisei s, i ordonatei unui punct din plan. Scriet, i o secvent, ă de instruct, iuni Pascal
pentru afis, area abscisei s, i ordonatei centrului de greutate al triunghiului de vârfuri A,B,C.
(6p.)
3. În secvent, a de instruct, iuni de mai jos, variabilele i s, i j sunt de tip ı̂ntreg. Fără a utiliza
alte variabile, scriet, i instruct, iunile Pascal ce pot ı̂nlocui punctele de suspensie, astfel ı̂ncât,
ı̂n urma executării secvent, ei obt, inute, să se afis, eze matricea de mai jos, dreapta. (6p.)
for i :=1 to 5 do < < < < *
begin < < < * >
for j :=1 to 5 do < < * > >
............... < * > > >
writeln ; * > > > >
end ;

SUBIECTUL al III-lea (30 de puncte)


Scriet, i pe foaia de examen răspunsul corect pentru fiecare dintre cerint, ele
următoare.

1. Scriet, i, ı̂n pseudocod, un algoritm care cites, te de la tastatură două numere naturale n
s, i m de maxim 9 cifre, apoi determină s, i afis, ează factorii primi care se găsesc atât ı̂n
descompunerea ı̂n factori primi a lui n, cât s, i ı̂n descompunerea ı̂n factori primi a lui m,
la aceeas, i putere, iar dacă nu există factori comuni la aceeas, i putere se va afis, a mesajul
Imposibil. (10p.)
Exemple: Pentru n=2100 s, i m=600 se afis, ează 3, 5 (2100 = 22 · 3 · 52 · 7, 600 = 23 · 3 · 52 ),
pentru n=120 s, i m=1400 se afis, ează 2 (120 = 23 · 3 · 5, 1400 = 23 · 52 · 7), iar pentru n=6 s, i
m=9 se afis, ează Imposibil (6 = 2 · 3, 9 = 32 ).
2. Se citesc de la tastatură trei numere naturale nenule: n, m s, i k. Considerând un tablou
unidimensional x cu elementele 1,2,...,n, scriet, i un program Pascal care elimină ele-
mente din x, ı̂ncepând cu al m-lea, apoi din k ı̂n k. (10p.)
Exemplu: Pentru n=15, m=2 s, i k=3, tabloul unidimensional x devine (1,3,4,6,7,
9,10,12,13,15).
78 PROBLEME DE INFORMATICĂ PENTRU EXAMENE

3. Fis, ierul bac.txt cont, ine pe prima linie valoarea naturală nenulă a variabilei n (≤ 106 ), iar
pe următoarea linie un s, ir de n numere naturale de cel mult 2 cifre fiecare, acestea fiind
separate prin spat, ii. Se cere afis, area pe ecran a celui mai mare palindom de 7 cifre impare
care apar ı̂n component, a celor n numere de pe linia a doua din fis, ier (dacă nu se poate
contrui un astfel de palindrom, se va afis, a pe ecran valoarea 0).
a) Descriet, i ı̂n limbaj natural un algoritm eficient de rezolvare a problemei. (3p.)
b) Scriet, i un program Pascal care cites, te datele din fis, ier s, i rezolvă problema. (7p.)
Exemplu: Dacă ı̂n fis, ierul bac.txt se află numerele:
10
65 19 38 4 67 16 4 7 81 15
atunci pe ecran se vor afis, a valorile 7519157.
PROBLEME DE INFORMATICĂ PENTRU EXAMENE 79

Teste pentru examenul de Bacalaureat, specializarea


Matematică-Informatică

Testul 1

Maria Tătulea 1

Limbajul C/C++
SUBIECTUL I (20 de puncte)
Pentru fiecare dintre itemii de la 1 la 5, scriet, i pe foaia de examen litera cores-
punzătoare răspunsului corect.
1. Variabila x este de tip ı̂ntreg s, i poate memora un număr din intervalul [0..109 ). Indicat, i
valoarea maximă pe care o poate avea expresia C/C++: x%19. (4p.)
a) 19 b) 0.19 c) 18 d) 52631578
2. Fie funct, ia de mai jos. Câte caractere ’*’ se vor afis, a dacă se va apela f(9)? (4p.)
void f ( int x )
{ cout < <"*"; | printf ("*");
if (x >3)
{ f (x -1);
cout < < x ; | printf ("% d " , x );
}
cout < <"*"; | printf ("*");
}
a) 14 b) 13 c) 15 d) 12
3. Utilizând metoda backtracking se generează toate modalităt, ile de a forma o delegat, ie de 3
sau 4 persoane din mult, imea {Ionel, Gigel, Ina, Elena, Andrei}. Primele trei solut, ii sunt:
{Ionel, Gigel, Ina}, {Ionel, Gigel, Ina, Elena}, {Ionel, Gigel, Ina, Andrei}. Care este a
s, aptea solut, ie? (4p.)
a) {Ionel, Ina, Gigel, Andrei} c) {Ionel, Ina, Elena}
b) {Ionel, Ina, Elena} d) {Ionel, Ina, Elena, Andrei}
4. Un arbore cu 11 noduri, numerotate de la 1 la 11, este reprezentat prin vectorul de ”tat, i”
(8, 5, 10, 7, 0, 2, 8, 5, 5, 2, 9). Indicat, i lungimea celui mai lung lant, din arbore. (4p.)
a) 3 b) 6 c) 5 d) 4
5. Se dă graful neorientat cu 6 noduri numerotate de la 1 la 6 s, i muchiile: [1,2], [1,3], [1,6],
[2,3], [2,5], [3,4], [4,5], [4,6], [5,6]. Indicat, i numărul minim de muchii care trebuie adăugate
pentru ca graful obt, inut să fie eulerian. (4p.)
a) 2 b) 0 c) 3 d) 4

SUBIECTUL al II-lea (40 de puncte)


Scriet, i pe foaia de examen răspunsul pentru fiecare din cerint, ele următoare.

1. Algoritmul următor este reprezentat ı̂n pseudocod. S-a notat cu a%b restul ı̂mpărt, irii
numărului natural a la numărul natural nenul b s, i cu [c] partea ı̂ntreagă a numărului
real c.

1
Profesor, Colegiul Nat, ional ,,Dinicu Golescu”, Câmpulung, mariatatulea@yahoo.com
80 PROBLEME DE INFORMATICĂ PENTRU EXAMENE

a) Scriet, i valoarea afis, ată dacă numărul citeste n


citit este 91. (6p.) x ← 1
b) Care sunt cea mai mică, respectiv y ← 2
cat timp x6= 0 si y * y≤n executa
cea mai mare valoare pe care o poate | k ← n
lua n ı̂n intervalul [1..300] astfel | cat timp k≥y executa
ı̂ncât algoritmul să afis, eze valoarea | |_ k ← k-y
11? (6p.) | x ← k
c) Scriet, i programul C/C++ cores- | _ y ← y +1
punzător algoritmului dat. (10p.) y ← y -1
scrie y
d) Scriet, i ı̂n pseudocod un algoritm, echi-
valent cu cel dat, care să folosească o
singură structură repetitivă. (6p.)
2. Variabila e memorează informat, ii pentru fiecare dintre cei 25 de elevi ai unei clase. Ştiind
că: e[0].nume reprezintă numele (un s, ir de maxim 30 de caractere), e[0].medie repre-
zintă media (un număr real), iar e[0].datan.zi, e[0].datan.luna, e[0].datan.an
reprezintă ziua, luna, respectiv anul nas, terii (numere naturale) ale primului elev, scriet, i
definit, ia unei structuri cu eticheta elev, declarat, i corespunzător variabila e s, i scriet, i o
expresie care va avea valoarea 1 dacă ultimul elev este născut ı̂n primele zece zile ale lunii
iulie. (6p.)
3. Variabilele i s, i j sunt de tip ı̂ntreg, iar variabila a memorează un tablou bidimensional cu 8
linii s, i 8 coloane, numerotate de la 0 la 7, având init, ial toate elementele egale cu caracterul
@. Fără a utiliza alte variabile, scriet, i secvent, a de instruct, iuni de mai jos, ı̂nlocuind
punctele de suspensie astfel ı̂ncât, ı̂n urma executării secvent, ei obt, inute, variabila a să
memoreze tabloul alăturat. (6p.)
for ( i =0; i <8; i ++) $ ( ( ( ) ) ) $
for ( j =0; j <8; j ++) [ $ ( ( ) ) $ ]
............ [ [ $ ( ) $ ] ]
[ [ [ $ $ ] ] ]
] ] ] $ $ [ [ [
] ] $ ( ) $ [ [
] $ ( ( ) ) $ [
$ ( ( ( ) ) ) $

SUBIECTUL al III-lea (30 de puncte)


Scriet, i pe foaia de examen răspunsul pentru fiecare din cerint, ele următoare.
1. Subprogramul modifica primes, te un număr natural n ∈ [2, 102 ], apoi cele n elemente
ale unui tablou unidimensional v, numere ı̂ntregi din intervalul [−102 , 102 ], dintre care
cel put, in unul este pozitiv s, i cel put, in unul este negativ. Subprogramul transformă ı̂n
memorie tabloul, eliminând din component, a sa toate numerele negative, apoi inserează
ı̂ntre oricare două elemente aflate pe pozit, ii consecutive cel mai mare divizor comun al lor.
Tabloul modificat este transmis tot prin parametrul v.
Exemplu: pentru n = 10 s, i tabloul unidimensional (2, 5, 2, 4, -3, 4, -2, -7, -2, 9), se
obt, ine tabloul unidimensional (2, 1, 5, 1, 2, 2, 4, 4, 4, 1, 9). (10p.)
2. Într-un text cu cel mult 200 de caractere, cuvintele sunt formate din litere mici ale
alfabetului englez s, i sunt separate prin câte un spat, iu. Scriet, i un program C/C++ care
cites, te de la tastatură un text de tipul ment, ionat s, i afis, ează pe ecran, separate de câte o
PROBLEME DE INFORMATICĂ PENTRU EXAMENE 81

virgulă, perechile de cuvinte din text cu proprietatea că toate caracterele din al doilea
cuvânt se regăsesc ı̂n primul cuvânt, ı̂n aceeas, i ordine, nu neapărat pe pozit, ii consecutive.
Dacă nu a fost găsită nicio pereche se va afis, a mesajul ”nu există”.
Exemplu: pentru textul
dupa ore de mers prin pestera, se puteau observa pe tavan forme conice
se vor afis, a perechile
forme ore, pestera se, observa se, puteau pe, pestera pe. (10p.)
3. Se consideră s, irul 1, 1, 1, 8, 29, 85, 246, 715 ... definit astfel: f1 = f2 = f3 = 1,
fn = 3 · fn−1 − fn−2 + 2 · fn−3 + 4 (unde n este un număr natural, n ≥ 4). Se citesc de
la tastatură trei numere naturale x, y, z (numere naturale cu maxim 9 cifre), valorile a
trei termeni aflat, i pe pozit, ii consecutive ı̂n s, irul dat, s, i se cere să se scrie ı̂n fis, ierul text
bac.txt, ı̂n ordine descrescătoare, separat, i prin câte un spat, iu, tot, i termenii s, irului care
sunt mai mici sau egali cu z. Proiectat, i un algoritm eficient din punctul de vedere al
memoriei utilizate s, i al timpului de executare.
Exemplu: dacă se citesc numerele 85 246 715, fis, ierul bac.txt cont, ine numerele 715 246
85 29 8 1 1 1 .
a) Scriet, i programul C/C++ corespunzător algoritmului proiectat. (8p.)
b) Descriet, i ı̂n limbaj natural algoritmul proiectat, justificând eficient, a acestuia. (2p.)

Testul 2

Cristina Constantinescu 2

Limbajul C/C++
SUBIECTUL I (20 de puncte)
Pentru fiecare dintre itemii de la 1 la 5, scriet, i pe foaia de examen litera cores-
punzătoare răspunsului corect.
1. Variabila x este de tip ı̂ntreg s, i poate memora un număr din intervalul [1000,2000].
Indicat, i valoarea minimă pe care o poate avea expresia C/C++: (x-999)%1000. (4p.)
a) 0 b) 2 c) 999 d) 1000
2. Subprogramul f este definit mai jos. Indicat, i ce se afis, ează ı̂n urma apelului: f(9); (4p.)
void f ( int x )
{ if (x >=2)
{ if ( x %2==0) cout < <0; | printf ("0");
f ( x /2);
}
else cout < <7; | printf ("7");
cout < < x ; | printf ("% d " , x );
}

2
Profesor, Colegiul Nat, ional ,,Zinca Golescu”, Pites, ti, cristina.constantinescu70@gmail.com
82 PROBLEME DE INFORMATICĂ PENTRU EXAMENE

a) 91 b) 942 c) 0071 d) 0071249


3. Utilizând metoda backtracking, se generează toate meniurile care cuprind câte patru feluri
distincte de mâncare din mult, imea {aperitiv, desert, legume, paste, salată, supă},
fiecare meniu respectând următoarele condit, ii:
· dacă există aperitiv, este servit primul;
· dacă există desert, este servit ultimul;
· NU sunt servite legume s, i salată ı̂n acelas, i meniu;
· dacă există paste s, i supă ı̂n acelas, i meniu, pastele NU sunt servite ı̂nainte de supă.
Două meniuri sunt distincte dacă diferă prin cel put, in un fel de mâncare sau prin ordinea ser-
virii acestora. Primele cinci meniuri generate sunt, ı̂n această ordine: (aperitiv, legume,
paste, desert), (aperitiv, legume, supă, desert), (aperitiv, legume, supă, paste),
(aperitiv, paste, legume, desert), (aperitiv, paste, salată, desert). Indicat, i al
s, aselea meniu generat. (4p.)
a) (aperitiv, salată, paste, desert) c) (aperitiv, salată, supă, paste)
b) (aperitiv, salată, supă, desert) d) (aperitiv, supă, legume, desert)
4. Un graf orientat cu 5 vârfuri, numerotate de la 1 la 5, este reprezentat ı̂n figura de mai
jos. Indicat, i numărul de componente tare conexe ale grafului. (4p.)
a) 1
b) 2
c) 3
d) 4

5. Un arbore cu rădăcină are 20 de noduri, dintre care 10 noduri de tip ”frunză”. Indicat, i
numărul maxim de noduri care au acelas, i ”tată” ı̂n acest arbore. (4p.)
a) 5 b) 7 c) 10 d) 15

SUBIECTUL al II-lea (40 de puncte)


Scriet, i pe foaia de examen răspunsul pentru fiecare dintre cerint, ele următoare.
1. Se consideră algoritmul de mai jos, scris ı̂n pseudocod. S-a notat cu a%b restul ı̂mpărt, irii
numărului natural a la numărul natural nenul b s, i cu [c] partea ı̂ntreagă a numărului
real c.
citeste n ( numar natural nenul )
m←0
pentru i =1 , n executa
| citeste x ( numar natural )
| cat timp x %10 > [ x /10]%10 executa
| | _ x←[ x /10]
| _ m←m + x
daca m >0 atunci scrie m
| _altfel scrie " niciunul "

a) Scriet, i ce se afis, ează dacă se citesc, ı̂n această ordine, numerele 5, 127, 2019, 1005,
7, 1900. (6p.)
b) Dacă primul număr citit este 2, scriet, i un set de numere distincte din intervalul
[103 , 104 ) care pot fi citite ı̂n continuare astfel ı̂ncât, ı̂n urma executării algoritmului,
să se afis, eze mesajul niciunul. (6p.)
PROBLEME DE INFORMATICĂ PENTRU EXAMENE 83

c) Scriet, i programul C/C++ corespunzător algoritmului dat. (10p.)


d) Scriet, i ı̂n pseudocod un algoritm echivalent cu cel dat, ı̂nlocuind adecvat structura
pentru ... execută cu o structură repetitică de alt tip. (6p.)
2. Variabila c memorează date specifice unui cerc: coordonatele reale (abscisa s, i ordonata),
ı̂n planul xOy, ale centrului cercului, precum s, i lungimea razei acestuia. S, tiind că
expresiile C/C++: c.centru.x, c.centru.y, respectiv c.raza au ca valori numere reale
reprezentând datele specifice ale unui cerc, scriet, i definit, ia unei structuri cu eticheta
figura, care permite memorarea datelor precizate s, i declarat, i corespunzător variabila c.
(6p.)
3. Variabilele i s, i j sunt de tip ı̂ntreg, iar variabila m memorează un tablou bidimensional
cu 7 linii s, i 7 coloane, numerotate de la 0 la 6, cu elemente de tip char. Init, ial, fiecare
element memorează caracterul *. (6p.)
Fără a utiliza alte variabile, scriet, i a b c d e f g
secvent, a de instruct, iuni de mai jos, + a b c d e f
ı̂nlocuind punctele de suspensie astfel ı̂ncât, + + a b c d e
ı̂n urma executării secvent, ei obt, inute, va- + + + a b c d
riabila m să memoreze tabloul alăturat. + + + + a b c
+ + + + + a b
for ( i =0; i <7; i ++)
+ + + + + + a
for ( j =0; j <7; j ++)
............

SUBIECTUL al III-lea (30 de puncte)

1. Subprogramul multiplu are un singur parametru, n, prin care primes, te un număr natural
(n∈[1,104]). Subprogramul returnează cel mai mic multiplu nenul al lui n cu proprietatea
că este pătrat perfect. Scriet, i definit, ia completă a subprogramului.
Exemplu: dacă n=72 sau n=144, subprogramul returnează numărul 144. (10p.)
2. Numim citat ı̂ntr-un text o secvent, ă de caractere din acel text care ı̂ncepe cu un caracter
< s, i se termină cu un caracter >, celelalte caractere ale secvent, ei fiind diferite de < s, i >.
Un text de cel mult 100 de caractere (litere mici ale alfabetului englez, spat, ii s, i caracterele
< s, i >) cont, ine cel put, in un citat. Textul nu cont, ine alte caractere < s, i > decât cele care
mărginesc citatele, s, i oricare două citate nu au nici caractere < s, i > s, i nici alte caractere ı̂n
comun. Scriet, i un program C/C++ care cites, te de la tastatură un text de tipul precizat
s, i ı̂l transformă ı̂n memorie prin ı̂nlocuirea tuturor literelor mici cuprinse ı̂n citate cu
literele mari corespunzătoare, celelalte rămânând nemodificate, ca ı̂n exemplul de mai jos.
Programul afis, ează pe ecran textul obt, inut. (10p.)
Exemplu: pentru textul
mai bine sa fii un <om de valoare> decat un <om de succes>
se afis, ează
mai bine sa fii un <OM DE VALOARE> decat un <OM DE SUCCES>.
3. Se numes, te vârf ı̂ntr-un s, ir de numere naturale un termen al s, irului care este strict mai
mare decât fiecare dintre cei doi termeni vecini cu el, aflat, i ı̂n s, ir pe pozit, ia din stânga,
respectiv din dreapta sa. Fis, ierul bac.in cont, ine un s, ir de cel mult 106 numere naturale
din intervalul [0,109], separate prin câte un spat, iu. Se cere să se afis, eze pe ecran vârful
din s, irul aflat ı̂n fis, ier pentru care valoarea absolută a diferent, ei dintre cei doi vecini ai
săi este minimă. Dacă există mai multe astfel de numere, se afis, ează cel mai mare dintre
ele, iar dacă nu există niciun vârf, se afis, ează pe ecran mesajul nu exista. Proiectat, i un
84 PROBLEME DE INFORMATICĂ PENTRU EXAMENE

algoritm eficient din punctul de vedere al timpului de executare s, i al spat, iului de memorie
utilizat.
Exemplu: dacă fis, ierul cont, ine s, irul 2 7 10 5 6 2 1 3 20 17 9 11 7 3 10 6 2 se afis, ează
pe ecran 11.
a) Descriet, i ı̂n limbaj natural algoritmul proiectat, justificând eficient, a acestuia. (2p.)
b) Scriet, i programul C/C++ corespunzător algoritmului proiectat. (8p.)
PROBLEME DE MATEMATICĂ PENTRU CONCURSURI 85

PROBLEME DE MATEMATICĂ PENTRU


CONCURSURI

Rezolvarea problemelor pentru liceu din MATINF nr. 3

Clasa a IX-a

M 81. Fie s, irul (xn )n≥0 ⊂ [0, ∞) astfel ı̂ncât x0 = x1 = 0 s, i


1
x2n+2 = 3x2n+1 − x2n + 2
, ∀n ∈ N∗ .
n

1 1 1
Demonstrat, i că xn ≥ 1 + + + ... + , ∀n ∈ N, n ≥ 3.
2 3 n−2
Cristinel Mortici, Viforâta

Solut, ie. Demonstrăm prin induct, ie că xn+1 ≥ xn , ∀n ∈ N∗ , (1). Afirmat, ia este adevărată
pentru n = 1 (x2 ≥ 0) s, i n = 2 (x23 = 3x22 + 1, deci x3 ≥ x2 ). Presupunem că xk+1 ≥ xk , unde
1
k ≥ 2, s, i avem x2k+2 − x2k+1 = 2x2k+1 − x2k + 2 ≥ 0, deci xk+2 ≥ xk+1 .
k
Rezultă că xn ≥ x3 , pentru orice n ≥ 3. Cum x3 ≥ 1 (deoarece x23 = 3x22 + 1 ≥ 1), obt, inem
2
că xn ≥ , ∀n ≥ 3, (2). Folosind (1) s, i (2), pentru orice n ≥ 3 avem x2n+1 = 3x2n − x2n−1 +
n−1  ‹2
1 2 1 2 2 1 1 1
2
≥ 2xn + 2
≥ xn +xn · + 2
= xn + , deci xn+1 ≥ xn + ,
(n − 1) (n − 1) n − 1 (n − 1) n−1 n−1
1 n−1
P n−1
P 1
adică xn+1 − xn ≥ . Astfel pentru orice n ≥ 4 avem (xk+1 − xk ) ≥ , adică
n−1 k=3 k=3 k − 1
1 1 1
xn − x3 ≥ + + . . . + . Cum x3 ≥ 1, rezultă inegalitatea din enunt, .
2 3 n−2
M 82. Fie D, E s, i F punctele de intersect, ie ale cercului exı̂nscris triunghiului ABC cores-
punzător laturii BC cu (BC), (AB, respectiv (AC, iar P s, i Q punctele de intersect, ie ale acestui
P Q · EF
cerc cu (BF ), respectiv (CE). Arătat, i că = 3.
P E · QF

Miguel Ochoa Sanchez, Peru

Solut, ie. Conform Teoremei lui Ptolemeu pentru patrulaterul inscriptibil BEF C avem
P F · EQ
P Q · EF + P E · QF = P F · EQ, deci este suficient să arătăm că = 4.
P E · QF
A
Cu notat, iile uzuale din 4ABC, avem AE = AF = p, deci EF = 2p sin s, i BF 2 =
2
p−c BP PE (p − c)2
p2 + c2 − 2pc cos A. Din 4BEP ∼ 4BF E avem = = , deci BP =
BF p−c A BF
2p sin
2
86 PROBLEME DE MATEMATICĂ PENTRU CONCURSURI

A A A
2p · BP · sin 2p(p − c) sin 2p(p − b) sin
s, i P E = 2 = 2 . Analog, QF = 2 . Prin urmare, avem
p−c BF CE
2 A 2 A
(p − c) 2
2pc(1 − cos A) 4pc sin 4pb sin
P F = BF − BP = BF − = = 2 . Analog, EQ = 2.
BF BF BF CE
2 A 2 A
4pc sin 4pb sin A
2 · 2 4bc sin2
P F · EQ BF CE 2 = 4.
Astfel, obt, inem = =
P E · QF A A (p − b)(p − c)
2p(p − c) sin 2p(p − b) sin
2 · 2
BF CE
M 83. Fie n ∈ N, n ≥ 2 s, i a1 , a2 , . . . , an ≥ 0. Demonstrat, i că
Í
n n n
X X √ X √
ak ≥ n−1· ai .
i=1 k=1 i=1
k6=i

Daniel Jinga, Pites, ti


√ √
Solut, ie. Inegalitatea dată se√scrie, desfăs, urat, a2 + a3 + .. . + an + a1 + a3 + . . . + an +
√ √ √ √
. . . + a1 + a2 + . . . + an−1 ≥ n − 1 a1 + a2 + . . . + an . Aplicând Inegalitatea Cauchy-
√ √ √ 2
Buniakowski-Schwarz avem (n − 1) (a2 + a3 + . . . + an ) ≥ a2 + a3 + . . . + an , deci
√ √ √ √ √
(n − 1) a2 + a3 + . . . + an ≥ n − 1 ( a2 + a3 + . . . + an ) .
Scriind ı̂ncă n − 1 relat, ii analoage s, i adunând rezultă inegalitatea dorită.
M 84. Demonstrat, i că pentru orice n ∈ N, n ≥ 2 are loc egalitatea

π 3π 5π (2n−1 − 1) π 2
sin n sin n sin n · . . . · sin n
= 2n−2 .
2 2 2 2 2
Ionel Tudor, Călugăreni
n−2
2Q (2k − 1)π
Solut, ie. Notăm cu Pn produsul din membrul stâng, adică Pn = sin . Avem
k=1 2n
n−1 n−2 n−1
2Q (2k − 1)π 2Q (2k − 1)π 2Q (2j − 1)π
Pn+1 = sin n+1
= sin n+1
· sin . Inversând ordinea facto-
k=1 2 k=1 2 j=2n−2 +1 2n+1
n−2
n−1
2Q (2k − 1)π
rilor ı̂n al doilea produs, adică notând 2 − j + 1 = k, rezultă că Pn+1 = sin ·
2n+1
n−2  ‹ k=1
2Q (2n − 2k + 1)π (2n − 2k + 1)π π (2k − 1)π (2k − 1)π
sin n+1
. Cum sin n+1
= sin − n+1
= cos , re-
2 2 2 2 2n+1
n−2  ‹ n−2  ‹
k=1
2Q (2k − 1)π (2k − 1)π 2Q 1 (2k − 1)π
zultă că Pn+1 = sin cos = · sin , prin urmare
k=1 2n+1 2n+1 k=1 2 2n √
1 2
Pn+1 = 2n−2 · Pn . Folosind această relat, ie, prin induct, ie rezultă us, or că Pn = 2n−2 , pentru
2 √ √ √ 2
π 2 2 2
orice n ≥ 2. Într-adevăr, P2 = sin = = 20 , iar presupunând că Pn = 2n−2 , unde n ≥ 2,
2√ 2 √2 2
1 1 2 2
avem Pn+1 = 2n−2 · Pn = 2n−2 · 2n−2 = 2n−1 , ceea ce ı̂ncheie demonstrat, ia.
2 2 2 2
PROBLEME DE MATEMATICĂ PENTRU CONCURSURI 87

M 85. Demonstrat, i că ı̂n orice triunghi neobtuzunghic ABC are loc inegalitatea
√ √ €p p p Š2 √
(3 3 − 4)(ctg A + ctg B + ctg C) + (2 − 3) ctg A + ctg B + ctg C ≥ 2 3.

În ce triunghiuri inegalitatea devine egalitate?

Leonard Giugiuc, Drobeta Turnu Severin


√ √ √
Solut, ie. Notăm ctg A = x, ctg B = y s, i ctg √ C = z. Avem x, y, z ≥ 0 s, i x2√y 2 +y 2 z 2 +z 2 x2 =
2 2 2 2
p inegalitatea din enunt, se poate scrie ca (3 3 − 4)(x + y + z ) + (2 − 3)(x + y + z) ≥
1.√Deci
2 3 x2 y 2 + y 2 z 2 + z 2 x2 , (1). Vom demonstra că inegalitatea (1) este adevărată pentru orice
numere x, y, z ≥ 0. Remarcăm că x2 y 2 + y 2 z 2 + z 2 x2 = (xy + yz + zx)2 − 2xyz(x + y + z). Deci
dacă fixăm pe x + y + z s, i xy + yz + zx, atunci membrul drept din (1) este descrescător ı̂n funct, ie
de variabila xyz. Conform Teoremei 3 de la pag. 41 din RMGO 1/2018 (http://rmgo.upit.ro),
este suficient să verificăm că (1) este adevărată ı̂n următoarele două cazuri:
√ 2
√ 2
√ Cazul
√ 1. y = z = 1 s, i x ∈ [0, 1]. Avem de arătat
√ că (3 32− 4)(x + 2) + (2 − 3)(x + 2) ≥
2 3 2x2 + 1, inegalitate echivalentă cu x(x + 2 3)(x − 1) ≥ 0, adevărat. Egalitatea are loc
pentru x = 0 sau x = 1.
√ √ √
Cazul 2. z = 0. Avem de arătat că (3 3−4)(x2 +y √ 2
)+(2− 3)(x+y)2 ≥ 2 3xy, √ inegalitate
2 2
ce√se obt, ine folosind Inegalitatea
√ mediilor
√ astfel: (3 3 − 4)(x + y ) + (2 − 3)(x + y)2 ≥
(3 3 − 4) · 2xy + (2 − 3) · 4xy = 2 3xy. Egalitatea are loc pentru x = y.
Demonstrat, ia inegalităt, ii (1) este ı̂ncheiată. Cazurile de egalitate ı̂n (1) sunt (x, x, x), (x, x, 0),
(x, 0, x) s, i (0, x, x), deci inegalitatea din enunt, devine egalitate pentru triunghiurile echilaterale
sau dreptunghice isoscele.

Clasa a X-a

M 86. Arătat, i că pentru orice a, b, c > 0 are loc inegalitatea


‚É É É Œ  ‹
a 3 b c a3 b 3 c 3
9 3
+ + 3
− 3 + 3+ 3 ≤ 24.
b c a b c a

Sorin Ulmeanu, Pites, ti


É É É
a b c
Solut, ie (Daniel Văcaru, Pites, ti). Notând 3 = x, 3 = y s, i 3 = z, inegalitatea din enunt,
b c a
devine 9(x + y + z) − (x9 + y 9 + z 9 ) ≤ 24. Dar, folosind Inegalitatea mediilor, avem x9 + 8 ≥ 9x,
y 9 + 8 ≥ 9y, z 9 + 8 ≥ 9z s, i prin adunare obt, inem inegalitatea dorită.

M 87. Fie a, b ∈ (1, ∞) astfel ı̂ncât ab = 4. Arătat, i că


1 1
+ < 1.
2 loga (a + 1) − 1 2 logb (b + 1) − 1

Dinu Teodorescu, Târgovis, te


88 PROBLEME DE MATEMATICĂ PENTRU CONCURSURI

√ 1 1
Solut, ie. Avem 2 loga (a + 1) > 2 loga (2 a) = 1 + 2 loga 2, deci < =
2 loga (a + 1) − 1 2 loga 2
log2 a 1 1 log2 a + log2 b log2 (ab) log2 4
. Astfel + < = = = 1.
2 2 loga (a + 1) − 1 2 logb (b + 1) − 1 2 2 2

M 88. Fie n ∈ N, n ≥ 2. Rezolvat, i ı̂n C ecuat, ia z n + |z| = 2.

Daniel Jinga, Pites, ti

Solut, ia 1. Scriind ecuat, ia sub forma z n = 2 − |z|, trecând la modul obt, inem |z|n = 2 − |z| .
Dacă |z| ≤ 2, rezultă că |z|n = 2 − |z|, adică |z|n − 1 + |z| − 1 = 0, sau, echivalent,
(|z| − 1) (|z|n−1 + . . . + |z| + 1) = 0. Cum |z| ≥ 0, rezultă că |z| = 1 s, i ecuat, ia dată devine
z n = 1, deci z ∈ Un .
Dacă |z| > 2, rezultă că |z|n = |z| − 2, adică |z| (|z|n−1 − 1) + 2 = 0, fals.
În concluzie, mult, imea solut, iilor ecuat, iei date este S = Un .
Solut, ia 2 (Daniel Văcaru, Pites, ti). Cu forma trigonometrică z = r(cos t + i sin t), unde r ≥ 0 s, i
t ∈ [0, 2π), ecuat, ia devine rn cos nt + irn sin nt + r = 2, deci rn cos nt + r = 2 s, i rn sin nt = 0.
r = 0 nu convine, deci sin nt = 0, de unde cos nt = ±1. Obt, inem rn + r = 2, (1), sau
−rn + r = 2, (2). Din monotonia funct, iei r → rn + r, (1) are solut, ia unică r = 1. De asemenea,
(2) se transformă ı̂n rn − r + 2 = 0. Această ecuat, ie nu are solut, ii, deoarece pentru r < 1 avem
rn − r + 2 = rn + (1 − r) + 1 > 1, iar pentru r ≥ 1 avem rn − r + 2 = r (rn−1 − 1) + 2 ≥ 2. Am
2kπ 2kπ 2kπ
obt, inut că r = 1 s, i cos nt = 1. Astfel t = cu k ∈ {0, 1, . . . , n−1}, iar z = cos +i sin .
n n n
M 89. Un număr natural nenul n se numes, te interesant dacă 14 + 24 + . . . + n4 se divide cu
12 + 22 + . . . + n2 . Demonstrat, i că dintre orice cinci numere naturale nenule consecutive se pot
alege două interesante.

Cristinel Mortici, Viforâta

n n (n + 1) (2n + 1)
Solut, ie (Daniel Văcaru, Pites, ti). Fie n ∈ N∗ . Utilizând formulele k2 =
P
k=1 6
n n (n + 1) (2n + 1) (3n2 + 3n − 1)
k4 =
P
s, i , rezultă că n este interesant dacă s, i numai dacă
k=1 30
3n2 + 3n − 1 se divide cu 5. Astfel avem echivalent, ele: n este interesant ⇔ 3n(n + 1) = M5 + 1
⇔ n(n + 1) = M5 + 2 ⇔ n = M5 + 1 sau n = M5 + 3.
Rezultă că printre orice cinci numere naturale nenule consecutive există exact două interesante,
unul de forma M5 + 1 s, i unul de forma M5 + 3.

M 90. Arătat, i că ı̂n orice triunghi ABC are loc inegalitatea
Ê Ê Ê
2ab 2bc 2ca
+ + ≥ 3.
c(a + b) a(b + c) b(c + a)

Leonard Giugiuc, Drobeta Turnu Severin


PROBLEME DE MATEMATICĂ PENTRU CONCURSURI 89

Solut, ie. Vom utiliza următoarele două rezultate.


Lema 1 (Rădulescu - Maftei, Gazeta Matematică; o generalizare recentă obt, inută de L. Giugiuc
este Problema 485 din Gazeta Matematică Seria É A, nr. 3-4/2018). Dacă x, yÉs, i z sunt lungimile
É
y+z−x z+x−y x+y−z
laturilor unui triunghi ascut, itunghic, atunci + + ≥ 3.
x y z
1 1 1 1
Lema 2. Dacă a, b s, i c sunt lungimile laturilor unui triunghi, atunci numerele + , + s, i
a b b c
1 1
+ sunt lungimile laturilor unui triunghi ascut, itunghic.
c a  ‹  ‹  ‹
1 1 2 1 1 2 1 1 2
Demonstrat, ie: Este suficient să arătăm că + + + > + , inegalitate
b c c a a b
1 a+b−c
echivalentă cu 2 + > 0, adevărat.
c abc
Revenind acum la problema dată, concluzia se obt, ine imediat aplicând Lema 1 pentru
1 1 1 1 1 1
x = + , y = + s, i z = + .
b c c a a b

Clasa a XI-a

 ‹
2019 1 2 3 4 5 6 7 8
M 91. Câte solut, ii are ecuat, ia x = , x ∈ S8 ?
2 3 4 5 6 7 8 1

Stelian Corneliu Andronescu s, i Costel Bălcău, Pites, ti

Solut, ie. Notând cu a permutarea din membrul drept, din x2019 = a rezultă ax = xa, de
unde, deoarece permutarea a este un ciclu, deci are ordinul ord (a) = 8, rezultă că x = ak cu
k ∈ {0, 1, . . . , 7} (ı̂ntr-adevăr, dacă x(1) = k + 1, cu k ∈ {0, 1, . . . , 7}, atunci x(1) = ak (1), deci
x(2) = x(a(1)) = a(x(1)) = ak+1 (1) = ak (2) s, i, analog, x(3) = ak (3), . . . , x(8) = ak (8), deci
x = ak , care verifică ax = xa). Astfel ecuat, ia devine a2019k = a, care, deoarece ord (a) = 8, este
echivalentă cu 2019k = M8 + 1. Cum 2019 = M8 + 3, această ecuat , ia devine 3k = M8 + 1, cu‹
3 1 2 3 4 5 6 7 8
solut, ia k = 3. Rezultă că ecuat, ia dată are solut, ia unică x = a = .
4 5 6 7 8 1 2 3

M 92. Fie A, B ∈ M3 (C) cu proprietatea că A2 + B 2 = 2AB. Arătat, i că

det(A + B)2 = 8 det(A2 + B 2 ).

Daniel Jinga, Pites, ti

Solut, ie. Fie f : C → C, f (x) = det (A2 + B 2 + x(AB − BA)) = x3 det(AB − BA) +
ax2 + bx + det(A2 + B 2 ), unde a, b ∈ C. Avem f (i) = det (A2 + B 2 + iAB − iBA) =
det ((A − iB)(A + iB)) s, i f (−i) = det (A2 + B 2 − iAB + iBA) = det ((A + iB)(A − iB)). Re-
zultă că f (i) = f (−i), adică i (b − det(AB − BA))−a+det(A2 +B 2 ) = −i (b − det(AB − BA))−
a + det(A2 + B 2 ), deci b = det(AB − BA) s, i astfel f (x) = bx3 + ax2 + bx + det(A2 + B 2 ). Dar
f (0) = det(A2 + B 2 ) = det(2AB) iar f (−2) = det(A2 + B 2 − 2AB + 2BA) = det(2BA). Rezultă
90 PROBLEME DE MATEMATICĂ PENTRU CONCURSURI

5b
că f (0) = f (−2), adică det(A2 + B 2 ) = −10b + 4a + det(A2 + B 2 ), deci a = s, i astfel f (x) =
 ‹ 2
35bx2 1 b 5b b
bx + + bx + det(A2 + B 2 ). Atunci f − = − + − + det(A2 + B 2 ) = det(A2 + B 2 ).
2  ‹  2 8 8 ‹2  ‹
1 2 2 1 1 1 1
Pe de altă parte, f − = det A + B − AB + BA = det 2AB − AB + BA =
 ‹ 2 2 2 2 2
3 1 1 1 1
det AB + BA = det(3AB + BA) = det(2AB + AB + BA) = det(A2 + B 2 + AB +
2 2 8 8 8
1
BA) = det(A + B)2 s, i astfel problema este rezolvată.
8
M 93. Fie (xn )n≥1 un s, ir de numere reale pozitive astfel ı̂ncât s, irul (n2 xn + nx3n )n≥1 este
mărginit superior. Arătat, i că s, irul (yn )n≥1 definit prin yn = n3 x2n + n2 x4n este convergent s, i
calculat, i limita sa.

Dinu Teodorescu, Târgovis, te

Solut, ie. Deoarece s, irul (n2 xn + nx3n )n≥1 este mărginit superior s, i n2 xn , nx3n ≥ 0 pentru orice
1
n ≥ 1, rezultă că s, irul (n2 xn )n≥1 este mărginit, deci nxn = · (n2 xn ) → 0 (cazul 0 × mărginit).
n
Analog, yn = nxn (n2 xn + nx3n ) converge către 0.
(1 + π e ) (2 + π e ) · . . . · (n + π e )
M 94. Calculat, i lim .
n→∞ (1 + eπ ) (2 + eπ ) · . . . · (n + eπ )

Florică Anastase, Lehliu Gară

Solut, ie. Notăm cu xn s, irul din enunt, . Folosind inegalităt


n e n
‹ ln x ≤ x − 1 pentru
 , ile ecunoscute n
orice
k + π k + π 1
x > 0 s, i eπ > π e , avem ln xn = − 1 = − (eπ − π e )
P P P
ln π
≤ π π
=
k=1 k + e k=1 k + e k=1 k + e
eπ − π e P n 1 + eπ eπ − π e Pn 1
− ≤ − → −∞, deci lim ln xn = −∞, prin urmare lim xn = 0.
1 + eπ k=1 k + eπ 1 + eπ k=1 k n→∞ n→∞

M 95. Pentru ce valori reale pozitive ale lui k s, i t inegalitatea


Ê
a2 + b 2 a + b |a − b|t
− ≥k·
2 2 (a + b)t−1
are loc pentru orice numere reale pozitive a s, i b?

Leonard Giugiuc, România s, i Tran Hong, elev, Vietnam

Solut, ie. Putem presupune, fără a restrânge generalitatea, că a + b = 2 (ı̂nmult, ind inegalitatea
2 2a 2b
dată cu s, i notând a0 = , b0 = se obt, ine aceeas, i inegalitate, dar ı̂n variabilele
a+b É a2+ b 2 a+b
a +b √ √
a0 , b0 s, i cu a0 + b0 = 2). Fie

= x, deci x ∈ [1, 2). Mai mult, {a, b} = 1 ± x2 − 1 ,
√ 2 t √
deci |a − b| = 2 x2 − 1. Inegalitatea dată se rescrie ca x − 1 ≥ 2k (x2 − 1) 2 , ∀x ∈ [1, 2), √(1).
2−t t
Presupunem prin reducere la absurd că t < 2. Din (1) avem (x−1) 2 ≥ 2k(x+1) 2 , ∀x ∈ (1, 2),
2−t t t
deci 0 = lim (x − 1) 2 ≥ lim 2k(x + 1) 2 = 2k · 2 2 , fals. Rezultă că t ≥ 2. În altă ordine de idei,
x&1 x&1

√ 2
t 2−1
tot din (1) avem 2 − 1 = lim √ (x − 1) ≥ lim √ 2k (x − 1) = 2k, deci k ≤
2
.
x% 2 x% 2 2
PROBLEME DE MATEMATICĂ PENTRU CONCURSURI 91

 √ ˜
2−1
Reciproc, demonstrăm că (1) are loc pentru orice k ∈ 0, s, i orice t ≥ 2. Este
√ 2
t−2 t
suficient să arătăm că 1 ≥ 2k(x − 1) 2 (x + 1) 2 , ∀x ∈ (1, 2). Evident, funct, ia €√f (x) =Š
t−2 t √
2k(x − 1) 2 (x + 1) 2 este strict crescătoare pe (1, 2), deci f (x) < lim√ f (x) = 2k 2 + 1 ,
x% 2

√ €√ Š 2 − 1 €√ Š
∀x ∈ (1, 2). Dar 2k 2+1 ≤2· · 2 + 1 = 1. Demonstrat, ia este completă. În
 √ ˜ 2
2−1
concluzie, k ∈ 0, s, i t ≥ 2.
2

Clasa a XII-a

M 96. Pe mult, imea numerelor reale se consideră legea de compozit, ie


Ê Ê
y2 + y + 1 x2 + x + 1 1
x ∗ y = (2x + 1) + (2y + 1) − , ∀x, y ∈ R.
3 3 2

a) Demonstrat, i că (R, ∗) este un grup abelian izomorf cu grupul (R, +).
b) Arătat, i că |0 ∗ 0 ∗{z. . . ∗ 0} ∈ Q.
de 2019 ori 0

Stelian Corneliu Andronescu s, i Costel Bălcău, Pites, ti


(ex − e−x ) 3 − 2
Solut, ie. a) Considerăm funct, ia f : R → R, f (x) = . Avem f (x) ∗ f (y) =
s– √ 2™ s – √ 24
™ √
ex − e−x (ey − e−y ) 3 3 ey − e−y (ex − e−x ) 3 3 1 ex − e−x 3
+ + + − = · ·
2 4 4 2 4 4 2 2 4
√ √
−y ey − e−y 3 −x 1 3 x+y 1
y
(e + e ) + · x
· (e + e ) − = (e − e−x−y ) − = f (x + y), ∀x, y ∈ R.
2 4 2 4 2p
2y + 1 + 2 y 2 + y + 1
Pentru orice y ∈ R ecuat, ia f (x) = y are solut, ia unică x = ln √ , deci f este
√ 3
−1 −1 2x + 1 + 2 x2 + x + 1
bijectivă, cu inversa f : R → R, f (x) = ln √ . Cum (R, +) este grup
3
abelian, rezultă că s, i (R, ∗) este grup abelian, iar f este un izomorfism de la (R, +) la (R, ∗).

b) Avem f −1 0| ∗ 0 ∗{z. . . ∗ 0} = 2019f −1 (0) = 2019 ln 3, deci


de 2019 ori 0

h€√ Š2019 €√ Š−2019 i √


√ 3 − 3 3−2 31010 − 3−1009 − 2
0| ∗ 0 ∗{z. . . ∗ 0} = f (2019 ln 3) = = ∈ Q.
4 4
de 2019 ori 0
92 PROBLEME DE MATEMATICĂ PENTRU CONCURSURI

M 97. Fie a ∈ R∗ , b ∈ R s, i legea de compozit, ie ◦ : R × R → R,

x ◦ y = axy − ab(x + y) + b(1 + ab), ∀x, y ∈ R.

Pentru orice n ∈ N∗ , fie xn solut, ia ecuat, iei x p


| ◦ x ◦{z. . . ◦ x} = a + b, unde p este un număr natural
de 2n+1 ori x
fixat. Calculat, i lim xn .
n→∞

Marin Chirciu, Pites, ti

Solut, ie (Daniel Văcaru, Pites, ti). Se observă că x ◦ y = a (x − b) (y − b) + b s, i, prin induct, ie,
2n+1
2n
| ◦ x ◦{z. . . ◦ x} = a (x − b)
se arată că x + b. Se obt, ine ecuat, ia a2n (xn − b)2n+1 + b = ap + b,
de 2n+1 ori x

2n+1 1
având solut, ia xn = b + ap−2n . Trecând la limită, găsim lim xn = b + .
n→∞ a
M 98. Determinat, i cel mai mare număr real k pentru care inegalitatea

(a2 + k)(b2 + k)(c2 + k) ≤ (1 + k)3

are loc pentru orice a, b, c ∈ [0, ∞) astfel ı̂ncât a + b + c = 3.

Leonard Giugiuc, Drobeta Turnu Severin s, i Costel Bălcău, Pites, ti

Solut, ie. Considerând funct, ia f : [0, 3] → R, f (x) = ln(x2 + k), inegalitatea din enunt, este
2(k − x2 )
evident echivalentă cu f (a) + f (b) + f (c) ≤ 3f (1). Avem f 00 (x) = 2 .
(x + k)2

Cazul
√ 1. k > 1. Atunci √ f este strict convexă pe [0, 3] ∩ [0, k]. Pentru k ≤ 4 luăm
a = k, b = 1 s, i c = 2 − k, √ iar pentru k > 4 luăm a = 2, b = 1 s, i c = 0. În ambele
situat, ii avem a, b, c ∈ [0,3] ∩ [0, k]
‹ s, i a + b + c = 3. Conform Inegalităt, ii lui Jensen avem
a+b+c
f (a) + f (b) + f (c) > 3f = 3f (1), deci orice k > 1 nu verifică proprietatea dată.
3
Cazul 2. k = 1. Atunci luând a = 3 s, i b = c = 0 avem f (a) + f (b) + f (c) = ln 10 > 3 ln 2 =
3f (1), deci nici k = 1 nu verifică proprietatea dată.
Cazul 3. k ∈ (0, 1). Atunci f este strict concavă pe [1, 3]. Vom utiliza următorul rezultat ce
extinde Inegalitatea lui Jensen:
Teorema funct, iilor semi-convexe (Vasile Cı̂rtoaje, 2004) (Teorema de la pag. 3 din Mathema-
tical inequalities, Volumul 4: Extensions and refinements of Jensen’s inequality, Editura Univer-
sităt, ii Petrol-Gaze din Ploies, ti, 2018; http://ac.upg-ploiesti.ro/vcirtoaje/math_ineq_4.pdf)
Fie I ⊆ R un interval, s un punct interior al lui I s, i f : I → R o funct, ie convexă
 a + pe I ∩ (−∞, s]
1 a2 + . . . + an 
sau pe I ∩ [s, +∞). Atunci inegalitatea f (a1 ) + f (a2 ) + . . . + f (an ) ≥ nf
n
are loc pentru orice a1 , a2 , . . . , an ∈ I a.ı̂. a1 + a2 + . . . + an = ns dacă s, i numai dacă
f (x) + (n − 1)f (y) ≥ nf (s) pentru orice x, y ∈ I a.ı̂. x + (n − 1)y = ns.
Aplicând această teoremă (pentru funct, ia −f ), inegalitatea f (a) + f (b) + f (c) ≤ 3f (1) are loc
• a,˜b, c ∈ [0, ∞) cu a + b + c = 3 dacă s, i numai dacă 2f (t) + f (3 − 2t) ≤ 3f (1) pentru
pentru orice
3
orice t ∈ 0, . Astfel k verifică proprietatea din enunt, dacă s, i numai dacă g(t) ≤ g(1) pentru
2
PROBLEME DE MATEMATICĂ PENTRU CONCURSURI 93

• ˜
3
orice t ∈ 0, , unde g(t) = (t2 +k)2 [(3 − 2t)2 + k]. Avem g 0 (t) = 12(t−1)(2t2 −3t+k)(t2 +k) =
2 √ √
2 3 − 9 − 8k 3 + 9 − 8k
24(t − 1)(t − t1 )(t − t2 )(t + k), unde t1 = s, i t2 = . Cum 0 < t1 <
§ 4 ª 4
3  3
1 < t2 < , deducem că max3 g(t) ∈ g(0), g(1), g . Deci k verifică proprietatea din enunt,
2 2
t∈[0, 2 ]
 ‹
3
dacă s, i numai dacă g(0) ≤ g(1) s, i g ≤ g(1), adică 6k 2 − 3k − 1 ≤ 0 s, i 24k 2 + 33k − 16 ≤ 0,
2
 √ ˜  √ ˜  √ ˜
33 + 3 5 105 − 33 5 105 − 33
cu k ∈ (0, 1), adică k ∈ 0, ∩ 0, = 0, .
12 48 48

5 105 − 33
În concluzie, cea mai mare valoare a lui k cu proprietatea din enunt, este k = .
48
M 99. Se consideră funct, ia f : [0, ∞) → [0, 1) astfel ı̂ncât
 È   È 
ln 1 + f (x) = x + ln 1 − f (x) , ∀x ∈ [0, ∞).

a) Demonstrat, i că ecuat, ia funct, ională are solut, ie.


b) Arătat, i că f admite primitive.
Z ab−1 Z b1−a √
1+ x
c) Fie a, b ∈ (0, 1) cu a < b. Arătat, i că f (x) dx + ln √ dx > 1.
0 0 1− x

Florică Anastase, Lehliu Gară


p
1 + f (x)
Solut, ie (Daniel Văcaru, Pites, ti). a) Putem scrie, echivalent, p = ex , deci
1 − f (x)
 ‹2
ex − 1
f (x) = ∈ [0, 1), ∀x ∈ [0, ∞).
ex + 1

b) Evident, f este continuă, deci admite primitive.


1−x
ln x + ln x
c) Fie funct, ia auxiliară g : (0, 1) → R, g(x) = , având derivata g 0 (x) = x .
1−x (1 − x)2
1−x x−1
Fie h : (0, 1] → R, h(x) = + ln x. Avem h0 (x) = < 0 pentru orice x ∈ (0, 1),
x x2
deci h este strict descrescătoare. Pentru orice x ∈ (0, 1) avem h(x) > h(1) = 0, prin urmare
ln a ln b
g 0 (x) > 0, deci g este strict crescătoare. Astfel avem implicat, iile a < b ⇒ < ⇒
1−a 1−b
a1−b < b1−a ⇒ 1 < b1−a ab−1 . Cum √ funct, ia f este strict crescătoare, bijectivă, cu inversa
1+ x
f : [0, 1) → [0, ∞), f −1 (x) = ln √ s, i f (0) = 0, conform Inegalităt, ii lui Young obt, inem
1− x
Z ab−1 Z b1−a √
1+ x
f (x) dx + ln √ dx ≥ ab−1 b1−a > 1.
0 0 1− x
94 PROBLEME DE MATEMATICĂ PENTRU CONCURSURI

M 100. Fie f, g : [a, b] → R două funct, ii derivabile cu derivatele continue astfel ı̂ncât
f 2 (x) + g 2 (x) 6= 0, oricare ar fi x ∈ [a, b]. Demonstrat, i că
s Ê
(f 0 (x))2 + (g 0 (x))2
Z b
f 2 (b) + g 2 (b)
dx ≥ ln .
a f 2 (x) + g 2 (x) f 2 (a) + g 2 (a)

Cristinel Mortici, Viforâta

Solut, ie (Alexandru Daniel Pı̂rvuceanu, elev, Drobeta Turnu Severin; Leonard Giugiuc, Drobeta
Turnu Severin). Conform Inegalităt, ii Cauchy-Buniakowski-Schwarz avem
È
[(f 0 (x))2 + (g 0 (x))2 ] · [f 2 (x) + g 2 (x)] ≥ f (x)f 0 (x) + g(x)g 0 (x), ∀x ∈ [a, b].

Împărt, ind această inegalitate cu f 2 (x) + g 2 (x) > 0, ∀x ∈ [a, b], obt, inem
Ê
(f 0 (x))2 + (g 0 (x))2 f (x)f 0 (x) + g(x)g 0 (x) 1 [f 2 (x) + g 2 (x)]0
≥ = · , ∀x ∈ [a, b].
f 2 (x) + g 2 (x) f 2 (x) + g 2 (x) 2 f 2 (x) + g 2 (x)

Prin integrarea ultimei inegalităt, i pe [a, b] obt, inem


Z bÊ b
Ê
(f 0 (x))2 + (g 0 (x))2 1 f 2 (b) + g 2 (b)
dx ≥ ln(f 2 (x) + g 2 (x)) = ln .
a f 2 (x) + g 2 (x) 2 a f 2 (a) + g 2 (a)
PROBLEME DE MATEMATICĂ PENTRU CONCURSURI 95

Probleme propuse pentru liceu

Clasa a IX-a

M 121. Demonstrat, i că oricum s-ar considera patru puncte ı̂n interiorul sau pe conturul
poligonului OABCDE, unde O(0, 0), A(4, 0), B(4, 2), C(2, 2), D(2, 4) s, i E(0, 4), se pot alege
două, notate M1 (x1 , y1 ) s, i M2 (x2 , y2 ), astfel ı̂ncât |x1 − x2 | + |y1 − y2 | ≤ 4.
Cristinel Mortici, Viforâta
M 122. Fie a, b, c, x, y, z numere reale strict pozitive. Arătat, i că
1 1 1 a+b+c
x y z + y z x + z x y ≤ x + y + z.
+ + + + + +
a b c a b c a b c
Când are loc egalitatea?
Dorin Mărghidanu, Corabia
M 123. Arătat, i că pentru orice n, x, y, z > 0 are loc inegalitatea
n(n + 3)(x + y) + 3 n(n + 3)(y + z) + 3 n(n + 3)(z + x) + 3 27
+ + ≥ 6n + .
x + y + (n + 1)z y + z + (n + 1)x z + x + (n + 1)y (n + 3)(x + y + z)
Daniel Jinga, Pites, ti
M 124. Fie n ∈ N, n ≥ 2 s, i a1 , a2 , . . . , an ∈ R astfel ı̂ncât 0 < a1 ≤ a2 ≤ . . . ≤ an . Arătat, i că
X 3a2i − 2ai aj + 3a2j  ‹
n(n − 1) 3n(n − 1) a1 an 2
≤ ≤ + − .
2 1≤i<j≤n
(ai + aj )2 8 an a1 3

Sorin Ulmeanu, Pites, ti s, i Nicolae Papacu, Slobozia


M 125. Se consideră un cerc de centru O s, i rază R, A un punct exterior cercului s, i B, C, D, E
patru puncte distincte situate pe cerc astfel ı̂ncât B ∈ (OA), C ∈ (AD), AC = R s, i O ∈ (DE).
Notăm cu x lungimea segmentului AB.
a) Arătat, i că 2m (^BCE) = 3m (^BAD).
R
b) Calculat, i astfel ı̂ncât 4ADE să fie echilateral.
x
R
c) Calculat, i astfel ı̂ncât [AB] ≡ [CD].
x
Thanos Kalogerakis, Grecia
M 126. Demonstrat, i că ı̂n orice triunghi ABC avem:
(1 + ab)(a + b) (1 + bc)(b + c) (1 + ca)(c + a) p2 + r2 − 2Rr
a) + + ≤ ;
(1 + a2 )(1 + b2 ) (1 + b2 )(1 + c2 ) (1 + c2 )(1 + a2 ) 4Rr
(1 + ra rb )(ra + rb ) (1 + rb rc )(rb + rc ) (1 + rc ra )(rc + ra ) 2R − r
b) 2 2
+ 2 2
+ 2 2
≤ .
(1 + ra )(1 + rb ) (1 + rb )(1 + rc ) (1 + rc )(1 + ra ) r
Mihály Bencze, Bras, ov
96 PROBLEME DE MATEMATICĂ PENTRU CONCURSURI

Clasa a X-a

M 127. Fie k un număr real fixat. Determinat, i mult, imea

Mk = 2 b2 + c2 − a2 a ≥ b ≥ c ≥ 0, a + b + c = 6, a2 + b2 + c2 = k .
 

Leonard Mihai Giugiuc, Drobeta Turnu Severin


M 128. Arătat, i că log2 3 + log4 5 + log6 7 + . . . + log14 15 > 7 7 2.

Dorin Mărghidanu, Corabia

n
X (3k 3 + 9k 2 + 11k + 4) C2k+1
k
M 129. Calculat, i suma , unde n ∈ N∗ .
k=1
k+2

Mihály Bencze, Bras, ov

M 130. Fie z1 , z2 , . . . , z12 numere complexe distincte două câte două s, i având modulele egale.
Dacă |z1 − z2 | = |z4 − z5 | = |z7 − z8 | = |z10 − z11 |, |z2 − z3 | = |z5 − z6 | = |z8 − z9 | = |z11 − z12 |,
|z3 − z4 | = |z6 − z7 | = |z9 − z10 | = |z12 − z1 | s, i punctele de afixe z1 , z2 , . . . , z12 sunt, ı̂n această
ordine, vârfurile unui poligon convex, atunci arătat, i că z1 + z2 + . . . + z12 = 0.

Sorin Ulmeanu, Pites, ti

M 131. Rezolvat, i ı̂n mult, imea numerelor naturale ecuat, ia

π 3π √
n · ctg 2 + n · ctg 2 = 10 10n.
n n

Ionel Tudor, Călugăreni

M 132. Fie ABC un triunghi, I centrul cercului ı̂nscris ı̂n triunghiul ABC, iar D, E s, i F
punctele de intersect, ie ale acestui cerc cu segmentele (IA), (IB), respectiv (IC). Arătat, i că:
AABC PABC
a) ≥ 4; b) ≥ 2.
ADEF PDEF

Sorin Borodi, Dej


PROBLEME DE MATEMATICĂ PENTRU CONCURSURI 97

Clasa a XI-a

M 133. Fie a, b, c, d ∈ C s, i A ∈ Mn (C) astfel ı̂ncât A3 = (a − 1)A2 + (a + b)A + bIn s, i


det (A + In ) 6= 0. Calculat, i det (A4 − 2aA3 + (a2 + c + d)A2 − a(c + d)A + cdIn ).

Marin Chirciu, Pites, ti

M 134. Fie p, q, n ∈ N∗ astfel ı̂ncât p < q ≤ n + 1 s, i fie a1 , a2 , . . . , an ∈ R \ {0, 1} numere


n n aq+k − ap+k
i i
distincte două câte două. Pentru orice k ∈ N∗ notăm sk = aki s, i tk =
P P
.
i=1 i=1 a i − 1
Rezolvat, i ı̂n Rn sistemul


 s1 x1 + s2 x2 + . . . + sn xn = t1
s2 x1 + s3 x2 + . . . + sn+1 xn = t2

.

 ...
sn x1 + sn+1 x2 + . . . + s2n−1 xn = tn

Stelian Corneliu Andronescu, Costel Bălcău, Pites, ti s, i


Leonard Mihai Giugiuc, Drobeta Turnu Severin

M 135. Demonstrat, i că s, irul (xn )n≥1 definit prin

2 π 3 3π 4 5π n+1 (2n − 1)π


2Cn+1 cos + 3Cn+1 cos + 4Cn+1 cos + . . . +(n + 1)Cn+1 cos
xn = 2n + 1 2n + 1 2n + 1 2n + 1
π 3π 5π (2n − 1)π
Cn1 sin + Cn2 sin + Cn3 sin + . . . + Cnn sin
2n + 1 2n + 1 2n + 1 2n + 1
este convergent s, i calculat, i limita sa.

Mihai Florea Dumitrescu, Potcoava s, i Costel Bălcău, Pites, ti



X k2 1
M 136. Arătat, i că 2k
≤ .
k=1
e 4

Sladjan Stankovik, Macedonia de Nord

M 137. Fie S suma solut, iilor din intervalul (0, 2π) ale ecuat, iei (tg x)tg x = 3sin x .
 ‹
4π 3π
Arătat, i că S∈ , .
3 2

Daniel Jinga, Pites, ti



 x

+y+z =3
‹ ‹ ‹ √
3
M 138. Rezolvat, i ı̂n R sistemul 1 1 1 169 − 15 5 .
 x2 + y2 + z2 + =
2 2 2 64

Florentin Vis, escu, Bucures, ti


98 PROBLEME DE MATEMATICĂ PENTRU CONCURSURI

Clasa a XII-a

M 139. Arătat, i că nu există n ∈ N∗ \ {1} astfel ı̂ncât inelul claselor de resturi (Zn , +, ·) să aibă
exact 2020 de elemente inversabile.

Stelian Corneliu Andronescu s, i Costel Bălcău, Pites, ti

M 140. Fie n ∈ N, n ≥ 2 s, i a, b, c, d ∈ C, a 6= 0. Demonstrat, i că cel mai mare divizor comun al


polinoamelor f = X n + aX + b s, i g = X n+1 + cX + d este un polinom de grad mai mic sau egal
cu 2.

Cristinel Mortici, Viforâta

M 141. Fie a ∈ R, a ≥ 1. Calculat, i integrala


√ € p Š
Z 2a−1 1+ x2 − 2ax + a2 + 1 1+ 3
(x − 1)(x − a)(x − 2a + 1) cos πx
I= dx.
1 x2 − 2ax + a2 + 1

Marin Chirciu, Pites, ti

M 142. Fie f : [0, 2] → [0, ∞) o funct, ie derivabilă s, i crescătoare. Arătat, i că


Z 2 Z 2 Z 2
2
4 f (x) dx ≤ 4 x f (x) dx − x2 f (x) dx.
1 1 0

Daniel Jinga, Pites, ti

M 143. Fie a, b, c s, i d numere reale nenegative. Arătat, i că


É
3 abc + abd + acd + bcd
qX
2
(a − b) + 4 ≥ a + b + c + d,
4
(a − b)2 = (a − b)2 + (a − c)2 + (a − d)2 + (b − c)2 + (b − d)2 + (c − d)2 .
P
unde
Când are loc egalitatea?

Leonard Mihai Giugiuc, Drobeta Turnu Severin

(a + b + 2)2
M 144. Fie funct, ia f : R2 → R, f (a, b) = . Determinat, i valorile k ∈ R pentru care
a2 + b2 + 2
inegalitatea
(a + b + 2)2 (ab + 2a + 2b + 1)2 ≥ 144ab(a2 + b2 + 2)
are loc pentru orice numere reale a s, i b astfel ı̂ncât f (a, b) = k.

Michael Rozenberg, Israel s, i Leonard Mihai Giugiuc, România


PROBLEME DE INFORMATICĂ PENTRU CONCURSURI 99

PROBLEME DE INFORMATICĂ PENTRU


CONCURSURI

Probleme propuse

Clasa a IX-a

I 61 (numar prim). Se dă N număr natural cu maxim 100 de cifre. Se cere să se determine
cel mai mare număr mai mic sau egal cu N care are suma cifrelor un număr prim.
Exemplu
Pentru N = 1107 se va afis, a: 1105.
Doru Anastasiu Popescu, Pites, ti
I 62 (deosebit). Un număr natural cu cifre distincte se numes, te deosebit dacă fiecare cifră a
sa divide suma celorlalte.
Exemple
246 este număr deosebit pentru că: 2 divide 4 + 6, 4 divide 2 + 6, 6 divide 2 + 4.
1236 este număr deosebit pentru că: 1 divide 2 + 3 + 6, 2 divide 1 + 3 + 6, 3 divide 1 + 2 + 6, 6
divide 1 + 2 + 3.
Cerint, ă
Pentru un număr natural dat, cu cifre distincte, se cere să se verifice dacă este sau nu deosebit.
De exemplu, pentru numărul 1236 se va afis, a DA, iar pentru numărul 1246 se va afis, a NU.
Marian Teler, Costes, ti
I 63 (spadă). La un turneu de spadă s-au ı̂nscris n spadasini. Organizatorii turneului au
hotărât că fiecare spadasin trebuie să se dueleze cu tot, i ceilalt, i, dar o singură dată pe zi (ı̂n
scopul dozării efortului). Turneul trebuie să se desfăs, oare ı̂ntr-un număr minim de zile (pentru
a nu se exagera cu cheltuielile de cazare) s, i nu există zi liberă pentru niciun spadasin.
Cerint, ă
Să se determine modul de organizare a turneului, adică ce dueluri se desfăs, oară ı̂n fiecare zi.
Spadasinii sunt identificat, i după numerele de ordine 1, 2, . . . , n.
Date de intrare
Numărul natural nenul n se găses, te pe prima linie a fis, ierului de intrare spada.in s, i este
număr par.
Date de ies, ire
Fis, ierul de ies, ire spada.out va cont, ine modul de desfăs, urare a turneului. Dacă pentru
acesta sunt necesare k zile atunci ı̂n fis, ier se vor găsi k perechi de linii. Prima linie a fiecărei
perechi cont, ine cuvântul ”Ziua” urmat de un număr de ordine al zilei, separate de un spat, iu.
Prima zi are numărul de ordine 1 iar cuvântul ”Ziua” ı̂ncepe cu o majusculă.
Cea de-a doua linie a perechii cont, ine duelurile care se desfăs, oară ı̂n ziua respectivă. Sintaxa
100 PROBLEME DE INFORMATICĂ PENTRU CONCURSURI

este:
spadasin1 – spadasin2 spadasin3 – spadasin4 ... spadasinn−1 – spadasinn
unde spadasini este o valoare din mult, imea {1, 2, . . . , n}. Cei doi spadasini care sunt la stânga
s, i la dreapta caracterului ”-” (adică minus) reprezintă cei doi spadasini care se vor duela.
Caracterul ,,-” este precedat s, i urmat de câte un spat, iu iar perechile de spadasini care dispută
duelul, de asemenea sunt separate prin câte un spat, iu.
Restrict, ii s, i precizări
• 2 ≤ n ≤ 100
• n este număr par
Exemplu
spada.in spada.out Explicat, ie
4 Ziua 1 Sunt necesare 3 zile,
1-32-4 o modalitate de organizare
Ziua 2 fiind cea prezentată.
1-23-4
Ziua 3
1-42–3
Timp maxim de execut, ie: 0.1 sec./test. Memorie totală disponibilă 8 MB.
Marin Cristea, Pites, ti
I 64 (cifra). Se dă un tablou pătratic de dimensiune n cu elemente numere naturale. Determinat, i
cifra k care se găses, te ı̂n cât mai multe numere din tablou. Dacă există mai multe astfel de
cifre, k va fi cea mai mare dintre ele.
Cerint, ă
Dându-se n s, i un tablou pătratic de dimensiune n cu elemente numere naturale, se cere k cu
proprietăt, iile din enunt, .
Date de intrare
Numărul natural nenul n se găses, te pe prima linie a fis, ierului de intrare cifra.in, iar
elementele tabloului pe n linii, separate prin câte un spat, iu.
Date de ies, ire
Fis, ierul de ies, ire spada.out va cont, ine cifra k cu proprietăt, iile din enunt, .
Restrict, ii s, i precizări
• 1 ≤ n ≤ 100
• Elemente tabloului sunt numere naturale cu maxim 17 cifre
Exemplu
cifra.in cifra.out Explicat, ie
3 5 Cifrele 2 si 5 se găsesc ı̂ntr-un număr
23 1055 71 maxim de elemente, rezultă k = 5
85623 99 5522 pentru că 5 este cel mai mare dintre 2 s, i 5.
22 55 525
Timp maxim de execut, ie: 0.1 sec./test. Memorie totală disponibilă 2 MB.
Ion Alexandru Popescu, Bucures, ti
PROBLEME DE INFORMATICĂ PENTRU CONCURSURI 101

I 65 (FIBO). Fie un tablou bidimensional cu m linii s, i n coloane, iar elementele sunt numere
naturale. Determinat, i indicele cel mai mic de linie cu proprietatea FIBO (notat cu k). Indicii
liniilor incep cu 1, iar o linie are proprietatea FIBO dacă toate elementele sale sunt termeni
din s, irul lui Fibonacci (1, 1, 2, 3, 5, 8, . . .).
Cerint, ă
Pentru un tablou bidimensional cu m linii s, i n coloane dat, se cere k cu proprietatea din
enunt, .
Date de intrare
Fis, ierului de intrare FIBO.in cont, ine pe prima linie m s, i n separate printr-un spat, iu, iar pe
următoarele m linii elementele tabloului separate prin câte un spat, iu.
Date de ies, ire
Fis, ierul de ies, ire FIBO.out va cont, ine cifra k cu proprietatea din enunt, .
Restrict, ii s, i precizări
• 1 ≤ m, n ≤ 40
• Elemente tabloului sunt numere naturale cu maxim 17 cifre
Exemplu
FIBO.in FIBO.out Explicat, ie
3 3 2 Prima linie numai
18 55 71 cu elemente din s, irul
1 13 2 lui Fibonacci este
258 cu indicele 2.

Timp maxim de execut, ie: 0.1 sec./test. Memorie totală disponibilă 2 MB.
Ion Alexandru Popescu, Bucures, ti

Clasa a X-a

I 66 (similar). Definit, ie: Două cuvinte distincte sunt similare dacă au acelas, i număr de
litere distincte s, i numărul de vocale distincte din component, a lor este un număr par.
De exemplu, cuvintele care s, i arce sunt cuvinte similare, deoarece au acelas, i număr de litere
distincte s, i au două vocale distincte. La fel, cuvintele ani s, i anii sunt cuvinte similare.
Cerint, ă
Dându-se un text pe mai multe linii, determinat, i toate grupările de cuvinte distincte similare
din text. O grupare poate avea minim două cuvinte. Cuvintele textului sunt despărt, ite prin:
virgulă (,), spat, iu (‘ ‘), punct s, i virgulă (;) sau punct (.).
Date de intrare
Din fis, ierul text similar.in se cites, te textul dat. Textul dat poate fi pe mai multe linii ale
fis, ierului text, iar fiecare linie a textului se termină cu caracterul ENTER.
102 PROBLEME DE INFORMATICĂ PENTRU CONCURSURI

Date de ies, ire


În fis, ierul de ies, ire similar.out se va afis, a pe prima linie un singur număr natural ng,
reprezentând numărul de grupări de cuvinte similare din textul dat, iar pe următoarele ng linii
ale fis, ierului se vor afis, a grupările de cuvinte similare, pe fiecare linie a fis, ierului afis, ându-se
cuvintele unei singure grupări despărt, ite prin câte un spat, iu. Grupările de cuvinte similare se
vor afis, a ı̂n ordinea aparit, iilor din text.
Restrict, ii s, i precizări
• Fiecare linie a textului poate cont, ine maxim 300 de caractere, fiecare cuvânt din text are
maxim 30 de caractere, iar textul cont, ine cel mult 100 de linii
• Textul va cont, ine doar litere mici ale alfabetului englez
ar litere mici ale alfabetului englez.
Exemplu
similar.in similar.out Explicat, ie
cei mai frumosi ani sunt anii de liceu. 3 Cuvântul are se
care are arce multe are si ace la fel de multe. ani anii găses, te de
ion are un cojoc nou, dar are s, i ghete noi. care arce două ori ı̂n
ion noi textul dat, dar
fiind două
cuvinte identice,
ele nu pot forma
o grupare de
cuvinte similare.
Timp maxim de execut, ie: 0.2 sec./test. Memorie totală disponibilă 3 MB.
Adriana Oprea, Pites, ti
I 67 (codtel). Un telefon are un cod de intrare format din patru cifre. Codul este format
din primele 4 cifre ale unui număr natural X. Dându-se un cuvânt, notat cu s, X reprezintă
numărul de submult, imi ale mult, imii literelor cuvântului dat. Dacă X are mai put, in de 4 cifre,
atunci ı̂n fat, a lui se vor pune atâtea cifre de 0 cât este nevoie.
Cerint, ă
Pentru un cuvânt s dat, determinat, i cele 4 cifre ale codului de intrare ı̂n telefon.
Date de intrare
Fis, ierul de intrare codtel.in cont, ine pe prima linie cuvântul s.
Date de ies, ire
Fis, ierul de ies, ire codtel.out va cont, ine pe prima linie secvent, a de cifre necesară deschiderii
telefonului.
Restrict, ii s, i precizări
• Cuvântul s are maxim 1000 de litere
• Cuvântul s este format numai din litere mici
PROBLEME DE INFORMATICĂ PENTRU CONCURSURI 103

Exemplu
codtel.in codtel.out
valea 0016

Timp maxim de execut, ie: 0.1 secunde/test. Memorie totală disponibilă 2 MB.
Costel Bălcău, Pites, ti

I 68 (codificare). Pentru transmiterea unor texte codificate se folosesc literele din configurat, ia
de mai jos:

Fig. 1: As, ezarea literelor pentru codificare


În această configurat, ie se formează cinci pătrate pe care le vom numerota astfel: 1 pătratul de
sus, 2 pătratul din stânga, 3 pătratul din mijloc, 4 pătratul din dreapta, respectiv 5 pătratul de
jos. Pentru pătratele configurat, iei se stabiles, te o ordine s, i un sens de rotire al fiecăruia.
Codificarea unui text se face prin codificarea fiecărui caracter ı̂n parte, ı̂n ordinea ı̂n care apar
ı̂n text, astfel:
- Înainte de codificarea fiecărui caracter, caracterele fiecărui pătrat din configurat, ia anterioară se
rotesc, ı̂n ordinea s, i sensurile precizate, cu o pozit, ie. Init, ial se pornes, te de la configurat, ia dată.
- Ulterior, pentru codificarea unui caracter se va folosi configurat, ia rezultată după codificarea
caracterului anterior.
- Prin codificare, unui caracter din configurat, ia init, ială ı̂i va corespunde caracterul de pe aceeas, i
pozit, ie din configurat, ia finală.
Cerint, ă
Pentru un text, care cont, ine cel mult 1000 de caractere existente ı̂n configurat, ia dată, se
cere sa se obt, ină codificarea acestuia.
Date de intrare
Fis, ierul de intrare codificare.in cont, ine 3 linii:
p1 p2 p3 p4 p5 - numerele asociate pătratelor ı̂n ordinea rotirii lor
s1 s2 s3 s4 s5 - sensul de rotire al caracterelor p1, p2, p3, p4, p5 (1 - rotire spre dreapta, 0 -
rotire spre stânga)
textul - secvent, a de caractere ce urmează a fi codificată
Date de ies, ire
Fis, ierul de ies, ire codificare.out va cont, ine pe o singură linie textul codificat.
Restrict, ii s, i precizări
• Textul pentru codificare are maxim 1000 de caractere, aflate ı̂n figura de mai sus
104 PROBLEME DE INFORMATICĂ PENTRU CONCURSURI

Exemplu
codificare.in codificare.out
12453 Bafta
11011
Cbfxb
Timp maxim de execut, ie: 0.1 sec./test. Memorie totală disponibilă 2 MB.
Doru Anastasiu Popescu, Pites, ti
I 69 (permnefixe). Se dau n s, i k două numere naturale. Prin permutare fară puncte fixe
ı̂nt, elegem o permutare a mult, imii {1, 2, . . . , n} ı̂n care orice element al ei este diferit de pozit, ia
pe care se află. De exemplu pentru n = 4 permutările (2, 1, 4, 3) s, i (3, 1, 4, 2) nu au puncte fixe.
Dintre toate permutările fără puncte fixe ne interesează a k-a permutare a mult, imii ı̂n ordine
lexicografică.
Cerint, ă
Pentru n s, i k date se cere a k-a permutare fără puncte fixe ı̂n ordine lexicografică.
Date de intrare
Fis, ierul de intrare permnefixe.in cont, ine pe prima linie n s, i k separate printr-un spat, iu.
Date de ies, ire
Fis, ierul de ies, ire permnefixe.out va cont, ine pe prima linie a k-a permutare fără puncte
fixe ı̂n ordine lexicografică; elementele ei vor fi separate prin câte un spat, iu. Dacă nu există o
astfel de permutare se va scrie cifra 0.
Restrict, ii s, i precizări
• 1 ≤ n ≤ 20
• 1 ≤ k ≤ 200
Exemplu
permnefixe.in permnefixe.out
42 2341
Timp maxim de execut, ie: 0.1 sec./test. Memorie totală disponibilă 2 MB.
Ion Alexandru Popescu, Bucures, ti
I 70 (zerouri). Se dă n, număr natural. Notăm cu N r numărul de funct, ii bijective de forma
f : {1, 2, ..., n} → {1, 2, ..., n}.
Cerint, ă
Pentru n dat afis, at, i numărul de zerouri ı̂n care se termină N r.
Date de intrare
Fis, ierul de intrare zerouri.in cont, ine pe prima linie n.
Date de ies, ire
Fis, ierul de ies, ire zerouri.out va cont, ine numărul de zerouri ı̂n care se termină N r.
PROBLEME DE INFORMATICĂ PENTRU CONCURSURI 105

Restrict, ii s, i precizări
• 1 ≤ n ≤ 10000
Exemplu
zerouri.in zerouri.out
23 4
Timp maxim de execut, ie: 0.1 sec./test. Memorie totală disponibilă 2 MB.
Doru Constantin, Pites, ti

Clasele XI-XII

I 71 (bijuterii). Doi hot, i au ajuns la sapă de lemn, motiv pentru care se hotărăsc să spargă
singura bancă din oras, . În momentul implementării planului, ei constată ı̂nsă că ı̂n loc de
bani au găsit bijuterii. Cum cei doi aveau nevoie de bani, au hotărât să ducă toate bijuteriile
altei persoane cu preocupări asemănătoare. După o evaluare temeinică a bijuteriilor conform
cerint, elor piet, ei, ei au primit o listă a pret, urilor pentru fiecare bijuterie.
Cerint, ă
Ajutat, i cei doi hot, i aflat, i ı̂n pragul disperării să ı̂s, i ı̂mpartă cât mai egal sumele obt, inute pe
bijuterii.
Date de intrare
Fis, ierul de intrare bijuterii.in cont, ine pe prima linie numărul n de bijuterii iar pe cea
de-a doua linie pret, urile bijuteriilor.
Date de ies, ire
Fis, ierul de ies, ire bijuterii.out va cont, ine pe prima linie două valori ce reprezintă suma
pret, urilor obt, inute de cei doi hot, i. Pe următoarea linie se găsesc pret, urile bijuteriilor primului
hot, iar pe ultima linie pret, urile celui de-al doilea hot, .
Restrict, ii s, i precizări
• Numărul maxim de bijuterii este 800
• Pret, ul maxim al unei bijuterii este 1000
• Cele două liste de pret, uri din fis, ierul de ies, ire vor fi ı̂n ordine crescătoare
• Primul hot, va primi o sumă cel mult egală cu cea a prietenului lui
Exemplu
bijuterii.in bijuterii.out
7 48 49
28 7 11 8 9 7 27 9 11 28
7 7 8 27
Timp maxim de execut, ie: 0.4 sec./test. Memorie totală disponibilă 32 MB.
Marin Cristea, Pites, ti
106 PROBLEME DE INFORMATICĂ PENTRU CONCURSURI

I 72 (seif ). Un seif se poate deschide cu ajutorul unei secvent, e de cifre. Secvent, a de cifre este
formată din cifrele unui număr natural N r. Dându-se un cuvânt, notat cu c, N r reprezintă
numărul de cuvinte de lungime maximă formate numai cu litere distincte ale cuvântului dat.
Cerint, ă
Pentru un cuvânt c dat, determinat, i secvent, a de cifre necesară deschiderii seifului (cifrele lui
N r).
Date de intrare
Fis, ierul de intrare seif.in cont, ine pe prima linie cuvântul c.
Date de ies, ire
Fis, ierul de ies, ire seif.out va cont, ine pe prima linie secvent, a de cifre necesară deschiderii
seifului.
Restrict, ii s, i precizări
• Cuvântul c are maxim 1000 litere
• Cuvântul c este format numai din litere mici
Exemplu
seif.in seif.out
sinaia 24

Timp maxim de execut, ie: 0.1 sec./test. Memorie totală disponibilă 2 MB.
Doru Constantin, Pites, ti
I 73 (pavari). Se dă un dreptunghi cu lungimea egală cu 2N decimetri s, i lăt, imea egală cu 3
decimetri.
Cerint, ă
Să se determine numărul M al pavărilor distincte ale dreptunghiului dat cu dale dreptun-
ghiulare care au lăt, imea egală cu un decimetru s, i lungimea egală cu 2 decimetri.
Date de intrare
Fis, ierul de intrare pavari.in cont, ine pe prima linie numărul N , reprezentând jumătatea
lungimii dreptunghiului.
Date de ies, ire
Fis, ierul de ies, ire pavari.out va cont, ine numărul M reprezentând numărul modalităt, ilor de
a pava dreptunghiul.
Restrict, ii s, i precizări
• 1 ≤ N ≤ 100
PROBLEME DE INFORMATICĂ PENTRU CONCURSURI 107

Exemplu
pavari.in pavari.out
2 11

Timp maxim de execut, ie: 0.1 sec./test. Memorie totală disponibilă 2 MB.
Doru Anastasiu Popescu, Pites, ti

I 74 (drum). În regatul XLand oras, ele erau ı̂nconjurate de ziduri ı̂n formă de poligoane convexe.
Împăratul a dispus construirea unui drum de legătură directă ı̂ntre capitală s, i un alt oras, dat.
Fiecare extremitate a drumului poate fi orice punct situat pe zidul oras, ului, respectiv capitalei.
Lungimea drumului este distant, a dintre extremităt, ile sale.
Cerint, ă
Determinat, i cel mai scurt drum dintre capitală s, i oras, ul dat.
Date de intrare
Fis, ierul de intrare drum.in cont, ine:
Linia 1: K1 - număr natural nenul, reprezentând numărul de colt, uri ale zidurilor capitalei;
Linia 2: x1 y1 x2 y2 ... xK1 yK1 - K1 perechi de numere ı̂ntregi, separate prin câte un spat, iu,
reprezentând coordonatele vârfurilor zidurilor capitalei;
Linia 3: K2 - număr natural nenul, reprezentând numărul de colt, uri ale zidurilor oras, ului dat;
Linia 4: x1 y1 x2 y2 ... xK2 yK2 - K2 perechi de numere ı̂ntregi, separate prin câte un spat, iu,
reprezentând coordonatele vârfurilor zidurilor acestui oras, .
Date de ies, ire
Fis, ierul de ies, ire drum.out va cont, ine o singură linie cu numerele:
x1 y1 x2 y2 - patru numere reale trunchiate la 4 zecimale, separate prin câte un spat, iu,
reprezentând extremităt, ile unui drum minim.
Restrict, ii s, i precizări
• 1 ≤ K1, K2 ≤ 20
• Coordonatele vârfurilor zidurilor ce ı̂nconjoară oras, ul, respectiv capitala sunt numere
ı̂ntregi apart, inând intervalului [-100,100] s, i sunt date fie ı̂n ordinea deplasării acelor de
ceasornic, fie ı̂n sens invers deplasării acelor de ceasornic
• Capitala s, i oras, ul nu au niciun punct comun (nu au puncte interioare comune s, i nu au
puncte comune pe zidurile lor)

Exemplu
drum.in drum.out
4 5 3.5 8 3.5
34325254
4
8 3 8 6 11 6 11 3

Timp maxim de execut, ie: 0.1 sec./test. Memorie totală disponibilă 2 MB.
Doru Anastasiu Popescu, Pites, ti
108 PROBLEME DE INFORMATICĂ PENTRU CONCURSURI

I 75 (arbori). Se dă un graf neorientat prin numărul de noduri n, numărul de muchii m s, i m


perechi de numere reprezentând extremităt, ile muchiilor.
Cerint, ă
Verificat, i dacă graful dat este bipartit complet s, i ı̂n caz afirmativ determinat, i numărul de
arbori part, iali modulo 9991.
Date de intrare
Fis, ierul de intrare arbori.in cont, ine pe prima linie n si m separate printr-un spat, iu, iar
pe următoarele m linii perechi de numere separate prin câte un spat, iu, reprezentând muchiile
grafului neorientat.
Date de ies, ire
Fis, ierul de ies, ire arbori.out va cont, ine pe prima linie unul dintre cuvintele DA sau NU, ı̂n
funct, ie de situat, ia graf bipartit complet sau graf care nu este bipartit complet. În cazul ı̂n care
pe prima linie s-a scris DA, atunci pe linia a doua se va scrie numărul de arbori part, iali modulo
9991.
Restrict, ii s, i precizări
• 1 ≤ n ≤ 100
Exemplu
arbori.in arbori.out
56 DA
12 12
14
15
32
43
35
Timp maxim de execut, ie: 0.1 sec./test. Memorie totală disponibilă 2 MB.
Costel Bălcău, Pites, ti
ISTORIOARE DIN LUMEA MATEMATICII S, I A
INFORMATICII

Carl Friedrich Gauss, ,,. . .teoria numerelor este regina


matematicii”

Stelian Corneliu Andronescu 1

Carl Friedrich Gauss s-a născut la 30 aprilie 1777 ı̂n Braunschweig s, i a decedat la 23 februarie
1855 ı̂n Göttingen. Genialul matematician a spus: ,,matematica este regina s, tiint, elor, iar teoria
numerelor este regina matematicii”, un adevăr us, or de acceptat s, i greu de contestat.
Vom puncta câteva aspecte din viat, a s, i din deosebita sa activitate s, tiint, ifică.
Un adevărat geniu ı̂n matematică, Gauss s, tia să adune ı̂ncă de la vârsta de trei ani, când a
ı̂nceput să corecteze socotelile tatălui său. C.F. Gauss a fost nevoit să ı̂nvet, e singur calculul
algebric, deoarece părint, ii săi nu aveau posibilitatea de a se ocupa de educat, ia lui. În 1792,
cu aprobare de la Ducele de Braunschweig, a intrat la Colegiul Carolinum, iar ı̂n 1795 la
Universitatea Göttingen.
Marele matematician a scris lucrări ı̂n mai multe domenii, precum matematică s, i fizică, dar
s, i geodezie s, i astronomie. Lui Gauss ı̂i datorăm metoda ,,construirii unui poligon cu 17 laturi
folosind numai rigla s, i compasul”. Opera sa se axează pe teoria numerelor, fiind considerat
creatorul acestui domeniu. A studiat teoria conguent, elor făcând distinct, ie ı̂ntre congruent, ele
algebrice s, i cele transcendente s, i indicând o metodă directă pentru rezolvarea congruent, elor
binome; de asemenea a dat prima demonstrat, ie riguroasă a legii de reciprocitate pătratică
ı̂n anul 1825. În algebră, ı̂n teza de doctorat a demonstrat teorema fundamentală a algebrei.
În anul 1801 a publicat cartea ,,Disquisitiones Arithmeticae”, un studiu sistematic al teoriei
numerelor, lucrare ce a stat la bazele dezvoltării moderne a acestei teorii.
În lumea matematicii a rămas faimos următorul dialog de la distant, ă dintre Carl Fiederich
Gauss s, i Niels Henrich Abel. Abel spune despre Gauss că este ca vulpea care ı̂s, i s, terge urmele
din nisip cu coada. Replica lui Gauss a fost: niciun arhitect care se respectă nu lasă schela după
ce s, i-a finalizat clădirea.
În anul 1807, genialul matematician va pleca la Göttingen, preluând postul de Director al
Observatorului, motivul plecării fiind uciderea ı̂n luptă a Ducelui de Braunschweig. Redăm ı̂n
continuare citatul din prefat, a cărt, ii ,,Disquisitiones Arithmeticae”, din care reiese recunos, tint, a
lui Gauss fat, ă de binefăcătorul său, dar s, i pasiunea extraordinară pentru matematică.

SERENISIMULUI PRINT, S, I LORD CHARLES WILLIAM FERDINAND


DUCE DE BRAUNSCHWEIG Şl LÜNEBURG
PREALUMINATE PRINŢ,
1
Lect. univ. dr., Universitatea din Pites, ti, corneliuandronescu@yahoo.com

109
110 S.C. Andronescu

Consider că cel mai mare noroc al meu este că ı̂mi permitet, i să ı̂nnobilez lucrarea mea cu prea
onorabilul DUMNEAVOASTRĂ nume. Fără favoarea pe care mi-at, i acordat-o, Prealuminate
Print, , eu nu as, fi avut s, ansa de a deveni om de s, tiint, ă. Fără ajutorul DUMNEAVOASTRĂ
inestimabil ı̂n sust, inerea studiilor mele nu as, fi avut posibilitatea de a mă dedica ı̂n ı̂ntregime
dragostei mele pasionale, studiul matematicii. Mărinimia DUMNEAVOASTRĂ m-a scutit de
orice griji, permit, ându-mi să dedic mult, i ani studiilor s, i meditat, iilor ı̂n domeniul matematicii s, i, ı̂n
final, oferindu-mi prilejul de a prezenta semenilor mei, prin volumul de fat, ă, câteva din rezultatele
cercetărilor mele. Mai mult decât atât, generozitatea DUMNEAVOASTRĂ a fost aceea care a
ı̂nlăturat s, i obstacolele ce ı̂ntârziau, nejustificat, publicarea ei. Mărinimiei DUMNEAVOASTRĂ,
care mă coples, es, te s, i ı̂ndatorează mai mult decât pot spune, nu-i pot răspunde pe măsură prin
această lucrare; dar râvna mea ı̂n a ı̂ncerca să precumpănesc Generozitatea DUMNEAVOASTRĂ
nu se va opri aici.
Ştim cu tot, ii că nu at, i văduvit de ı̂nalta DUMNEAVOASTRĂ protect, ie nicio s, tiint, ă, nici
măcar pe acelea care, ı̂n ochii unora, par prea abstracte s, i ı̂ndepărtate de viat, a de zi cu
zi, deoarece DUMNEAVOASTRĂ ı̂ns, ivă, prin suprema DUMNEAVOASTRĂ ı̂nt, elepciune,
suntet, i un excelent cunoscător al legăturilor intime ce unesc toate s, tiint, ele, s, tiint, e care permit
prosperitatea societăt, ii omenes, ti. Publicarea acestei cărt, i se vrea deci o dovadă a profundului
respect pe care Vi-l port, un omagiu personal celei mai frumoase s, tiint, e. Prealuminate Print, , dacă
vet, i avea bunăvoint, a să considerat, i această lucrare demnă de extraordinara DUMNEAVOASTRĂ
favoare, cu care m-at, i onorat, strădaniile mele sunt răsplătite mai mult decât mi-as, fi dorit
vreodată.
PREALUMINATE PRINŢ
Cel mai consecvent slujitor al Altet, ei Voastre
C.F. Gauss

Braunschweig, iulie 1801

Bibliografie

[1] Carl Friedrich Gauss, Cercetări Aritmetice, Traducere de C.I. Giurescu, Ed. Amarcord,
Timis, oara, 1999.
Revistă sponsorizată de
ROWEB Development SRL
s, i
OSF Global Services SRL

S-ar putea să vă placă și